Sie sind auf Seite 1von 736

Q)

Q) Sq cell CA accounts for approxly how many


cases of vulval CA ? 90%
Q) Chance of malignant transformation occurring
in premenopausal benign ovarian cysts is Mar15 10%
Q) Adrenal Crisis will present with shock syndromes in
combination with Hyponatremia, Hyperkalemia and
Hypoglycemia
Primary Mineralocorticoid Excess (Conns) can be due to
tumor of Zona Glomerulosa, which will cause Hypokalemia,
Bicarbonate Retention and Sodium Retention,alkalosis
CAH 21-OH defcy
hyperkalemia,hyponatremia,hypoglyc,met acidosis
Addisons Aldosterone def leads to nonanion gap hyperkalemic, hyponatremic
,Hypercalcemia,uremia,eosinophilia,metabo
lic acidosis.
SIADH hyponatremia, low serum
osmolarity and inapropriately high urine
osmolarity.

Q)
Q)
Q)
Q)
Q)
Q)
Q)
Q)
Q)

Q)
Q)
Q)
Q)
Q)
Q)
Q)
Q)
Q)
Q)
Q)
Q)
Q)
Q)
Q)
Q)
Q)
Q)
Q)
Q)

Q)
Q)
Q)
Q)
Q)
Q)
Q)
Q)
Q)
Q)
Q)
Q)
Q)
Q)
Q)
Q)
Q)
Q)
Q)
Q)

Q)
Q)
Q)
Q)
Q)
Q)
Q)
Q)
Q)
Q)
Q)
Q)
Q)
Q)
Q)
Q)
Q)
Q)
Q)
Q)

Q)
Q)
Q)
Q)
Q)
Q)
Q)
Q)
Q)
Q)
Q)
Q)
Q)
Q)
Q)
Q)
Q)
Q)
Q)
Q)

Q)
Q)
Q)
Q)
Q)
Q)
Q)
Q)
Q)
Q)
Q)
Q)
Q)
Q)
Q)
Q)
Q)
Q)
Q)
Q)

Q)
Q)
Q)
Q)
Q)
Q)
Q)
Q)
Q)
Q)
Q)
Q)
Q)
Q)
Q)
Q)
Q)
Q)
Q)
Q)

Q)
Q)
Q)
Q)
Q)
Q)
Q)
Q)
Q)
Q)
Q)
Q)
Q)
Q)
Q)
Q)
Q)
Q)
Q)
Q)

Q)
Q)
Q)
Q)
Q)
Q)
Q)
Q)
Q)
Q)
Q)
Q)
Q)
Q)
Q)
Q)
Q)
Q)
Q)
Q)

Q)
Q)
Q)
Q)
Q)
Q)
Q)
Q)
Q)
Q)
Q)
Q)
Q)
Q)
Q)
Q)
Q)
Q)
Q)
Q)

Q) The following combinations of buffer system and body compartment of


major action is correct, except!? A. Plasma - bicarbonate
B. RBC - bicarbonate ..ans ,Hb
C. Intracellular - protien
D. Urine - phosphate
E. RBC - Hb

Q) ou see a 55 year old lady in menopause clinic. Taking a history she tells you she gets
general aches and pains and has been worried about diabetes as she is constantly thirsty.
Otherwise she has no significant past medical history and is on no regular medication. You
organise bloods and the results are below. What is the likely diagnosis? Na 135 mmol/l
K3.8 mmol/l
Ur 7.6 mmol/l
Cr 82 mmol/l
ESR 15 mm/h
PTH 80 pg/ml (HIGH)
HBA1C 38 mmol/mol
Adjusted calcium 2.76 mol/l (HIGH)
Parathyroid adenoma
Secondary hyperparathyroidism
Tertiary hyperparathyroidism
MEN type 1
MEN type 2
High PTH with hypercalcaemia is seen in Primary Hyperparathyroidism.
Although tumours of the parathyroid occur in MEN, parathyroid adenoma accounts for 97%
of cases of primary hyperparathyroidism.
High PTH with hypercalcaemia is also seen in tertiary hyperparathyroidism. Tertiary
hyperparathyroidism usually follows a long period of secondary hyperparathyroidism (most
commonly due to renal failure). This patient has no medical history of being unwell or having
had a long period of renal impairment.
Parathyroid
Hyperparathyroidism
Primary Hyperparathyroidism
Excessive parathyroid hormone production by parathyroid adenoma.
Causes hypercalcaemia
Secondary Hyperparathyroidism
Secondary to hypocalcaemia

PTH rises to try and correct calcium


Causes:
Chronic renal failure
Vitamin D Deficiency (which leads to hypocalcaemia. Several causes)
Tertiary Hyperparathyroidism
After long periods of excessive PTH secretion (i.e. long periods of secondary
hyperparathyroidism) the parathyroid gland secretes PTH autonomously even if the cause of
secondary hyperparathyroidism is corrected

Q) Vitamin K is essential for the synthesis of - Vitamin A


Vitamin D
Clotting factor III
Clotting factor IV
Protein S.ans
Vitamin K
Vitamin K is a fat soluble vitamin that is stored in the liver and adipose tissues. Other fat
soluble vitamins are A, D and E.
It is essential for the synthesis of:
- Clotting factors (X)10,(IX) 9, 7(VII), 2 (II) (pneumonic 1972)
- Proteins C, S and Z
- Osteocalcin and GLA proteins

Q) Which one of the following statements concerning breast cancer is correct?


A A GP with 2,500 patients will see on average 2 new cases per year This is the correct
answer
B Alpha-fetoprotein is typically raised above normal
C Surgery has been shown to improve survival rates more than radiotherapy
D The five year survival of all patients with breast cancer is less than 50%
E The incidence has been shown to be higher in women who have had previous bilateral
oophorectomy for non-malignant disease
The annual incidence of breast cancer in the UK is 157 per 100,000 women (1.57 per 1000)
and a GP will see on average 10 new diagnoses of any cancer per year.
Alpha-fetoprotein is raised in cancer of the:
Liver
Testis
Ovary
Pancreas
Lung

Stomach, and
Colon.
Current figures suggest an 80% five year survival of all patients with breast cancer

Q)

Ans D

Excessive fluid intake is obvious, it's not the interpretation. Sensory function
can't be interpreted from bladder diary. For cystitis you need a urine culture. It's
not urge incontinence as she has this only once. It's overactive bladder

Q) background radiation exposure of DEXA


equivalent to? 3hr..ans
3day
1month
3month
Q) In a woman with one previous molar pregnancy, the risk of
having another molar pregnancy is- A 1 in 10
B 1 in 20
C 1 in 50
D 1 in 80ans
E 1 in 300
Recurrence risk of molar pregnancy = 1:80; 1:6.5
after two previous molar pregnancies.
Q) Chlamydia trachomatis is present in 11% of
the sexually active population aged 19 years
It is a major cause of PID, leading to chronic
abdominal pain, ectopic pregnancy and
tubal factor infertility
The prevalence of C. trachomatis in sub-fertile
women in the UK is 1.9%
Q)What is incidence rate of ovarian CA per 100,000 females
in UK? 2
7
15

22.ans
37
1 in 1000
3in 1000 ...in 100000
Endometrial is 18/100,000
Vulval is 1.8/100,000
Q) dome of bladder from yolk sac
-trigone from mesonephric duct
-bladder from urogenital sinus
-ureter until collecting duct are from ureteric bud
- glomerulus and tubules til the distal convoluted
tubules are from mesonephric duct
- urethal meatus from ectoderm
Q) Type I found in skin, fascia, tenson and dentin
Type II found in cartilage
Type III (reticulin) found in skin, blood vessels,
alongside of type I, embryonic dermis,
uterus,fetus, heart and granulation tissue,
Type IV found in Basement laminae of epithelial
and endothelial cells
Type V: cell surfaces hair and placenta
1 in bone dermis tendone
2 cartilage
3 fetal / cardiac / scar/ synovium
4 basement membrane

Type 1 collagen makes 50% of total body protein


4
Q)

Q) -dome of bladder from yolk sac


-trigone from mesonephric duct
-bladder from urogenital sinus

-ureter until collecting duct are from ureteric bud


- glomerulus and tubules til the distal convoluted
tubules are from mesonephric duct
- urethal meatus from ectoderm
Q) Vit A: functions for eye and lungs .lack of it
cause: night blindness
Vit D: functions for bone and teeth formation lack
of it cause: rickets
Vit C:functions for gums ,wound and also
increase immune system lack of it cause:
inflammation of the tongue and lateral margins
of the tongue and gums that become swollen and
red+scurvy
VitE: functions for sex glands lack of it causes.
Hemolysis and sterility
Vit K: functions for blood clotting lack of it cause
hameorraghe
Vit B1: functions: nervous system.lack : beri beri
Vit B2: functions: eye,skin and blood.lack: slow
growth and sore of eyes
Vit B4.functions.gums tongue .lack of it .bleeding
Q) Which stage of the cell cycle is a mitosis
proceeds?
Answer :G1
In which stage cell prepare to enter into mitosis?
Answer :G2
DNA damage checked in ?

Answer :G1
DNA & histone synthesis & centriole replication?
Answer : S phase
Cell become quiescent in ?
Answer :G0
Chromosomal study done in?
Answer : metaphase
Q) Following are correctly matched except A)ACTH 29 A A..ans ,39
B)prolactin 199 A A
C)growth hormone 191 AA
D)leptin 167 AA
E)none above
Q)

For HRT

For HRT -15 30 /100,000


Q)

Q) Wrt oestrogen production during pregnancy


A- Placental oestrogen production is independent of fetal
adrenal activity
B- The placenta converts progesterone to oestradiol in early
pregnancy
C- The placenta converts pregnenolone to oestradiol
D- The placenta converts fetal DHEAS to oestradiol . ans
E- The addition of a sulphate group to steroid hormones
enhances their biological activity
OESTROGEN SECRETION
Under fetal control and is a fundamental fetomaternal signalling mechanism
The placenta lacks 17-alpha hydroxylase and 1720 desmolase activity and cannot convert C21

products (progesterone / pregnenolone) to C19


products (androstendione / DHEA)
In early pregnancy, the placenta utilises maternal
androgens for oestrogen production
By 20 weeks gestation, the majority of placental
oestrogen production is from fetal DHEASulphate.
The fetus rapidly sulphates steroids, preventing
biological activity.
The placenta has an active sulphatase to remove
sulphate groups
Fetal DHEA-S is converted to oestradiol and
oestrone. The placenta cannot produce oestriol
from DHEA-S
Fetal DHEA-S is hydroxylated by the fetal liver to
16-alpha-hydroxy-DHEA-S. This is then utilised by
the placenta for OESTRIOL production. Oestriol is
first detectable at 9 weeks gestation when fetal
adrenal gland secretion of precursor begins. In
the absence of a normal fetal adrenal gland,
maternal oestrogen concentrations are very low
(for instance, anencephaly) OESTRIOL is the main
placental oestrogen. Oestradiol and oestrone are
derived equally from maternal and fetal
precursors
Maternal oestraDIOL levels are higher than in the
fetus
Fetal oesTRIOL levels are higher than maternal
levels
Placental aromatisation of maternal androgens is

so effective that the fetus is protected from


masculinisation. Extremely high androgen levels
or non-aromatisable analogues are required for
fetal effects
Q) A lady was subjected for a colposcopy in view
of her abnormal pap smear findings. The
procedure is considered incomplete if which area
is not visualised?
a. endocervix
b. ectocervix
c. transitional zone
d. transformation zone ..ans
e. posterior fornix
Q) International Ovarian Tumour Analysis (IOTA) has come
up with a guideline to distinguish benign from malignant
tumour via ultrasound. The following are the features of
benign tumour on ultrasound except : a. unilocular cyst
b. presence of soild component < 7mm
c. presence of acoustic shadowing
d. no blood flow
e. presence of ascites..ans

Q) Pap smear is a good screening tool for cervical CA. The


following are the qualities it has, to be a good screening tool,
except : a. simple,safe and acceptable
b. high sensitivity, high specificity with good predictive value
c. reproducible
d. ill defined cut off levels ans
e. cost effective

Q) Maternal serum HCG levels at the time of the first missed


period
A 10 IU/L
B 100 IU/L .ans
C 500 IU/L
D 1000 IU/L
E 2000 IU/L

Q)

Ans D
Q) 35 year old women attends clinic following laparotomy
and unilateral oophorectomy. The histology shows mucin
vacuoles. What type of tumour would this be consistent with?
Serous
Mucinousans
Endometrial
Transitional
Cear cell

Q)55-y M is admitted to ED with 'tearing' chest pain radiating


through to his back.Examn reveals a P 96 / min regular, BP
130/85 & O2 sats - 97% on room air. A CxR - mediastinal

widening.A CT shows dissection of ascending aorta. What is


most suitable initial Mx?A. IV sodium nitroprusside
B. Oral verapamil
C. Observe only
D. IV labetalol ..ans
E. Surgical repair
Aortic dissection
type A - ascending aorta - control BP(IV labetalol)
+ surgery
type B - descending aorta - control BP(IV
labetalol)
Q)Meckel's diverticulum is an adult remnant of
vitelline duct. Which of foll embryonic structures
are joined by the vitelline duct? A. Foregut and
yolk sac
B. Midgut and yolk sac.ans
C. Hindgut and yolk sac
D. Foregut and allantois
E. Midgut and
allantois

Q)Tumor Necrosis Factor is produced by


a.T-lymphocytesans
b.B lymphocytes?

Q)

Ans d-Excessive fluid intake is obvious, it's not the


interpretation. Sensory function can't be interpreted from
bladder diary. For cystitis you need a urine culture. It's not

urge incontinence as she has this only once. It's overactive


bladder
Q)What % age of patients with breast CA have
hypercalcaemia - <1%
2-3%
5%
20% ans
>50%

Q)55 y presents to clinic due to vulval itch and


discolouration. examination reveals pale white discoloured
areas to the vulva. A biopsy conforms Lichen Sclerosus (LS).
What is risk of developing SCC compared to patients with
normal vulval Bx ? 0%
<5%..........ans
5-15%
25%
100% - histological features confirm SCC
Q) Inner most layer of testis - Tunica albugenia
Tunica vaginalis
Tunica vasculosa.ans

Q) which hormones shows diurnal variation; (t/f) - a.estrogen


b.progesterone
c.fsh
d.melatonin
e.cortisol..ans
Q) The following serum markers inc in preg except.- T3
T4ans
Protein
Alkaline phosphatase.
TBG
Q) Which ascending artery can be damaged
during open appendicectomy? - a Iliolumbar
artery -

b Superficial circumflex artery


c Deep.circumflex artery. -ans
d Sup gluteal artery
Artery damaged deep circumflex iliac artery
Nerve damaged iliohypogastric nerve
Q) Which test is most appropriate in diagnosis
the thyroid dysfunction in pregnancy ? TSH
Thyroid binding globulin
Serum iodine levels
Serum tri iodithyronine (T3)
Free thyroxine (T4) .ans
Q) Robertsonian Translocation

Q) reciprocal Translocation

Q) The zygot contain how many ceels when enter


the uterus? A. 8.ans
B. 16
C. 30
D. 150 cell

Q) Brain liquifective...
Kidney n heart coagulative...
Limb n gut gangrene...
Fat necrosis breast n pancreas

Q) Mc cause of breast lump in reproductive age is


A)fibro adenoma
B)proliferative breast diseaseans
C)ductal ca
D)lobular ca
E)ductal papilloma
Proliferative breast disease also known as
fibrocystic breast disease most common cause of
breast cyst in reproductive age in 50 %
Q) While performing laparoscopy the surgeon
identified the medial umbilical folds on the deep
surface of the anterior abdominal wall. The 2
medial umbilical folds represent remnants of
which of foll structures? A. Urachus
B. Inferior epigastric vessels
C. Obliterated umbilical veins
D. Obliterated umbilical arteries..ans
E. Round ligaments of the uterus
Q) Following are the features of benign tumour
except :
a. slow growth
b. presence of limiting capsule
c. aberrant mitotic formsans
d. able to function as original tissue
e. Inability to spread beyond tissue of origin
Q) How many telomeres are in the cell in
metaphase of mitosis? A. 1

B. 2
C. 4ans
D. 6
E. 8
Q) Which artery supplies distal portion of round
ligament of uterus? 1 . ovarian artery
2.ut artery
3.inf epigastric artery..ans
Q) WHICH VESSELS CAN BE POSSIBLY INJURED IN
THE SUBCUTANEOUS TISSUE WHEN A
TRANSVERSE SUPRAPUBIC SKIN INCISION IS
MADE? A. SUPERFICIAL EPIGASTRICans
B. SUPERFICIAL CIRCUMFLEX ILLIAC
C. DEEP CIRCUMFLEX ILLIAC
D. SUPERIOR EPIGASTRIC
E. INFERIOR EPIGASTRIC
Q) At what level of HBA1 C the diabetic lady Should not
conceive - 1_6%
2_7%
3_8%
4_10% . ans
5_11%
Q) Which of foll ligaments allows us to stand
upright with a minimum of muscular support?A.
Sacrospinous ligament
B. Sacrotuberous ligament
C. Iliolumbar ligament

D. Iliofemoral ligamentans
E. Pubofemoral ligament

Q)G2 phase presceed Mitosis stage (and cyto


kinesis)

Q)

Q) How much percentage of the cardiac output


passes through the kidneys at
term? A. 5%
B. 10%
C. 25%........ans
D. 50%
E. 75%

Q) A 7 year old girl is brought to see you by mother because


girl has developed breast and few pubic hair .which of the
following is best treatment for her - A)reassurance
B)ethinyl estradiol
C)gnrh analogans
D)HMG
E)exogenous gonadotropin
Q) The proportion of miscarriages that occur in first trimester
A. 5-10%
B. 15-20%
C. 25-30%
D. 45-50%
E. 75-80%.......ans
Q)

Q) in an average pregnancy, how much does cardiac output


increase during the 1st trimester? 0.5lit/min
1 lit/min
1.5lit/min .ans
2 lit/min
doesn't change
Q)

Q)

Q)

Q)Classical bld test results in Addisons disease?


a. Hyperkalaemia, hyponatraemia, uraemia, hypercalcaemia
and basophila
b. Hyperkalaemia, hyponatraemia, uraemia, hypercalcaemia
and eosinophila.ans
c. Hyperkalaemia, hyponatraemia, uraemia, hypocalcaemia
and basophila
d. Hyperkalaemia, hyponatraemia, uraemia, hypocalcaemia
and eosinophila
e. Hypokalaemia, hypernatraemia, uraemia, hypercalcaemia
and eosinophila

The answer is Hyperkalaemia, hyponatraemia,


uraemia, hypercalcaemia and eosinophila. Lack
of mineralocorticoids results in reduction of
sodium reabsorption and potassium excretion.
Due to the loss of sodium, there is associated
water loss and fluid depletion, thus causing
uraemia and hypercalcaemia.
Q)Extremely anxious parturient requests labor
epidural analgesia.During epidural placement,
cerebrospinal fluid obtained in the needle
catheter.Which along spinal cord dose the needle
traverse obtaining CSF leakage from the needle
catheter? A. Vertebral canal
B. Space between wall of vertebral canal and
dura mater
C. Space between arachnoid and dura mater
D. Space between arachnoid and pia mater
E. Spinal canal
ANSWER : Space between arachnoid and pia
mater - CSF leakage
Option C is normal site for EA
Q) The blood test of a 36-year-old fertility patient shows
elevated luteinising hormone (LH) levels and high estradiol
levels. What is the most likely cause of this?
a. Hypogonadotrophic hypogonadism
b. Midcycle LH surge..ans

c. Ovarian failure
d. Polycystic ovary syndrome
e. Weight-related amenorrhoea
The answer is Midcycle LH surge. Polycystic ovary syndrome
(PCOS) is associated with high LH levels and usually normal
estradiol levels. Ovarian failure will result in high LH levels
but low estradiol levels. Hypogonadotrophic hypogonadism
and weight-related amenorrhoea are associated with low LH
levels and low estradiol levels. PCOS results in a ratio of
LH/Follicle-stimulating hormone (FSH) > 1. The blood test
may have been taken midcycle. In this case we are not able to
diagnose PCOS because we do not have a value for FSH.
Generally, women with PCOS have normal estradiol levels.
Q) Monopolar diathermy cause coagulation effect
with sparkling to stop the bleeding vessels by
Which of the following procedures? A.
Electrosurgical cutting
B. Electrosurgical desiccation
C. Electrosection
D. Fulguration
E. Vaporization
ANSWER : Fulguration
Q)Which hypothalamic hormone stimulates release of Prl ?
a. Dopamine
b. Growth hormone releasing hormone
c. Gonadotrophin-releasing hormone
d. Somatostatin
e. Thyrotrophin-releasing hormone . ans

The answer is Thyrotropin-releasing hormone.


Thyrotrophin-releasing hormone stimulates the
release of TSH and prolactin from the anterior
pituitary. Dopamine also released from the
hypothalamus inhibits prolactin release
Q) When does the hypothamo-pituitary axis
establish in a growing fetus?a. 12 weeks
b. 16 weeks
c. 20 weeks..ans
d. 24 weeks
e. 28 weeks
Q)Nv supply to perineal skin Inf rectal nerve

Q)

Q) The physiological changes in preg are assoc with a change


in serum concn of various hormones. Which test is most
appropriate in diagnosing the thyroid dysfunction in
pregnancy? A. Free thyroxine (T4) levels .ans
B. Serum iodine levels
C. Serum triiodothyronine levels
D. TBG levels
E. Thyroid-stimulating hormone
:
The answer is free thyroxine (T4) levels.
During pregnancy hepatic synthesis of TBG is increased.
Total T4 and T3 levels are raised to compensate for this rise.
TSH levels fall in 1st trim as concn of HCG rise, however they
may occasionally rise. The levels of fT4 are altered less in
pregnancy.
The normal pregnancy range for each trimester should be used
in diagnosing or monitoring Rx in cases of thyroid disorders.
Pregnancy is assoc with a relative iodine deficiency, which
has 2 mj causes. Maternal iodine requirement increases
because of active transport to fetoplacental unit. Also there is
inc iodine excretion due to inc glomerular filtration and dec
renal tubular excretion.
Q) All of foll are true about NTD, except?
a) Decrease in AFP..ans
b) Polyhydramnios
c) FA prophylaxis can decrease the risk of NTDs
d) There is a 4% chance of NTDs in 2nd preg
e) Can be detected by amnio centesis

Q) Which one of foll is readily transferred across


plac ?
A. Testosterone
B. PTH
C. Vit Dans
D. IgM
E. Calcitonin
Q) Which of the following pairing is not true in a
male?
a. zona glomerulosa - aldosterone
b. zona fasciculata - cortisol
c. zona reticularis - progesterone
d. zona reticularis - DHEA
e. adrenal medulla estrogen..ans
Q) 13 wks pregnant 35 year old
- bhg decrease
- estriol decrease
- AFP decrease : what u suggest the cause:
A. Down syndrome
B. Edward syndrom
C.missed miscarriage ans
D. Normal pregnancy
Q) . Below are the main actions of ACTH except :
a. stimulate the synthesis and release of
glucocorticoids
b. stimulate the synthesis and release of
androgens

c. stimulate the synthesis and release of


mineralocorticoids..ans,
mineralocorticoids are regulated by reninangiotensin system
d. increase the activity of cholesterol esterase
e. facilitate the transport of cholesterol into
mitochondria
Q) The RCOG define septic shock as the
persistence of hypoperfusion despite adequate
fluid replacement therapy. What is the mortality
rate in patients with septic shock?
5%
10%
20%
35%
60%...........ans
Q) what %age of cardiac output goes to kidneys at term?
A.25%
B. 50%.........ans,300ml/min
C. 75%
Q) followng r endodermal in origin? a.heart
b.spleen
c.adrenal cortex
d.dermis
e.germ cells ans

Q) A 16 year old girl is under your follow up in your growth


and development clinic. You examined her breast and noted
areolar enlargement with breast bud. What is her Tanner
staging? a. 1
b. 2..ans
c. 3
d. 4
e. 5

Q)

Q) A 36 years old patient presented to you in your clinic


complaining of minimal amount of menses. She gets her
menses monthly for 2 days and only uses 1 pad per day. UPT
was negative. On examination, you noted swelling over her
neck. You subjected her to a radioisotope study and below is
the findings. What is the most probable diagnosis?
a. Multinodular goiter
b. Hashimoto tyroditis
c. thyroid follicular tumor
d. Graves disease..ans
e. Toxic adenoma

Hyperthyroidism is associated with


oligomennorhea. Radio isotope finding shows
diffuse uptake of isotope which is the findings in
Graves disease

Q) What kind of cells produces parathyroid


hormone (PTH)?A. Chief cells .ans
B. Parathynotic cells
C. Oxyphilic cells
D. C cells
E. Follicular cells
Q) With regards to the development of the
alimentary system, which of the following is true?
a. embryonic foregut is supplied by the coeliac
artery
b. midgut includes the second part of duodenum
c. hindgut incorporates the appendix
d. Meckel's diverticulum is a remnant of the
vitelline duct..ans
e. gut is derived from mesoderm

Q)GnRH decapeptide
Q) GnRH is :
A. Produced by anterior pituitary gland
B. Produced by the posterior pituitary gland
C. Produced by placenta..ans

D. Plays important role in supporting corpus


luteum
E. Glycoprotein
Q)

Ans C
Q)The release of CRH,ACTH & cortisol follows a circardian
rhythm. When is highest peak of those hormones? a. at night
b. early morning..ans
c. afternoon
d. mid afternoon
e. midnight
Q) The axial filament of the sperm tail has what
type of arrangement of filaments?1+5
1+7
3+5
7+5
9+2 . ans

Q) growth n maturation of d breast at puberty is


promoted by:a.estrogen.ans
b.progesterone
c .insulin
d.cortisol
e.pth
Q) Mast cells A. Produce mast cell growth factor
B. Increase in blood in allergies
C. Mature in bone marrow
D. Secrete histamine and heparinans
E. All of the above
Q) In thalassemia mother is at Inc risk of all of
the following except - A))diabetes
B)cardiomyo pathy
C)hypothyroidism
D)DVT
E)hyperparathyroidism . ans

Q)

Ans C,C
Q) A patient was seen in your clinic and was diagnosed of
having pituitary adenoma. She was referred to an
ophthalmologist because she complained of blurring of vision
over the past 3 months. Which would be the finding of the
ophthalmologist? ( blind spot is shaded pink)
a. 1st image
b. 2nd image
c. 3rd image.ans,bitemp hemianopsia

d. 4th image
e. normal findings

Q) The HPV vaccine Gardasil is what type of vaccine?


Live attenuated viruses
Inactivated viruses
Toxoid based vaccine
Polysaccaride based vaccine
Recombinant vaccine of virus-like particles (VLPs).ans
Q) In regards to an endocrine disorder, it is said to be a:
a. primary disorder when the end organ is dysfunctional
b. secondary disorder when function is affected by defective
pituitary gland
c. both of the above
d. tertiary disorder when the hypothalamus is dysfunctional
e. all of the above..ans
Q) What's the most important buffer in interstitial space?
HCO3-

Q) In a normal pregnancy, following are true,


EXCEPT:
1. Glucosuria present
2. Plasma Aldosterone concentration
falls.ans
3. Creatinine clearance is increased
4. Folate excretion is increased
5. The presence of less than 300mg of protein in
24 hours urine collection is considered normal

Q)

Q)

Ans 1 D,2 B,3 E

Q)HPV types assoc with ^ r/o cx CA16,18,31,33


Q) Postoperative patient with history of DM and
asthma. develops some feeling of unwellness.
Sodium 112 mEq/L and other parameters were in
normal range.What is the diagnosis? A. Addison
diseaseB. Cushing syndromeC. Conn's
syndromeD. SIADHE. Dehydration
Q) What percentage of HPV infections will be cleared by the
host within 1 year?
5%
20%
50%
70%..............ans
90%
Q) The karyotype of a patient with Androgen
Insensitivity syndrome is - . 46XX

.47XXY
. 45XO
. 46 XY.ans
.45XY

Q) Concentration of this hormone peak in the first


trimester and again rise at term A. Inhibin
Aans
B. Action
C. HCG
D. HPL
E. Progesterone
Q) Which one of the following is not readily
transferred across the placenta A. Aminoacids
B. Glucose
C. Thyroxine.ans
D. IgG
E. CA+2

Q)

Q) Precocious puberty is said to occur in a female


if pubertal changes occur before the age of : a. 7
b. 8ans ,in males before 9
c. 9

d. 10
e. 11
Q) A girl is said to have delayed puberty when :
a. no pubic hair at 16 years old
b. no menses at 16 years old
c. no breast development at 13.5 years old .. ans
d. no pubic hair at 13.5 years old
e. no breast development at 16 years old
delayed puberty is when testicular volume is less than 4 mls at
14 years old
Q) Which lung volume is decreased in pregnancy?
A. Peak flow rate
B. Tidal volume
C. Vital capacity
D. Functional residual capacity
E. Forced expiratory volume
:
The answer is Functional residual capacity. The pushing up of
the diaphragm reduces the functional residual volume.

Q) trigone of bladder develops from : A.


mesonephric duct..ans
B. urogenital sinus

Q) Enlarged hands and feet, jaw protusion, dental


malocclusion, galactorrhea
Listed above are clinical feature of an endocrine disorder.
How would you confirm the diagnosis?
a. dexamethasone supression test
b. corticotrophin test
c. both of the above
d. visual field test
e. oral glucose tolerance test.ans,acromegaly
Q) What is the fetal case mortality rate
associated with listeria infection during
pregnancy?1%

10%
25%...........ans
50%
95%
Q) Placental hormone which is produced by the
syncytiotrophoblast but not by the
cytotrophoblast
A. Human chorionic gonadotropin.ans
B. Human placental lactogen
C. Inhibin
D. Human chorionic thyrotropin
E. Estriol
Q) Terbutaline has a preference for stimulation of
which of the following receptors? A. Alpha
B. Gamma
C. Beta 1
D. Beta 2 ans
E. Dopaminergic

Q)

Q) Which of the following cell types act as


professional antigen presenter cells (APCs) - A.
Neutrophils
B. Interferon
C. Basophils

D. Macrophagesans
E. Mast cells
Q) 19 years old female presented with
ammenorhea for 3 months. She was previously
having normal menses. She also complaints of
headache associated with certain degree of
vision loss. During the 2 hours consultation, she
excused herself to the washroom for 4 times.
What is the most probable diagnosis : a.
prolactinoma
b. craniopharyngiomaans
c. acromegaly
d. Addison's disease
e. Cushing's syndrome
menstrual disturbance, headache with vision loss
and diabetes insipidus are features of
craniopharyngioma. It is a tumour arising from
rathke cleft (embryogenic origin of pituitary
gland) and most of the time benign.
Q) vomiting:- meta alkalosis
diarrhoea:- meta acidosis
Hypokelemia.. Hypochloremia met alkalosis(due
to hypokal) in vomitings
Q) Which of the following was a disadvantage of
depoprovra
A)Inc risk of hepatic ca
B)impairment of lactation

C)prolonged anovulation..ans
D)irreversible bone loss
E)iron Def anemia
Q)

Ans A
Q) Which of the following is true regarding POP's
A)more effective than inj contraceptive
B)may worsen acneans

C)Dec risk of ovarian cysts


D)contraindication in migraine
E)mainly inhibit ovulation
Q) Hyperplasia and hypertrophy of alveolar cell of breast
during pregnancy are stimulated by which hormone :
A estrone and hcg
B human placental lactogen and estradiol
C human placental lactogen and progesterone
D prolactin and hplans
E prolactin and progesterone
Q) Normal ratio of LH:FSH? 1:1
Q) Which of these chromosomes have
centromere located at one side? A. Telocentric
chromosomeans
B. Metacentric chromosome
C. Dicenteric chromosome
D. Submetacentric chromosome
E. Acrocentric chromosome

Q) Concentrations of this hormone peak in 1st


trimester and rise again at term? A Inhibin
A.ans
B Activin
C Human chorionic gonadotrophin
D Human placental lactogen
E Progesterone
Q) At which stage of preg are women at greatest risk of severe
disease foll H1N1 influenza infection? A First trimester
B Second trimester
C Third trimester .. ans
D 24-48h post-partum
E 4-6 weeks post-partum

Q)Which hormone has a prim site of action on


granulosa cells? A. Gonadotropin releasing
hormone (GnRH)
B.FSH) .ans
C. Inhibin
D. Luteinising hormone (LH)
E. Relaxin
Q) A 25-year-old diabetic woman has been
morbidly obese for the past five years.In this
patient, which one of the following hormones
would decrease the appetite
as levels increase? A. Leptin.ans
B. Thyroxine
C. Ghrelin
D. Adiponectin
E. Insulin
Q) Ischio Rectal fossa Medialy external anal
sphincter and facia of levator anni
Laterally obturator facia
Anertiorly urogenital diaphragm
Posteriorly sacrotuberous ligament and gluteus
maximus

Q) 7. Following are the sequence of puberty in a female:


I : pubarche
II : thelache
III : menarche
IV : axillary hair
V : growth spurt
a. III - II - ! - V -IV
b. II - I - V - III IVans
c. II - III - I -IV - V
d. I - II - III - V - IV
e. I - II - III - IV - V
remember up(breast) - down(pubic hair)up(gowth spurt) up (axillary hair ) down
(menses)

Q) Blood supply of the cx? A. Ut artery


ans
B. Vaginal a.
C. Internal pudendal a

Q) The standard chest X-ray is equivalent to what


duration of natural background radiation - A. 3
days.ans
B. 3 weeks
C. 3 months
D. 18 months
E. 5 years
Q)

Q) Trilaminar disc forms in 3rd week


Anterior neurop closes at day 25
Upper limb buds are formed at day22

Q) Lymph node drain of the upper anal canal :


A. Int iliac ans

B. Inf mesentric
C. Sup inguinal

Q) From what does the blastocyst hatch? Corona radiata


Cumulus oophorus
Perivitelline space
Trophectoderm
Zona Pellucidaans
Q) The foll are all true with regarding to labour EXCEPT:
a) Labour typically occurs between the 37th and 42nd week
b) The 2nd stage of labour is assoc with dilatation of Cx..ans
c) 3rd stge of labour is interval betw del of fetus to del of plac
d) The Cx is drawn up into the LUS
e) It is associated with the passage of a mucus plug
Q) A woman is challenged with COCP, but does not get
withdrawal bleeding. Which organ is most likely to be

affected? a) Overies
b) Uterusans
c) Pituitary
d) Hypothalamus
e) Fallopian tubes
Q) Incidence of pheochromocytoma in pregnanc?
1/1000
1/10,000
1/50,000..ans
1/100
<1/1,000
Q) What is % age change in ventilation during 1st
trim ?
20% decrease
40% decrease
20% increase
40% increase ..ans
50% increase
Q) What is overall change in airway resistance in
preg compared to a non- pregnant woman?
Decreases by 10%
Decreases by 50%
Increases by 10%
Increases by 50%
None of the above .ans

Q) In regards to fetal circulation, which of foll has


the highest oxygen saturation? a. ductus venosus
b. umbilical vein.ans
c. ascending aorta
d. umbilical artery
e. pulmonary vein

Q)The foll developed from urogenital sinus except


:
a. urinary bladder
b. prostatic urethra in male
c. genital organs
d. whole urethra in female
e. ureter .. ans
Q) A woman comes to you 2 years after her menopause
asking for something to relieve her PM syndromes. You
decide to give her combined HRT. You explain to
her that all of foll are effects of progesterone, except?
a) She will have pre-menstrual like symptoms
b) She will have withdrawal bleeding every month

c) Progesterone will oppose the action of oestrogen on the


endometrium
d) Progesterone will potentiate the action of oestrogen in
increasing HDLans
e) Progesterone will augment the action of oestrogen for
prevention of osteoporosis
Q) The midgut goes through rotation during its
development. Which structure provides the axis
for its rotation?
a.
abdominal aorta
b. coeliac artery
c. superior mesenteric artery ans
d. inferior mesenteric artery
e. pudendal artery

Q) A patient was seen in the FMU for a suspected


congenital malformation. You noticed that the
AFP is raised. On physical appearance, she looks
like she is in her late teens, multicolored hair dye
with multiple ear, nose and tongue piercing. You
overheard that she is a cocaine abuser. What is
most probable diagnosis of her fetus? a.
anancephaly
b. gastrochisisans
c. spida bifida
d. down syndrome
e. edward syndrome
gastrochisis and omphalocele are strongly
associated with cocaine and heroine abuse.

Q) A 52-y with Cushings syndrome is referred to


PAC prior to a VH.Which of foll is a feature of
Cushings syndrome?
A Decreased plasma lactate dehydrogenase
B Hypoglycaemia
C Hypokalaemia ..ans
D Hyponatraemia
E Metabolic alkalo
Q) Which of foll drug combo should be avoided ?
A aspirin and clopidogrel
B ethanol and co-amoxiclav
C metformin and chlorpromamide
D sildenafil and isosorbide mononitrate
.ans
E simvastatin and fenofibrate
Q) What is the finite life span of the corpus
luteum? 8 days
10 days
12 days
14 days .ans
16 days
Q) In regards to the development of heart, the
anterior displacement of the conotruncal septum
results in the occurrence of a condition known as
tetralogy of fallot. Which of the following is the
feature of it? a. ASD
b. mitral stenosis

c. pulmonary hypertension
d. pulmonary stenosis ans
e. left ventricular hypertrophy

Q) Which of following is true of side effects of


vancomycin ?
A liable to occur with chronic liver disease
B effects not seen when given orally
C unusual in elderly
D includes irreversible vestibular damage
ans
E includes liver failure
Q) An infant is born at term by NVD. When the
baby is 18 days old, he is brought to ED by his
parents. He is V, severely dehydrated and
appears to be underwt. The paediatricians
diagnose a salt-wasting crisis and are concerned

that he has a form of CAH.What hormone


deficiency is characteristic of this disorder? A
Cholesterol
B Cortisol..ans
C Dihydrotestosterone
D Oestradiol
E Testosterone
Medical disorders in pregnancy250 Congenital
adrenal hyperplasia This is an AR disorder
affecting the synthesis of glu-cocorticoids and
mineralocorticoids. In response to low levels of
these hormones, the pituitary gland produces
large amounts of ACTH and this results in
excessive production of sex steroids. A number of
enzyme defi ciencies can lead to this condition:
the commonest is 21-hydroxylase defi ciency.
Many different gene mutations exist, which result
in vari-able clinical presentations.Rx is
replacement with corticosteroid +/
fludrocortisone. Affected individuals present in
several ways: Salt-losing crisis in neonate.
Masculinization of female fetus (ambiguous
genitalia at birth). Precocious puberty in boy.
2 If a couple has an affected child, risk in
subsequent pregnancies i
Q) The foll are all true with regards to PTL
EXCEPT:
a) The fetus is typically delivered between 24 and

37 weeks
b) Salbutamol promotes FLM.ans
c) It is often the result of pre-eclampsia
d) It is often the result of polyhydramnios
e) It is often associated with trichomonas
infection
Q)

Ans E
Q)What percentage of women with a diagnosis of Gonorrhoea
will develop PID? 2%
5%
15%
25% ans
50%

Q) Total pregnancy loss associated with CVS 1-2 %


Q) Which of foll is true of common bile duct? (Please select 1
option)
A)Lies posterior to the portal vein
B)Lies posterior to the second part of the duodenum
C)Lies to the left of the gastroduodenal artery
D)Lies to the right of the hepatic artery..ans
E)Opens into the third part of the duodenum

Q) The following are all risk factors for shoulder dystocia


EXCEPT: a) Slow progress in the first stage
b) Slow progress in the second stage
c) Macrosomia
d) Postmaturity
e) An underweight motherans
Table 1. Factors associated with shoulder dystocia
Pre-labourPrevious shoulder dystocia Macrosomia
>4.5kgDiabetes mellitusMaternal body mass index >30kg/m2
Induction of labour
Intrapartum- Prolonged first stage of labour Secondary
arrestProlonged second stage of labour Oxytocin
augmentationAssisted vaginal delivery

Q) Regarding soulder dystocia, which is correct?


A. It is not assoc. with maternal DM
B. Arrest occurs at pelvic inlet
C. Increased possibility if epidual anaesthesia given
D. Most cases can be resolved by hyperflexing the fetal thighs
onto the abdomen.
E. Erb's palsy (C8-T1) is a complication.ans
Q) Transpyloric plane passes through which vertebrae : T11
T12
L1 ..ans
L2
L3

Q) During oogenesis which cell type has the greatest number


of chromatids?
Oogonium
Primary Oocyteans
Secondary Oocyte
Ootid
Ovum
Q) Normal semen analysis (WHO criteria 2009) Volume
>1.5mL. Concentration >15 106/mL. Progressive motility
>32%. Total motility >40%.

Q)

Q) Where does the primodial germ cell arise from ? a. alantois


b. yolk sac . ans
c. syncytotrophoblast
d. gonadal ridge
e. paramesonephric duct
primordial germ cells arise from the cell wall of yolk sac
during second week of development. in the 6th week, it
migrates to occupy the gonadal ridge. Failure to migrates
causes gonadal agenesis.
Q) In regards to the following, all are true except :
a. epiblast forms the ectoderm
b. hypoblast forms the endoderm
c. amniotic cavity develops within hypoblastans,epiblast
d. the primary yolk sac derived from hypoblast
e..the primary yolk sac is the main source of nutrition before
the placenta takes over

Q) The following are true regarding early embryology


except :
a. Bilamilar embryonic disc is formed during second week
b. trilaminar disc is formed during third week
c. bilaminar disc consist of hypoblast and epiblast layer
d. the hypoblast gives rise to the third layer ans, the
third layer arises from the epiblast
e. formation of three germ layer is known as gastrulation
Q)Which Ar exits the pelvis via greater sciatic foramen and
re-enters via lesser sciatic foramen? Superior Gluteal
Inferior Gluteal
Internal Iliac
Internal Pudendal . ans
Superior Gluteal

Q)Which of foll derives from ectoderm:


a. adrenal medulla..ans
b. liver
c. thyroid gland
d. pancreas
e. parathyroid gland

Q) By which day does a fully formed blastocyst reaches the


uterine lumen in preparation for implantation? a. day 2
b. day 5..ans
c. day 7

d. day 10
e. day 12
Q)

Q) which of foll represents the correct sequence which occurs


during fertilization? I - formation of zygote
II - fusion of oocyte and sperm cell membranes
III - completion of meiosis in the oocyte
IV - sperm activation and penetration of corona radiata
V - attachment to zona and penetration of zona pelucida
a. IV - V- II - III I.ans
b. V - IV - II - III - I
c. IV - III - V - II - I
d. V - IV - II - III - I
e. III - IV - V - II - I

Q) The aldosterone works in: A. PCT


B. DCT .. ans
C. Connecting duct

Q)The 3rd pharyngeal arch gives rise to which of foll


structures? Thrigeminal Nerve
Glossopharyngeal nerve..ans
Facial nerve

Vagus nerve
Muscles of mastication

Q)

Q) List of notifiable diseases Diseases notifiable


to local authority proper officers under the Health
Protection (Notification) Regulations 2010: Acute

encephalitis Acute infectious hepatitis Acute


meningitis Acute poliomyelitis Anthrax Botulism
Brucellosis Cholera Diphtheria Enteric fever
(typhoid or paratyphoid fever) Food poisoning
Haemolytic uraemic syndrome (HUS) Infectious
bloody diarrhoea Invasive group A streptococcal
disease Legionnaires disease Leprosy Malaria
Measles Meningococcal septicaemia Mumps
Plague Rabies Rubella Severe Acute Respiratory
Syndrome (SARS) Scarlet fever Smallpox Tetanus
Tuberculosis Typhus Viral haemorrhagic fever
(VHF) Whooping cough Yellow fever
Q) The inferior one-third of the anal canal
originates from which structure?Cloacal
membrane
Ectoderm of the proctodeum.ans
Endoderm of the hind gut
Endoderm of the mid gut
Urorectal septum
The junction between the superior and inferior part is
delineated by a pectinate line, just below the anal columns.
The endoderm of the hind gut develops into the superior part
of the anal canal.
Q)ATP produced in Ribosomes
Q) The urorectal septum is responsible in partitioning the
cloaca. What does the tip of the urorectal septum forms?
a. lower 1/3 of vagina

b. lower 2/3 of vagina


c. anal canal
d. perineal bodyans
e. external urethral meatus
Q)PKU in Uk 1 in 10,000 births. inborn error of protein
metabolism in the UK.
Q) The following tissue are paired with the appropriate
primary germ cell layer except :
a. mammary duct epithelium : ectoderm
b. epithelium of the tongue : mesoderm.ans
c. pineal gland : ectoderm
d. dermis of the skin : mesoderm
e. endometrium : mesoderm
Q) The axial filament of the sperm tail has what
type of arrangement of filaments? 1+5
1+7
3+5
7+5
9+2 ans
Q) The intraembryonic coelom forms within
which structure?Endoderm
Intermediate mesoderm
Lateral plate mesoderm..ans
Neural tube
Somites

In the second week of development the lateral plate mesoderm


splits to create this cavity. It will form the space for the
thoracic and abdominal cavity.
Q) A 24 years old girl who is just recently married and not
emotionally prepared to conceive a child. In order to prevent
pregnancy, she is taking mercilon as a contraceptive method.
She came to you today because she had missed a pill for 24
hours. What would your advice be?
a. discard this pack and start a new pack
b. take the missed pill immediately and continue her current
pack with one week pill free interval..ans
c.take the missed pill immediately and continue her current
pack with no pill free interval
d. take the missed pill immediately and continue her current
pack with 7 days additional contraception
e. continue this pack with no pill free interval
Q) What is the progestogen component in an
implant?
a. desogestrel
b. levenogestrel
c. etonogestrel..ans
d. drosperinone
e. norethisterone
Q) What is the innervation to the sigmoid colon? T5-T9
T10/11
T12/L1.ans
L1/L2
L2/L3

Q) How many oocytes are available at the time of


puberty? 125 000
250 000ans,250,0000 to 500,000
500 000
1 million
2 million
Q) What happens in PE? Metabolic acidosis
Metabolic alkalosis
Respiratory acidosis
Respiratory alkalosis..ans
Q) There are a few processes or reactions that are important to
achieve fertilization. Which of the following is/are true :
a. capacitation : makes the sperm capable of penetrating the
ovum
b. acrosomal reaction : release of enzyme to facilitate ovum
penetration
c. zona reaction : prevent attachment of any more sperm
d. cortical reaction : aids the zona reaction
e. all of the above.ans
Q) Mc organism for surgical site infection?
A. Staph aureus .ans
B. Strepto pyogenes
A is 1/3 but B is 2/3
Q) Where in body is calcidiol produced? Kidneys
Liver ans

Parathyroids
Skin
Spleen

Q) With regard to NMJ, what causes a reduction in release of


Ach ? Alcuronium
Aminoglycosideans
Neostigmine
Suxamethonium
Tubocurarine
Q) Regarding MgSO4, the foll statement are true except :
a. it acts at motor end plate and reduces excitability of nerve
b. it acts on cell membrane and reduces calcium influx into

the cell.ans
c. it is use to prevent eclampsia
d. oliguria is a sign of magnesium toxicity..ans
e. calcium gluconate can be given if patient develops
cardiorespiratory depression
Q) Regarding transportation of drugs, following are true
except :
a. transcapillary movement is transfer of drugs with water due
to hydrostatic/ osmotic pressure
b. paracellular movement occurs between cell junction
c. passive transport is movement along a concentration
gradient
d. active transportation is movement against concentration
gradient and energy dependent
e. Facilitated diffusion is carrier mediated movement along
concentration gradient and energy dependent..ans
facilitated diffusion do not require energy. C is
true. passive diffusion do occur along
concentration gradient. active transportation
occurs against concentration gradient
Q) A 16-y is seen in the gyn OP with prim
amenorrhoea and excessive facial hair growth.
Examn reveals normal genitalia, apart from an
apparently large clitoris. DDx includes CAH.CAH
(21-OH defcy) is characterised by which of foll ?
A HTN, hypokalaemia and hyponatraemia
B Hypertension, hyperkalaemia and
hyponatraemia

C Hypotension, hyperkalaemia and


hypernatraemia
D Hypotension, hyperkalaemia and
hyponatraemia .ans
E Hypotension, hypokalaemia and hyponatraemia
Q) Acanthosis nigrecan most common seen in :
A. Uterian ca
B. Ovarian ca
C. Stomach ca ..ans
D. Skin ca
Q) From which germ layer does the Pancreas develop?
Neural tube of Ectoderm
Neural crest of Ectoderm
Intermediate Mesoderm
Coelom
Endoderm..ans
Q) A patient is found to have an end systolic left
ventricular volume of 30ml and an end diastolic
volume of 110ml. The pulse rate is 65/min.What
is the cardiac output, in litres per minute?
4.6
4.8
5.0
5.2ans
5.4
CO = SV HR c
SV = EDV- ESV

110-30=80 ml
Now 80 65=5200 ml=5.2 L
Q) The following is/are the correct
correspondents between male and female :
a. gubernaculum = ovarian ligament
b. gubernaculum = round ligament
c. both of the above.ans
d. penile glans = labia majora
e. scrotum = labia minora
Q) From which germ layer does myenteric plexus of GIT
developed: Neural crest of Ectoderm.ans
Neural tube of Ectoderm
Mesoderm
Endoderm
None of the above
Q) A 51-y has been referred by her GP for hot flushes and
night sweats. Her LMP was 6 w ago. She is concerned that she
may be going through the "change" and would like to go onto
HRT, because her symptoms are unbearable.Which of foll is
true regarding menopause?
Is associated with menorrhagia
Is synonymous with the climacteric
Occurs later in smokers
Occurs on average at age 45.5 years
Symptoms may occur long before menstruation ceases
ans

Q) A 52-y with Cushings syndrome is referred to the


preassessment clinic prior to a VH.Which of foll is a feature
of Cushings synd ? A Decreased plasma LDH
B Hypoglycaemia
C Hypokalaemia
D Hyponatraemia
E Metabolic alkalo
Cushings syndrome is a disorder of high serum cortisol, The
most common cause is exogenous administration of steroid
hormones. Cushings disease refers to Cushings syndrome
caused specifically by a tumour of the pituitary gland, which
secretes large amounts of adrenocorticotropic hormone,
leading to high cortisol. Patients may have hyperglycaemia
and insulin resistance, causing diabetes mellitus. All of the
above are potential features of Cushings syndrome, except
for hyponatraemia. Findingsof hyperglycaemia and
hypokalaemia may be accompanied by hyponatraemia as a
result of increased aldosterone levels.

Q) A 40-year-old woman undergoes amniocentesis for Down


syndrome screening. The karyotyping was normal; however,
alphafetoprotein was found to be elevated. Which is the most
likely pathology diagnosed by ultrasound scan? Cleft lip
Congenital diaphragmatic hernia
Microcephaly
Spina bifida..ans
Umbilical hernia
The answer is Spina bifida. Alphafetoprotein is a glycoprotein
synthesised by three fetal structures: the gut, liver and yolk

sac. It is may be elevated in a defect of the anterior abdominal


wall and the neural tube. As these defects are not covered by
skin, AFP freely enters the amniotic fluid from the fetal
circulation, leading to elevation of levels.
Q) During the development of midgut, which of
the following is true:
a. the midgut rotates a total of 270 degrees
clockwise
b. physiological herniation occurs during the first
90 degrees rotationans
c. intestinal loops re enter completely into the
abdominal cavity during the second 90 degree
rotation
d. the first part to re enter is the appendix
e. the last part to re enter is the proximal part of
jejunum
it rotates a total of 270 anticlockwise. the first part to enter is
the proximal jejunum and the last part is the caecal bud
( which then forms the appendix)
Q) What enzyme converts androstenedione to estradiol?
5 alpha reductase
Catalyse
CYP19 aromatase.ans
Transcriptase

Q) Glucagon is stimulated by which of foll?


Increased Keto-acids
Cholecystokinin..ans
Somatostatin
Raised Urea
Increased free fatty acids

Glucagon Stimulants
Hypoglycemia
Epinephrine
Arginine
Alanine
Acetylcholine

Cholecystokinin
Glucagon Inhibitors
Somatostatin
Insulin
Uraemia
Increased free fatty acids and keto acids into the blood
Q) You see a 32 year old women in clinic who mentions she
has been sweating and has frequent headaches. On
examination you note her blood pressure is 195/105 and pulse
rate is 110. You suspect pheochromocytoma.
Pheochromocytoma accounts for what percentage of cases of
hypertension? 0.1%........ans
0.5%
1%
2%
10%
Pheochromocytoma is rare accounting for around 0.1% of
cases of hypertension.
The rate of pheochromocytoma is quoted at around 1 in
54,000 pregnancies
20% are familial
The maternal mortality rate is 2% to 4% if diagnosed in the
antenatal period, rising to 14% to 25% if it is diagnosed
intrapartum or postpartum.

Q) autograft : tissue transferred from one part of


the body to another part in the same individual
isograft : tissue transfer in genetically identical
individuals
allograft : tissue transfer from a genetically
different but similar species
xenograft : tissue transfer between different

species

Q)

Ans Mixed Raised PCO2 and decreased HCO3 both


contributing to reduced pH
Q)Foll statements are correct regarding Doppler system
except - A.Can monitor fetal breathing movement
B.Can monitor fetal heart movement
C.Continues wave doppler have an excellent resolution..
ans
D.Duplex contains both continuous and pulsed waves
E.Can shift signals from blood vessels to be within the audible
sound waves

Q) What is definition of puberty in girls? Development of


pubic hair

Development of secondary sexual characteristics


Reaching stage 2 of breast development
Start of menses
Becoming capable of sexual reproduction.ans
Q)TSH is inhibited by which hormone? TRH
Prolactin
Growth hormone-releasing hormone (GHRH)
Somatostatinans
Follicle-stimulating hormone (FSH)

Q) Galactopoiesis is maintained by which hormone? Oxytocin


Follicle stimulating hormone
Human placental lactogen

Prolactin.ans
Luteinizing hormone
Q) With regard to the neuromuscular junction, what causes a
reduction in the release of acetylcholine?
A. Alcuronium
B. Aminoglycoside
C. Neostigmine
D. Suxamethonium
E. Tubocurarine
The answer is Aminoglycoside. All the other agents actions
are mediated on the postsynaptic membrane.
Q) Testosterone Binding
70% testosterone bound to SHBG,
25-30% testosterone bound to albumin
Percentage free testosterone can vary significantly
depending on the analysis method used. Typical laboratory
reference ranges are Male 1.5-3% and female approx 1%.
Q) A 23 year old primigravida at 32weeks of gestation
presented with very minimal per vaginal spotting for two
days. She is clinically stable. Scan reveal placenta previa
grade 3 posterior. Fetal status is good. Your consultation plans
for an expectant management. What is the definition of
expectant management?
a. To offer medical intervention as outpatient
b. To offer a surgical intervention
c. Offer no treatment and see if condition resolve
naturallyans

d. To offer medical intervention as in patient


e. none of the above
Q) You review a 30 year old women in clinic who's thyroid
function tests have demonstrated hypothyroidism. On taking a
detailed history she reports having felt tired for around 6
months and noticed some weight gain but there are no other
symptoms and no recent illness. Examination reveals a nontender hard goitre. Further bloods show anti-thyroid
peroxidase (anti-TPO) and also anti-thyroglobulin (anti-Tg)
antibodies are present. What is the likely diagnosis? De
Quervains thyroiditis
Graves disease
Toxic diffuse goitre
Hashimoto's..ans
Endemic goitres
Q) At ovulation the surge in LH causes rupture of the mature
oocyte via action on what?
Theca interna
Theca externaans
Granulosa interna
Granulosa externa
Large luteal cells
The luteinizing hormone (LH) surge during ovulation causes:
Increases cAMP resulting in increased progesterone and
PGF2 production
PGF2 causes contraction of theca externa smooth muscle
cells resulting in rupture of the mature oocyte

Ovarian Endocrine Function


Follicular Structure Luteal Structure Functions
Theca Cells Thecal Lutein Cells
(small luteal) Androgen (Androstenedione) production
Thecal Lutein cells produce progesterone
Granulosa Cells Granulosal Lutein Cells
(large luteal) Convert androgen to estradiol via aromatase
Granulosa Lutein cells produce progesterone
Roles of LH and FSH
FSH stimulates Aromatase production in the granulosa cells
LH stimulates Androgen production in the theca (interna)
cells
LH also stimulates the contraction of the smooth muscle
cells of the theca externa. This increases intrafollicular
pressure which results in rupture of the mature oocyte.
Q) You take some pre-op bloods on a patient with Addison's.
Which of foll abnormalities is mediated primarily by
glucocorticoid deficiency? Hypogylcaemiaans
Hyponatraemia
Hyperkalaemia
Hypercalcaemia
Metabolic acidosis
Remember in Addison's both glucocorticoid and
mineralocorticoid hormones are deficient.

The main mineralocorticoid is Aldosterone. Aldosterone


should drive the Na/K pump in the kidney and result in
sodium and water retention with potassium being excreted.
In aldosterone deficiency potassium rises (hyperkalaemia) as
it isn't adequately secreted whilst sodium and water are not
reabsorbed (leading to volume depletion & hyponatraemia).
Aldosterone should also drive H+ secretion in exchange for
potassium. In deficiency H+ accumulates leading to acidosis.
Volume depletion is thought to be the main cause of
hypercalcaemia.
Cortisol (hydrocortisone) is the main glucocorticoid. It
stimulates gluconeogenesis. Deficiency can therefore result in
hypoglycaemia.
Q) You are called to assist in an initially MW led delivery.
Upon delivering a female baby you notice the baby has partial
fusion of labioscrotal folds. You suspect CAH. Which of foll
genes is most likely to be mutated?Delta F508
HEF2A
CYP21A..ans
CYP11B1
BRCA1
Q)Which of foll statements best describes MOA of
benzodiazepines?
a. Benzodiazepines activate GABAB-receptors in spinal cord.
b. They inhibit GABA-transaminase leading to increased
levels of GABA.
c. They block glutamate receptors in hierarchical neuronal

pathways in the brain


d. Benzodiazepines increase freq of Cl--channels opening
which are coupled to GABAA receptors.ans
e. They are direct-acting GABA receptor agonists in the CNS.
Q)Foll should be considered in patient going for
c/s except :
a. administer histamine antagonist and antacid
b. operation table is positioned flat.ans, the
table should be tilted 15 degress to avoid
maternal hypotension from caval compression
c. ephidrine be given if hypotensive after regional
anaesthesia
d. antiemetics given preoperatively
e. general anaesthesia includes preoxygenation,
cricoid pressure and rapid sequence induction
Q) Hemorrhage produces which of foll ? A an
increase in CO
B causes splenic contraction
C inc aldosterone secretion ans
D vasodilation
E wide spread arteriolar dilation
Q) You review a 29 year old women in clinic who is noted to
have a raised prolactin. She takes the following repeat
medications: Amlodipine
Fluoxetineans
Omeprazole

Paracetamol
Ramipril
Q)You suspect a drug cause. Which of the above medications
is most likely to be responsible for her raised prolactin?
Amlodipine
Fluoxetine.ans
Omeprazole
Paracetamol
Ramipril
There are several causes of raised Prl
(hyperprolactinaemia):
Hypothyroidism
CRF
Liver disease
Pregnancy
Stress
Lactation
Chest wall stimulation & surgery
Drugs (Opiates, H2 antagonists e.g. Ranitidine,
SSRI's e.g. Fluoxetine, Verapamil, Atenolol, some
antipsychotics e.g risperidone and haloperidol,
Amitriptyline, Methyldopa and Oestragen
conatining compounds)
Hypothalamus tumours
Prolactinoma
Agromegaly
PCOS

Q) The NICE criteria which were changed in


February 2015. According to NICE FPG of
5.6mmol/l or above and 2 hour glucose of 7.8
mmol/l or above is diagnostic.
You really need to read the question here as you
may be asked for the WHO criteria where the
levels are FPG = 7.0 mmol/l and 2HG = 11.1
mmol/l
Q)Factors that reduce LSCS rate:
- partogram with 4 hour action line
- IOL beyond 41 week
- involvement of consultant in making decision
for LSCS
- fetal scalp sampling
factor increase : CTG
factor does not affect :
-walking in labour
-non-supine position during the second stage of
labour
-immersion in water during labour
-epidural analgesia during labour
- the use of rasberry leaves
- early amniotomy
- active management of labour

Q)Which of foll is NOT a recognised cause of


hyperprolactinaemia?H2 antagonists
Hyperthyroidismans
Chest wall surgery
PCOS
Pregnancy
Q) Aldosterone is responsible for approximately what
percentage of mineralocorticoid activity in the human body?
<5%
25%
50%
75%
>90%..........ans
Q) Which of foll would be typical of a patients with Addison's
Hypertension, hypokalaemia, hypernatraemia
Hypotension, hyperkalaemia, hyponatraemia.ans
Hypertension, hyperkalaemia, hyponatraemia
Hypotension, hyperkalaemia, hypernatraemia
Hypotension, hypokalaemia, hyponatraemia

Q)

Ans C
Q) 24 years old patient who is 10 week pregnant presented
with per vaginal bleed and passing out vesicular like structure.
Per abdomen, uterus is palpable at 14 week size. You
diagnosed her with a complete molar pregnancy. Following
are true except :
a. you expect a snow storm appearance on ultrasound
b. you may expect a theca lutein cyst on ultrasound
c. it is 46XX of both paternal origin
d. It is 46XY of both paternal origin
e. it is 69XXX of all paternal origin.ans

Q) Regarding hormone levels in PCOS which of the


following is typical
Elevated FSH
Elevated SHBG
FSH:LH ratio increased typically >2
LH:FSH ratio increased typically >2ans
None of the above
Q)

Ans A
Q) A patient is seen in clinic 6 w pp. The pregnancy was
complicated by intrapartum haemorrhage requiring fluid
resuscitation and a 2 U BT. Mum reports feeling very tired,
struggling to lose baby wt and needing to bottle feed due to
very low volume lactation. What is suspected diagnosis?
Hyperpituitarism

Delayed transfusion reaction


Idiopathic hypothyroidism
Sheehan's Syndrome
DM
Q) Which of the foll causes a decrease in SHBG?
COCP
Hyperthyroidism
Anorexia
Pregnancy
Obesity..ans
Q) Conn's syndrome is the result of what?
Aldosterone deficiency
Aldosterone hypersecretion.ans
Cortisol deficiency
Cortisol hypersecretion
Pregnenolone deficiency
Q) What is mc cause of acromegaly? MEN I
MEN II
Pituitary infarct
Pituitary adenocarcinoma
Pituitary adenoma.ans
Q) A patient develops hypocalcaemia as a result of
pancreatitis. What is the appropriate homeostatic response to
hypocalcaemia?
Increased PTH, Increased 1,25 dihydroxycholecalciferol &
Increased phosphate

Inc PTH, Increased 1,25 dihydroxycholecalciferol & Dec


phosphate..ans
Increased PTH, Decreased 1,25 dihydroxycholecalciferol &
Decreased phosphate
Decreased PTH, Decreased 1,25 dihydroxycholecalciferol &
Increased phosphate
Decreased PTH, Increased 1,25 dihydroxycholecalciferol &
Increased phosphate
Low calcium should stimulate PTH production by
the parathyroids
PTH increases hydroxylation of vitamin D. It also
increases resorption of calcium in the kidney in
exchange for phosphate hence the phosphate
level drops.
Q) Which of the following changes would you expect in
pregnancy?
Increased TSH, Increased Total T3 and T4
Increased TSH Decreased Total T3 and T4
Decreased TSH Increased Total T3 and T4ans
Decreased TSH Decreased Total T3 and T4
No change TSH Increased Total T3 and T4

Q)

Q) What role does messenger RNA play in the synthesis of


proteins?
A. Transcription
B. Reverse transcription
C. Translation
D. Reproduction
E. Replication
ANSWER: Transcription

Q)

Q)

Q) You see a 47 y in clinic who mentions she has been


sweating and has frequent headaches. On examn you note her
BP 195/105 and P 110. You organise a scan that shows an
adrenal mass. Which cell type is this mass likely to arisen
from? Chromaffin cellsans
Epsilon cell
Zona reticularis
Ganglion Cells
Oxyphil Cell
Q) Glucagon works by which of foll mechs to reverse
hypoglycemia? A. Glycolysis
B. Glycogenesis
C. Gluconeogenesis..ans

D. krebs cycle
E. Oxidative phosphorylation
Q) A 17 years old women is reffered by primary
care physician for the evaluation of primary
amenorrhea . on physical examination pt has sign
of virilization. She has also a pelvic mass . During
the work up of pt she is found to have sex
chromosome mosaicism (45x/46 XY )
A)immature teratoma
B)serous adenocarcinoma
C)sertoli leydig cell
D)granulosa cell tumor
E)gonadoblastoma..ans

Q) Testis determining factor

Q)
Q) Which pharyngeal arch is closest to the head of the
embryo? 1stans
2nd
3rd
4th
6th
Q) What process changes a spermatogonium into a prim
spermatocyte?Meiosis 1
Meiosis 2

Differentiation
Maturation.ans
Spermosis
Spermatogonium mature or grow into primary
spermatocytes (this process is also called
spermatocytogenesis)
Spermatocytes undergo 2 meiotic divisions to become
spermatids (this process is also called spermatidogenesis)
Spermatids differentiate into spermatozoa (this process is
also called spermiogenesis)
Q) Ootidogenesis refers to which process during Oogenesis?
1st Meiotic Division
2nd Meiotic Division
1st and 2nd Meiotic Divisions.ans
Differentiation
Growth and maturation
Oogonium become Primary Oocyte via Growth/Maturation.
This process is called oocytogenesis
Primary Oocyte undergoes 2 meiotic divisions to become
Ootids. This process is called Ootidogenesis
Ootids differentiate into Ovum
Q) What process would you expect a secondary spermatocyte
to undergo during spermatogenesis? 1st Meiotc Division
2nd Meiotc Division.ans
1st Mitotic Division
2nd Mitotic Division
Differentiation

-Secondary spermatocytes undergo the 2nd meiotic division


Primary spermatocytes undergo the 1st (or primary) meiotic
division
NOTE The process by which sperm undergoes the 2 meiotic
divisions is sometimes referred to as spermatidogenesis
because spermatids are generated by the process!

Q)

Q) During formation of the mid, fore and hind guts the initial
gut tube undegoes rotation. Which of the following is correct
regarding rotation of the midgut during this process?
rotates 90 degrees clockwise
rotates 130 degrees anticlockwise
rotates 180 degrees clockwise
rotates 270 degrees anticlockwise..ans
rotates 270 degrees clockwise

During development of the fetal gut the intestine moves


outside of the embryonic abdomen herniating into the base of
the umbilical cord. This occurs at around 8 weeks gestational
age. When the bowel is within the umbilical cord, the midgut
rotates 90 degrees anti-clockwise. At approximately 10 weeks
the abdomen enlarges and the intestines return to the
abdominal cavity. Once returned to the abdomen the midgut
rotates an additional 180 degrees anti-clockwise completing a
270 degree anticlockwise rotation. It then fixes to the
retroperitoneum.

Q) At what age does physiological gut herniation occur? 1


week
3 weeks

5 weeks
8 weeks..ans
12 weeks

Q) Testosterone Binding : 70% testosterone


bound to SHBG,
25-30% testosterone bound to albumin
Percentage free testosterone can vary
significantly depending on the analysis method
used. Typical laboratory reference ranges are
Male 1.5-3% and female approx 1%.
Q) The round ligament develops from which structure?

Gubernaculumans
Urachus
Allantois
Medial umbilical ligament
Median umbilical ligament
Q) Whilst reviewing a 34 year old patient with amenorrhoea
in clinic they tell you they have gained over 10kg in weight in
the past 8 weeks and have noticed worsening acne. Routine
bloods taken that morning show a random glucose
11.1mmol/l, normal thyroid function tests and negative
pregnancy test. BP is 168/96 mmHg.
Addisons disease
Addisonian crisis
Cushing's syndromeans
Flammer syndrome
Hyperthyroidism
Q) Which pharyngeal arch gives rise to no muscular or
skeletal structures? 1st
2nd
3rd
4th
5thans
Q)Branches V2 and V3 of Trigeminal nv develop from which
pharyngeal arch? 1st..ans
2nd
3rd
4th
6th

Q) The 2nd pharyngeal arch gives rise to which of foll


structures? Vagus Nerve
Thyroid cardilage
Muscles of facial expression.ans
Intrinsic muscles of larynx
Trigeminal Nerve

Q) A Bicornuate uterus is due to which of the following?


Crossed fused ectopia
Abnormal fusion and reabsorption of the paramesonephric
ductsans
PKD-1 gene mutations
Mullerian agenesis
None of the above
A Bicornuate uterus (heart shaped) is the result
of abnormal fusion and reabsorption of the
paramesonephric ducts during embryogenesis.
Mullerian agenesis typically results in failure to
form a uterus. Mullerian duct fusion abnormalities
can cause a bicornuate malformation.

PKD-1 gene abnormalities are associated with


PCOS.
Crossed fused ectopia result in Horseshoe kidney.
Q) Somites are derived from which germ layer? Endoderm
Ectoderm
Intermediate Mesoderm
Paraaxial Mesoderm..ans
Lateral Plate Mesoderm

Somites are bilaterally paired blocks of paraxial


mesoderm. They form along the head to tail axis
of the developing embryo as shown in the image.

In the developing embryo, somites split to form


dermis of the skin (dermatomes), skeletal muscle
(myotomes), tendons and cartilage (syndetomes)
and bone (sclerotomes).
Q) What is prevalence of HIV in UK obstetric population?
0.5 per 1000 live births
2 per 1000 live birthsans
1 per 10,000 live births
2 per 10,000 live births
5 per 10,000 live births
Q) what is largest immune cell found in placenta?
a. T cells
b. B cells
c. neutrophils
d. dendritic cells
e. natural killer cells.ans
placenta is rich with specialised NK cells. normal NK cells
are CD 16 positive. But the NK cells in placenta are CD 16
negative and CD 56 positive. Activation of CD56 positive NK
cells causes production of cytokines,chemokines and
angiogenics factors that facilitate trophoblastic invasion,
instead of destroying it.
Q) With regard to the cell cycle. In what phase do nuclear
envelopes form around daughter chromosomes? Anaphase
Telophase..ans
Cytokinesis

Metaphase
Interphase
Mitosis is where cell division occurs and this
happens in a the following phases:
Prophase Chromatin condenses to
chromosomes (paired as chromatids). Mitotic
spindle forms
Metaphase Chromatids align at the equatorial
plane AKA the metaphase plate
Anaphase Chromotids pulled apart into 2
constituent daughter chromosomes
Telophase New nuclear envelopes form around
each daughter chromosome
Cytokinesis Cells divide

Q) In immunology, there are two wings,namely the innate and


the adaptive immune system. The interaction between this two
wings is essential to establish an effective immune response.
Which cell acts as a bridge for this two wings? a. dendritic
cells
b. natural killer cellsans
c. neutrophils
d. T cells
e. B cells

Q) Which antibody is predominantly responsible in a


secondary immune response?
a. Ig G..ans
b. Ig M
c. Ig A
d. Ig D
e. Ig E
Q) Which part of an antibody determines its specificity? a.
Fab .. ans
b. Fc
c. heavy chain
d. light chain
e. all of the above

Q) What is the main goal of the complement system?


a. formation of antigen-antibody complex
b. formation of complement- antibody complex
c. stimulate apoptosis
d. formation of DNA inhibition
e. formation of membrane attack complexans
all three pathways in the complement system will finally
converge at the final lytic sequence ( C5,C6,C7,C8,C9). They
sequentially interact to form a membrane attack complex,
which binds to the membrane of the target cell , forming
trans-membrane channel through which salts and water can
flow in, resulting in lysis of target cell.

Q)

Ans D
Q) What is the causative organism of Scarlet Fever?
Staphylococcus aureus
Streptococcus epidermidis
Streptococcus pyogenes.ans

Streptococcus Pneumonia
Parvovirus B19
Q) Which of foll best describes MOA of Ondansetron?
Histamine H1-receptor agonist
Histamine H1-receptor antagonist
Serotonin 5-HT3 receptor antagonistans
Dopamine D2 receptor agonist
Dopamine D2 receptor antagonist
Q) Lidocaine works by blocking which of the following ion
channels?
slow voltage gated sodium channels
fast voltage gated sodium channelsans
slow voltage gated potassium channels
fast voltage gated potassium channels
fast voltage gated calcium channels
Q) Which of the following drugs is most appropriate to use to
stimulate stimulate lactation?
Cabergoline
Bromocriptine
Domperidone.ans
Sertraline
Metoclopramide
Domperidone and Metoclopramide are
recognised galactagogues (substances that
increase milk production). UK Medical Information

(UKMI) who reviewed the evidence on


galactagogues advise that domperidone is the
drug of choice.
Q) A patient is having PPH and you urgently
asked for blood products. Your intern rushed to
the blood bank and came back with packed cells,
cryopercipitate and fresh frozen plasma.
Following are the contents of cryopercipitate
except :
a. factor VIII
b. factor I
c. von willebrand factor
d. factor XIII
e. factor II..ans
factor II (thrombin) is not found in cryo. Cryo is
rich in fibrinogen ( factor I).
Q) . Robertsonian translocations result from
which of the following?
A. Breaks at or near the centromeres of two
acrocentric chromosomes followed by the
reciprocal exchange of broken partsans
B. A part of one chromosome becomes attached
to a non-homologous chromosome
C. Unequal crossing over during meiosis
D. The fusion of two small chromosomes end-toend such that a double centromere occurs

Q) What is the role of DHEA produced by the fetal adrenal


glands?
Stimulate formation of cholesterol
Stimulate placenta to form oestragen..ans
Stimulate development of the Thymus
Stimulate gonadal development of the fetus
None of the above

Q) With regard to opioid receptors. Morphine acts primarily


as a potent? Mu receptor agonistans
Mu receptor antagonist
Kappa receptor agonist

Kappa receptor antgonist


Delta receptor agonist
Q) With regard to opioid receptors. Fentanyl acts primarily as
a potent? Beta receptor agonist
Delta receptor agonist
Gamma receptor agonist
Kappa receptor agonist
Mu receptor agonistans
Opoids
Opoids may act at Mu, Kappa or Delta receptors.
Strong opioids commonly used in clinical practice i.e.
Morphine
Fentanyl
Methadone
are strong agonists of the Mu receptor.
Weak opioids such as codeine and tramadol are also agonists
of the Mu receptor but also act as agonists at the Delta and
Kappa receptors
Q) Which of the following best describes the mechanism of
action of Cyclizine?
Dopamine D2 receptor agonist
Dopamine D2 receptor antagonist

Serotonin 5-HT3 receptor antagonist


Histamine H1-receptor agonist
Histamine H1-receptor antagonistans
Q) What type of virus is HIVRubivirus
Hepacivirus
Lentivirus..ans
Flavivirus
Enterovirus

Q) Which of the following is used in protein analysis?


Northern Blotting
Eastern Blotting
Southern Blotting

Western Blotting..ans
Southwestern Blotting

Q) You are asked to review a 65 year old ladies legs preoperatively. You diagnose cellulitis. What is the most common
causative organism?
Staphylococcus Aureus
Staphylococcus Epidermidis
Clostridium Perfringens
Streptococcus Mutans
Streptococcus Pyogenes.ans
Q) What is the incidence of placenta accrete (including increta
and percreta) in the UK?
1.0 per 1000 deliveries
2.3 per 1000 deliveries
7 per 1000 deliveries
1.7 per 10,000 deliveries . ans
17 per 10,000 deliveries
Q) How can the mechanism of action of the
immunosupressive agent azathioprine best be described?
A. Blocking of T cell activation
B. Inhibition of nucleic acid synthesis in all mitotic cells
C. Inhibition of T cell proliferation
D. Prevention of the generation of cytotoxic effector cells and
general anti-inflammatory effect
E. Promotion of unresponsiveness in alloreactive T cells

The answer is Inhibition of nucleic acid synthesis in all


mitotic cells. Corticosteroids prevent generation of cytotoxic
effector cells and produce a generalised anti-inflammatory
response. Tacrolimus blocks T cell activation.
Q) A diabetic women attends your preconception clinic. You
have checked her HBA1C. According to NICE guidelines
what HBA1C level would prompt you to strongly advise this
women NOT to get pregnant due to the signifiant risks it
presents?
>6.5% or 48mmol/mol
>7.6% or 60mmol/mol
>10.0% or 86mmol/mol ..ans
>11.1% or 98mmol/mol
>12.6% or 114mmol/mol
Q) Fentanyl is approximately how many times more potent
than morphine? 2
5
10
20
100 ans

Q)

A-EXPIRATORY RESERVE VOLUME


B-INSPIRATORY RESERVE VOLUME
C-TIDAL VOLUME

D-VITAL CAPACITYans
E-RESIDUAL VOLUME
Q) Which test is most sensitive in detecting PID? A. Cell
culture
B. Endocervical biopsy
C. Enzyme immunoassay
D. Microscopy
E. Nucleic acid amplification test ans
The answer is Nucleic acid amplification test (NAAT). NAAT
(polymerase chain reaction or strand displacement
amplification) is more than 95% sensitive in detecting
Chlamydia or Gonorrhoea from the endocervical specimen.
The absence of endocervical or vaginal pus cells on a wetmount smear has a good negative predictive value (95%) for a
diagnosis of PID but their presence is non-specific (poor
positive predictive value).
NAAT is more sensitive and specific than enzyme
immunoassay (EIA) and the Department of Health has
advised that the use of sub-optimal EIA is no longer
appropriate and has provided funding to support laboratories
moving from EIA to NAAT. However, no test is 100%
sensitive or specific.
Q) Patient was diagnosed of having right ovarian
teratoma of 6x8cm. RMI score is 25. Patient was
counselled for surgery and opted for laparoscopic
cystectomy. What is the mc nerve injury
associated with this procedure? a. sciatic nerve

b. femoral nerve
c. obturator nerveans
d. genitofemoral nerve
e. ilioinguinal nerve
ovarian fossa is in close relationship to the
obturator nerve. post operatively, patient will
have difficulties in adducting her lower limb.
Q) You reviewed a 45 years old patient in your
clinic. She is diagnosed of having a huge
submucosal uterine fibroid. She is planned for a
total abdominal hysterectomy. You explained the
risk of bleeding and blood transfusion. Further
history revealed that she is a Jehovah's witness.
What option you may offer in regards to her
operation?
a. autologous blood donation preoperatively
b. intraoperative blood salvage
c. recombinant erythropoietin therapy
d. all of the above.ans
e. get a family member of her similar blood group
to donate
Q) A fetus is formed from partial maternal and
partial paternal genes. A fetus is, therefore,
considered foreign to the maternal's immune
system. However, the fetus escapes maternal
immunity because of a few reasons. Following are
the reason :

a. cytotrophoblast has HLA-G


b. cytotrophoblast has HLA-E
c. cytotrophoblast has HLA-C
d. Natural killer cells are negative of CD16 but
positive for CD56
e. All of the above.ans
Q) What is the main plasma Cation? Sodium..ans
Chloride
Potassium
Calcium
Phosphate

Q)

Q)
Q) Which of foll factors shifts O2 dissociation curve to Lt ?
Increased temperature
Increased partial pressure CO..ans
Increased [H+]
Increased 2,3 DPG
Increased partial pressure CO2

Q) Which antibody has the highest serum


concentration? a. Ig M
b. Ig Gans, lowest concentration is Ig E
c. Ig A
d. Ig E
e. Ig D
Q) Which pathogen is mc causative org in female UTI?
Chlamydia trachomatis
Staphylococcus Aureus
Escherichia Coli..ans
Neisseria gonorrhoea
Gardnerella vaginalis
Q) At birth which of foll circulatory changes occur?
20 fold rise in lung blood flow
Rise in right atrial pressure
Anatomical closure of ductus arteriosus
Anatomical closure of ductus vensosus
Flap closure of the fora men ovale.ans
Q)Most appropriate drug regimen for empiric Rx of Gm - ve
bacilli in blood is...? a. Ampicillin plus sulbactam.
b. Aztreonam..ans
c. Cefazolin.
d. Imipenem plus cilastatin.
e. Ticarcillin plus clavulanic acid.

Q) With regard to the cell cycle. In which part of the cycle


does DNA replication occur? G0
Interphase..ans
Anaphase
Telophase
Cytokinesis

Q) Buffers : Blood... HB

Urine... PO4
ECF... HCO3

Q) High levels of which Ig are found in breast milk? A.


IgA..ans

B. IgD
C. IgE
D. IgG
E. IgM
The correct answer is IgA.
Q) Which is the most abundant complement
protein? a. C2
b. C3.ans
c. C4
d. C5
e. C9
complement C3 is the most abundant. cleavage
of this protein will then be followed by activation
of the lytic sequence.
Q) Which immunoglobulin has the highest
molecular weight? a. IgA
b. IgD
c. IgE
d. IgG
e. IgM.ans
IgGsmallest

Q) Which of the following organelles is responsible for


holding mRNA for translation into protein : Nucleus
Nucleolus
Golgi Apparatus
Ribosome.ans
Smooth ER

Q) Fibrinogen is activated by which of foll ? Factor X


Prothrombin
Thrombinans
Factor Xa
Tissue Factor

Q) What is the main biochemical buffer in urine? Bicarbonate


Phosphateans
Ammonia
Calcium Carbonate
Protein
Q) What is main biochemical buffer in blood?
Bicarbonate.ans
Ammonia
Calcium Carbonate
Proteins
Phosphate
Q) Which of foll is most appropriate for diagnosis of Bacterial
Vaginosis (BV)?
Amsel's criteria..ans
Fitz-Hugh-Curtis score
Gardnerella vaginalis wet prep culture

Gardnerella vaginalis PCR


SAPS III score
Q) T cell cannot respond to protein antigen unless they have
been processed and presented as short peptide, complexed
wtih major histocompatibility (MHC) molecules,on the
surface of an antigen presenting cells. Which is responsible in
encoding MHC?
a. IL 12
b. IL 13
c. HLA ..ans
d. TNF
e. IFN gamma

Q) What biochemical process utilises pyruvate to form ATP?


Glycolysis
Gluconeogenesis
Calvin Cycle
Krebs Cycleans
Glycogenolysis
Q) Regarding pelvic Gonorrhoea infection in women. What
percentage of cases are asymptomatic? 5%
15%
30%
50%.........ans
90%
Q) Which of the following is a live attenuated vaccine? a.
BCG
b. MMR
c. both of the aboveans
d. hepatitis A
e. tetanus
hepatitis A is killed vaccine. tetanus is anti toxin.
Q) The COCP causes all of foll biochemical effects EXCEPT
which one?
Suppress LH
Elevate FSH.ans
Decreased ovarian androgen synthesis
Elevate Sex Hormone Binding Globulin
Reduction in adrenal androgen secretion

Q) Cytokines act in the following way : a. autocrine


b. endocrine
c. paracrine
d. all of the aboveans
e. exocrine
Q) Ultrasound picture of the ovary shows cystic lesion
measuring 3.3x 2.5 cm with echogenic septea. What is the
most likely finding?
A. Hemorragic cyst
B. benign teratoma
C. ovarian carcinoma
D. serous cystadenoma
E. Follicular cyst
Q) Which of the below drugs, induce hepatic
enzyme and reduce the efficacy of COCP
Fluconazole
Na valproate
Carbamazepineans
Ampicillin
Gabapentine
Q) In depth understanding of the immune system has cause a
revolution in the development of good vaccines. What is the
wanted end product from a vaccination?
a. promotes T cells production
b. promotes dendritic cells production
c. production of memory B cells.ans

d. promotes more sensitive neutrophils


e. all of the above
The goal of vaccination is to produce memory plasma cells, in
which, following exposure to the particular antigen, the
memory cells will be able to mount a quick and effective
secondary response, leading to protection from and
elimination of the pathogen.
Q)

Ans B
Q) Vitamin K is essential for the synthesis of : Vitamin A
Vitamin D
Clotting factor III
Clotting factor IV
Protein Sans

Q) What is the maximum dose of lidocaine with adrenaline?


1mg/kg
3mg/kg
5mg/kg
7mg/kg.ans
10mg/kg
Q) A 28 year old women is seen in the early pregnancy unit.
She has had a positive pregnancy test but is uncertain of her
LMP. Ultrasound doesn't visualise a pregnancy. You perform a
progesterone test. The result is 59 nmol/l. According to NICE
guidelines what level is consistent with viable pregnancy? >5
nmol/l
<15 nmol/l
>25 nmol/l

>60 nmol/l
None of the aboveans
Progesterone testing
Note there is a significant difference between what NICE
advise and advice that can be found on the RCOG website
regarding progesterone testing in Pregnancy of unknown
location
NICE advise do not use serum progesterone measurements as
an adjunct to diagnose either viable intrauterine pregnancy or
ectopic pregnancy
RCOG advise progesterone levels <20nmol/l strongly
suggestive non-viable pregnancy and levels >60nmol/l
suggestive intrauterine pregnancy
Q) Which cell adhesion molecules are dependent on calcium
ions to function?
E-selectin
P-selectin
Cadherinsans
Integrins
Kindlin-1

Q)In the foetus where is principle site of AFP production :


YS.T
Bone Marrow
GIT.T
Spleen
Liver..T,main,primarily
Q)Which diabetic complication worsened by
preg? A)neuropathy
B)proliferative retinopathy.ans
C)nephropathy
D)benign retinopathy
E)gasteroparesis
Q) Examination is normal for a 12-y . Microscopic
examn of discharge shows no e/o pseudohyphae,
clue cells, or trichomonas. Which of foll is most

likely diagnosis ? A. BV
B. Candida vulvovaginitis
C. Physiologic leukorrheaans
D. Syphilis
E. Trichomoniasis
Physiologic leukorrhea can be seen during 2
different periods of childhood. Some female
neonates develop a physiologic leukorrhea
shortly after birth as maternal circulating
estrogens stimulate the newborn's endocervical
glands and vaginal epithelium. The discharge in
these neonates is often gray and gelatinous.
Physiologic leukorrhea can also be seen during
the months preceding menarche. During this
time, rising estrogen levels lead to whitish
discharge not assoc
with any symptoms of irritation. This patient has
a whitish discharge, no other symptoms, and she
has had normal pubertal development up to this
point. The discharge itself has no characteristics
of infection.
Therefore, physiologic leukorrhea is the most
likely diagnosis.
BV (choice A) is not the most likely diagnosis in
this patient because the discharge is not
malodorous and there are no clue cells seen on
microscopic examination of the discharge.

Candida vulvovaginitis (choice B) is not the most


likely diagnosis because the discharge is not
thick and white (or "cottage-cheese"-like) and the
patient has no irritative symptomatology.
Syphilis (choice D) most often presents with a
painless ulcer (called a chancre) or is found with
serologic testing. A nonmalodorous, whitish
vaginal discharge in a 12- y female who is not
sexually active is almost certainly not
e/o syphilis.
Trichomoniasis (choice E) is also highly unlikely in
this patient and the lack of trichomonads on
microscopic examn effectively rules out this
diagnosis.
Q) The most frequent cause of haematosalpinx is :
A -Salpingitis
B -Tubal pregnancy.ans
C- Ectopic pregnancy
D- Adenomyosis
E- Streptococcus
Q) % of fetuses with trisomy 21.Would be wxpected to be
detected by nuchul transluency alone : 10%
25-30%
50-55%
70-75%..............ans
90-95%

Q) Which of the following inhalational anesthetic agents is


having fastest onset of action? a. Nitric oxide.
b. Isoflurane.
c. Nitrous oxideans
d. Enflurane.
e. Nitrogen dioxide.
Q) what percentage of of ovary are bilateral?1%
5%
10%................ans
15%
20%
Q) What is the maximum dose of lidocaine that should be
used on a 70kg woman who is 12 weeks pregnant?
0 - lidocaine is CI in pregnancy
10ml 1% lidocaine
21ml 1% lidocaineans
70ml 1% lidocaine
100ml 1% lidocaine
1% lidocaine =1g/100mL or 10mg/mL
Max dose for a 70kg patient is 210mg (3mg/kg)
21mL 1% plain lignocaine
Lidocaine
Lidocaine Key Points

Blocks fast voltage gated sodium channels


Anti-arrhythmic
Half life 2 hours
Safe to use in pregnancy
Hepatic metabolism
Max dose is 3mg/kg (7mg/kg with adrenaline)
Q) Which of foll metabolic processes produces
most energy needed for uterine contraction
during parturition?
A.
Aerobic glycolysis
B. Anaerobic glycolysis
C. krebs cycle
D. Oxidative phosphorylation.ans
E. Beta oxidation
Q) Which one of foll best describes the
histological type of endometrium soon after
ovulation? A. Decidual reaction
B. Atrophic endometrium
C. Proliferative endometrium
D. Secretory endometrium..ans
E. Arias-Stella phenomenon

Q)

Ans B
Q) Plasma concentration of estrogen increases throughout
pregnancy. By 40 weeks, the concentration of estriol increased
to estradiol by who much times? A. 100 times
B. 50 times
C. 20 timesans
D. 10 times
E. 5 times

Q)

Ans B
Q) An active 70-y F is admitted to the gyn clinic for a sling
procedure with a known h/o Pagets disease of bone. Which
biochemical findings are most likely?
Reduced calcium, reduced phosphate and elevated ALP
Reduced calcium, elevated phosphate and elevated ALP
Normal calcium, normal phosphate and elevated ALP.ans
Elevated calcium, reduced phosphate and reduced ALP
Elevated calcium, elevated phosphate and elevated ALP
Q) Identify which of foll gives correct pathway of piriformis
muscle.
Origin
Exits pelvis via
Insertion
A. Ventral surface of sacrum Greater siactic notch Greater
trochanter of femur..ans

B. Ventral surface of sacrum Lesser siactic notch Greater


trochanter of femur
C. Ventral surface of sacrum Greater siactic notch Lesser
trochanter of femur
D. Dorsal surface of sacrum Greater siactic notch Greater
trochanter of femur
E. Dorsal surface of sacrum Lesser siactic notch Lesser
trochanter of femur
Q) Meconium is found in the fetal gut from what gestational
age? A. 6 weeks
B. 10 weeks . ans
C. 18 weeks
D. 32 weeks
E. 35 weeks
The answer is 10 weeks. Meconium is found in the fetal gut
from 10 weeks of gestation but the passage of meconium is
rare before 34 weeks of gestation.

Q)

Ans D
Q) How can MOA of ranitidine best be described?
It delays gastric emptying
It hastens the gastric emptying
It mainly has an antiemetic action
It neutralises the acidic content of the stomach
It reduces the acid content and production by gastric parietal
cells..ans
The answer is It reduces the acid content and production by
gastric parietal cells. It is used preoperatively to reduce the
risk of aspiration pneumonia and Mendelsons syndrome.

Q) Gonads develop from which cell layer? A. Endoderm


B. Intermediate mesoderm .. ans
C. Lateral mesoderm
D. Paraxial mesoderm
E. Surface ectoderm
The answer is Intermediate mesoderm. The gonads and
urogenital system develop from the intermediate mesoderm.
Skeletal muscles, the skeleton, the dermis and connective
tissue develop from paraxial mesoderm. The lateral plate
mesoderm develops into the serous membrane of the body.
Q)Mc organism that cause sepsis during pregnancy ?
A. Group A streptococcians
B. Group B sterptocooci
C. Staph. aureus
D. E. coli
E. MRSA
infection, her symptoms are often atypical, with extremes of
temperature, unusual and vague pain, and pain in the
extremities. Imaging may appear normal, but removing a
small sample from the uterus along with a blood culture may
be a useful, rapid diagnostic tool.
Q) A 15-y F visits her GP requesting a method of
contraception. When taking her hist, you discover a personal
h/o thromboembolic events. She has had 3 sexual partners in 5
mths. She also c/o dysmenorrhea. What 1st -line Rx would you
recommend for painful periods? COCP
Ibuprofen..ans

Mefanamic acid
Tramadol
Tranexamic acid
As a 1st line Rx, it would be appropriate to use a Rx that
woman can purchase over counter.
Q) A 15-year-old patient visits her GP requesting a method of
contraception. When taking her history, you discover a
personal history of thromboembolic events. She has had three
sexual partners in 5 months. She also complains of
dysmenorrhea. What type of contraception would you
recommend? Barrierans
COCP
Female sterilisation
IUCD
Subdermal contraceptive implant
Q) Which structure is prim mech for shunting blood away
from fetal pulm circulation? FO..ans
Ductus Arteriosis
Arteria Umbilicalis
Ductus Venosus
Umbilical Arteries

Blood enters the right atrium of the fetal heart and most
passes through the foramen ovale into the left atrium. From
there it is pumped through the aorta. The foramen ovale is the
major structure for bypassing the fetal pulmonary circulation.

Q) At what gestational age do fetal breathing movements


commence? A. 12 w ans
B. 20 weeks
C. 30 weeks
D. 36 weeks
E. 0 weeks
The answer is 12 weeks. The movements
increase in frequency and strength further on in
the gestational period. The movements also
increase after maternal meals, acidosis and are
reduced by hypoxia, maternal alcohol
consumption and sedative drugs.
Q) Definition of prolong jaundice in term baby ?
A lasting for over 72 hours
B lasting for 7 days
C lasting for 14 days ans,Preterm 21 d
D lasting for 21 days
E lasting for 28 days
Q) You review a 58 year old patient in clinic. She asks what
the results of her recent DEXA scan are. You note her hip
BMD hip T-score is -1.4. You note she has a history of
olecranon fracture 4 years ago. What is her classification
according to WHO criteria?
Normal bone density
Osteopenia..ans
Severe Osteopenia

Osteoporosis
Severe Osteoporosis

Q) Till which gestational age we will find Grower 1


and 2 HB? 10 12 w
Q) The thyroid gland is the first of the body's
endocrine glands to develop, on approximately
the 24th day of gestation. The thyroid originates
from two main structures: the primitive pharynx
and the neural crest. The rudimentary lateral
thyroid develops from neural crest cells, while the
median thyroid, which forms the bulk of the
gland, arises from the primitive pharynx.
Q) What is best indicator of gest age on a dating scan in 1st
trimester? A. BPD
B. CRL ans
C. Femur length
D. Gestational sac diameter
E. Head circumference
The answer is Crown rump length. Biparietal
diameter is appropriate after 14 weeks.
Gestational sac measurements help in early
pregnancy diagnosis
Q) In the development of the oocyte, when does the second
meiotic division occur? A. At birth
B. At fertilisation ans

C. At menarche
D. At ovulation
E. Before fertilisation
The answer is at fertilisation. The primary oocyte
resumes meiotic maturation in response to a
surge in luteinising hormone. This results in a
polar body and the secondary oocyte. The oocyte
is then ovulated and enters the second meiotic
cycle. It is arrested in metaphase II and only
completes meiosis after the entry of the
spermatozoa at fertilisation.
Q)Serum levels of Li are inc in women taking this
anti -HTN ? A hydralizine
B methyl dopa
C labetalol
D amylodipine
E captoprilans
Captopril increase serum lithium level
Methyldopa also causes lithium toxicity
Q) A 55 yr old man visits his primary care physcian with a
complaint of urinary in frequency. Examination finds a 1 cm
nodule on his prostate gland.the physcian orders PSA serum
test.by common standards PSA level > 4 ng /ml is considered
abnormal.using this standard this test has a sensitivity of 80%
and specificity of 90%.a recently published epidemiological
article found in a cross sectional study 10% men of this age
have prostate cancer.the results on the pts PSA is 7 ng/ml.what

is your best estimate of likelihood that this man actually has


prostate cancer? A. 13%
B.25%
C.36%
D.47%...........ans
E.58%
F.69%
Q)

Ans C

Q)

Ans B
Q) Which part of the ovary is responsible for androgen
production? A. Granulosa cells
B. Leydig cells
C. Organ of Zuckerkandl cells
D. Sertoli cells
E. Theca cells
The answer is Theca cells. LH drives the conversion of
cholesterol into androgens in the theca cells. These androgens
are then transferred to the granulosa cells for conversion
(aromatisation) into estrogen (estradiol) under the influence of
follicle-stimulating hormones.

Q)

Ans D

Q)

Q) The chance of malignant transformation


occurring in premenopausal benign ovarian cysts
is A. 0.1%
B. 3%

C. 10% ..ans
D. 15%
E. 25%
Q) Considering the mechanism of pain, the
following are true except : a. prostagladin
indirectly causes pain
b. bradykinin is a potent cause of pain
c. 5HT is a transmitter on inhibitory neurons
d. Adenosine has a role in regulating pain
e. Glutamate is an inhibitory amino
acid..ans
Q)The below pairing of anti-neoplastic drugs are
true except :
a. Alkylating agent - cyclophosphamide
b. Antimetabolite - 5- fluorouracil
c. platinum based paclitaxel.ans
d. Vinca alkaloids - vincristine
e. antibiotic - doxorubicin
example of platinum based are cisplastin and carboplatin.
paclitaxel is in the taxane group.
Q) Which of the foll opioids can be used as analgesia during
labour: a. pethidine
b. meptazinol
c. both of the above . ans
d. nitric oxide
e. all of the above

pethidine and meptazinol are opioids that can be used as


intrapartum analgesic.
nitric oxide is not an opioid. nevertheless, it can be used as
intrapartum analgesic.
Q) Which drug prevents peripheral deiodination of thyroxine?
Carbimazole
Iodine
Labetalol
Potassium perchlorate
Propylthiouracilans
The answer is Propylthiouracil. Although propylthiourcil, like
carbimazole, prevents the peroxidase oxidation of iodide to
iodine in the thyroid gland, it also has a peripheral action.
Q) In the developing foetal testis, the cells that produce
mullerian-inhibiting substance are the A. Testicular mesenchymal cells
B. Interstitial cells of leydig
C. Sertoli cells..ans
D. Spermatogonia
E. Primordial germ cells
Q) Risk of recurrence of postpartum psychosis in subsequent
pregnancy : A)10%
B) 15%
C) 25%..........ans
D) 35%
E) 40%

Q) When a drug is said to be adequate and wellcontrolled studies have failed to demonstrate a
risk to fetus in 1st trimester (and there is no e/o
risk in later trimesters). In which FDA category
does it fall? a. Aans
b. B
c. C
d. D
e. N

Q) What is an appropriate endocrine response to low, detected


serum calcium levels?

A. Decreased parathyroid syndrome (PTH), increased


phosphate excretion, increased calcitriol and increased
calcitonin
B. Decreased PTH, reduced phosphate excretion, increased
calcitriol and increased calcitonin
C. Increased PTH, increased phosphate excretion, increased
calcitriol and reduced calcitonin
D. Increased PTH phosphate excretion, reduced calcitriol and
reduced calcitonin
E. Increased PTH, reduced phosphate excretion, increased
calcitriol and reduced calcitonin
The answer is Increased PTH, increased phosphate excretion,
increased calcitriol and reduced calcitonin. Low calcium
levels are detected by the parathyroid glands and PTH is
secreted. PTH acts on the kidneys to increase calcium
absorption and also increase phosphate excretion. PTH also
increases the activity of 25-hydroxycholecalciferol, which
converts calcidiol to calcitriol.

Q)

Ans A
Q) A 27 y is treated for severe bronchitis at 38 w. Her baby,
born at 41 w , has neonatal hemolysis. Which drug taken by
mother for bronchitis is cause of baby's neonatal hemolysis?
a. amoxycillin
b. chloramphenicol
c. co-trimoxazoleans
d. doxycycline
e. erythromycin
Q) Which chromosome contains the gene that codes for the
alpha globin chain (a constituent component of Hb)?
A. Chromosome 5
B. Chromosome 11

C. Chromosome 16.ans
D. Chromosome 18
E. Chromosome 21
Q) Upper vagina supplied by - A)uterine arteryans
B)aorta
C)int iliac
D)ext iliac
The vagina receives its blood supply from the vaginal arteries
and their anastomoses with branches of the uterine, inferior
vesical and internal pudendal
Q) the risk of uterine rupture is 52 /10.000 in previous 1 C.s
and 92 /10.00 in previous 2 C.s
Q) Which of the following forms part of the phase 2 reaction
in drug metabolism? a. conjugationans
b. cyclisation
c. hydrolysis
d. reduction
e. oxidation

Q)

Ans D
Q) The most common endometrial ca is..?Serous
the mc endometrial CA is endometroid tupe 90% the serous
10-15%
Q)

Q)

Ans C
Q) Which of the following cell types lyses cells that have
been infected with viruses? A. CD4+ T cells
B. B cells
C. CD8+ T cells.ans
D. Killer cells
E. Macrophages
Q) What is the progestogen component in an implant?
a. desogestrel
b. levenogestrel
c. etonogestrelans
d. drosperinone
e. norethisterone
Q) With regard to electrosurgery, if carbon is seen on the tip
of an electrode the surgeon can assume that, at some stage in

the operation, what temperature has been reached? 44C


70C
90C
150C
200C.ans
About tip of electrode :
Necrosis occurs at 44C. Coagulation occurs at 70C.
Desiccation occurs at 90C. Carbonisation occurs at 200C.
Q) CMV:- sensoryneural hearing loss, delayed psycomotor
development of child, jaundice & hepatomegaly after birth
Rubella:- Sensory neural hearing loss, Blue Berry muffin rash,
cong heart defects (PDA, ASD), cong eye defect-cataract etc
main point :-look for rash

Q)

Ans D

Q) Most common cause of post menopausal bleeding??


1) endometrial hyperplasia
2) endometrial atrophyans
3) endometrial cancer

Q)

Ans C
Q) Regarding MgSO4, the foll statement are true
except :
a. it acts at motor end plate and reduces
excitability of nerve
b. it acts on cell membrane & reduces calcium
influx into cell
c. it is use to prevent eclampsia
d. oliguria is a sign of magnesium toxicity
ans
e. Cagluconate can be given if patient develops
cardioresp depression
Magnesium sulphate is a cell membrane
stabiliser. It prevents influx of calcium into the

cell.
mgso4 toxicity dont cause oliguria. It is the other
way round. Oliguria causes MgSO4 toxicity.
it is used to prevent eclampsia
yes, we monitor urine output. but oliguria not
necessarily mean toxicity. oliguria could
percipitate toxicity as renal is the only way to
excrete mg. we monitor UO to prevent toxicity,
not as a sign of toxicity.
Q) Pt had C-Section developed S/S of PE. Most
likely source of embolism is? 1) pelvic
veins..ans
2) femoral veins
3) inf vena cava
4) Rt atrium
Q)Wrt labetolol, foll statements are true except:
a. it can be given orally for patient with chr HTN
in pregnancy
b. it is an antagonist of A1 and B1 receptor
c. it is a partial antagonist of B2
receptor..ans
d. can be given intravenously in hypertensive
emergencies
e. contraindicated in patient with heart block
it is an absolute contraindication in pt with heart
block. labetolol is an antagonist at A1 and B1 but
a partial agonist of B2. So, the net effect is

reduce peripheral resistance + reduce heart


rate..
Q) Parrafollicular C cells is derived from?Neural
crest

Q) Which of the following is the most effective in


reducing heavy menstrual bleed? a. tranexemic
acid
b. COCP
c. LNG-IUSans
d. POP
e. danazol
Q) Which of foll would you expect to be raised in preg?
Albumin
ALT
ALP..ans

AST
Bilirubin
Q) Least affects bioavailability : 1) gastric emptying
2) liver impairment
3) protein binding
4) drug formulation.ans
Q)

Ans D
Q) Regarding methyldopa,foll statement is true except :
a. it is a prodrug
b. it is use to treat hypertension in pregnancy
c. it is centrally acting
d. it does not cross the placentaans
e. depression is its side effect

Q) Regarding transportation of drugs, foll are true except :


a. transcapillary movement is transfer of drugs with water due
to hydrostatic/ osmotic pressure
b. paracellular movement occurs between cell junction
c. passive transport is movement along a concentration
gradient
d. active transportation is movement against concentration
gradient and energy dependent
e. Facilitated diffusion is carrier mediated movement along
concentration gradient and energy dependentans
facilitated diffusion do not require energy. C is true. passive
diffusion do occur along concentration gradient. active
transportation occurs against concentration gradient

Q)

Q)Wrt pharmacology,which of foll statement is/are true :


a. prodrug is an inactive form of a particular drug
b. receptor is a cellular molecule to which a drug binds to
initiate its effect
c. half life is the time taken for the plasma concentration of
the drug to reduce by 50%
d. steady state concentration is when drug elimination equals
drug availability
e. all of the aboveans

Q)

Ans C

Q)What is ratio of testosterone bound to SHBG and albumin


respectively?
A.1% free,19% albumin-bound & 80% SHBG-boundans
B. 10% free, 30% albumin-bound and 60% SHBG-bound
C. 30% free, 19% albumin-bound and 51% SHBG-bound
D. 5% free, 1% albumin-bound and 94% SHBG-bound
E. 5% free, 30% albumin-bound and 55% SHBG-bound
:
The answer is 1% free, 19% albumin-bound and 80% SHBGbound. Only 1% is free but it is the albumin-bound fraction
that is hormonally active.

Q) Where in body is aldosterone synthesized?


A. Distal tubules
B. Lung
C. Zona fasciculata
D. Zona reticularis
E. Zone glomerulosa
The answer is Zona glomerulosa. Remember that
glucocorticoids are secreted by the zona fasciculate and
androgens are secreted by the zona reticularis.
N)GH is structurally similar to Prl and HPL.
Q) Regarding tamoxifen , all of the following are true except :
a. it is a selective estrogen receptor modulator
b. it is an estrogen antagonist on the breat
c. it is an estrogen agonist on the bone and endometrium
d. it can cause endometrial hyperplasia and adenocarcinoma
e. it is widely used as a bone sparing agent.ans

Q)

Ans D
Q) When using the MDRD formula to calculate GFR
(Glomerular filtration rate) which of the following is not
factored into the equation? Creatinine
Urea..ans
Sex
Age
Ethnic Background
Estimated GFR (ml/min/1.73m2) = 186 x (Creat /
88.4) -1.154 x (Age)-0.203 x (0.742 if female) x
(1.210 if black)
Q) Where in the kidney is the majority of Sodium and
Glucose reabsorbed -Bowmans Capsule
Loop of Henle

Proximal Tubule..ans
Distal Tuble
Collecting Duct
Remember PCT is the main site for Reabsorption
for all ions!
Q) An antenatal usg demonstrates a hyperechogenic bowel.
Which one of the following is this finding most associated
with? Omphalocele
Polyhydramnios
Multiple pregnancy
Gastrochisis
Cystic fibrosisans
Hyperechogenic bowel is seen in Down's syndrome, cystic
fibrosis, cytomegalo virus infection
Q) What is the typical O2 consumption in 75kg non-preg
women? 5ml/min
15ml/min
50ml/min
150ml/min
250ml/min..ans
The typical Oxygen Consumption (VO2) is 250ml/min.
In pregnancy this increases by around 20% to 300ml/min
Q) Which of foll is responsible for Aquaporin-2 protein
channel openings in collecting duct? Angiotensin
Angiotensin II

ADH..ans
Aldosterone
Renin

Q) What inhibits glucagon releases? A. Adrenaline


B. Exercise
C. Hypoglycaemia
D. Increased amino acids
E. Increased free fatty acids .. ans
The answer is increased free fatty acids. Glucagon is secreted
by the alpha cells of the pancreas. It increases hepatic
glycogenolysis and also glyconeogenesis. Adrenaline
stimulates glucagon release in times of stress .This stimulates
liver glycogenolysis to ensure maximum glucose output to

prepare for flight or fight' reaction. Hyperglycaemia inhibits


glucagon release.
Q) Aldosterone promotes water retention via which of foll
mechs?
Up-regulation of Aquaporin protein channels in collecting
duct
Up-regulation of urea transport proteins in collecting duct
Down-regulation of urea transport channels in the collecting
duct
Up-regulation of Na/K pumps in the distal tubule and
CD.ans
Stimulation of Potassium resorption in the tubular lumen
Q) What % age of Hb is HbF by 6 months of age? <2%......ans
5-10%
20-25%
50%
70-90%
Q) Regarding blood vol in preg which of foll statements is
TRUE?
Blood volume remains constant
Blood Volume increases by approximately 5%
Blood Volume increases by approximately 10-15%
Blood Volume increases by approximately 20-25%
Blood Volume slowly increases by 40-50%........ans

Q)

Ans C
Q) Progestron have higher affinity to bind to : A.
SHBG
B. CBGans
Progesterone: 2% unbound, 80% bound to albumin, 18%
bound to cortisol binding globulin and < 1% bound to SHBG
Q) Dilatation of the uterine cervix at parturition is primarily
achieved by:
Muscle relaxation under the influence of oestriol
Prostaglandin E1 mediated Interleukin release
Constriction of cervical small vessels
Macrophage mediated collagen degradation
Degradation of type I collagen by interstitial
collagenaseans
Q) Minute ventilation in pregnancy increases due to which of
the following?
Effects of gravid uterus on diaphragm
Dliutional anaemia
Increased renal excretion of bicarbonate

Increased circulating oestragen


Increased circulating progesteroneans
Q) A 24 years old girl who is just recently married and not
emotionally prepared to conceive a child. In order to prevent
pregnancy, she is taking mercilon as a contraceptive method.
She came to you today because she had missed a pill for 24
hours. What would your advice be?
a. discard this pack and start a new pack
b. take the missed pill immediately and continue her current
pack with one week pill free interval..ans
c.take the missed pill immediately and continue her current
pack with no pill free interval
d. take the missed pill immediately and continue her current
pack with 7 days additional contraception
e. continue this pack with no pill free interval
Q) In various conversion of lactose to
glucose,lactose initially goes hydrolysis to which
substrate
A)sucrose
B)galactose..ans
C)fructose
D)glycerol
E)acetone
Q) A healthy 23yrs old woman wishes to discuss
contraception
After a vaginal delivery at term she is breast feeding and
wishes to use progestogen only
Pills ?

A start immediately additional contraception for 5 days


B start 21 days after delivery
C start after 6 weeks
D start 6 months after delivery
E start 21 d after delivery with additional contraception for 7
d..ans.
Q) Incidence of shoulder dystocia - A.1%
B.0.5%
C.0.6%............ans
D.0.7%
Recurrence 15 %
Q) An 18-y who had undergone a prev c/s, is admitted for
active labor. During labor, an intrauterine pressure catheter
displays normal uterine contractions every 3 minutes with
intensity up to 60 mm Hg. Fetal bradycardia ensues. Which of
foll statements is most accurate?
A. The normal IUPC display makes uterine rupture unlikely.
B. Mc sign of uterine rupture is a FHR abnormality ..ans
C. If the patient has a uterine rupture, the practitioner should
wait to see whether the heart tones return to decide on route of
delivery.
D. The IUPC has been found to be helpful in preventing
uterine rupture.
Q) A 22-year-old woman is being seen at 11 weeks gestation
for an ultra- sound examination. Besides the nuchal
translucency, which other sonographic finding is associated
with fetal aneuploidy?A. Fetal biparietal diameter

B. Fetal cerebellar diameter


C. Fetal nasal bone..ans
D. Fetal crown rump length
Q) Ovarian precursors of oestradiol include : a. Oestrone
b. Androstenedione
c. Testosterone
d. All of the above.ans
e. None of the above
Q) The Arius-Stella reaction may be seen with all except :a.
Ectopic pregnancy
b. Birth control pillsans
c. Abortion
d. Trophoblastic disease
Q)Passage of decidual cast in cases of ectopic usually means :
a. Impending tubal rupture
b. Reabsorption of embryo
c. Pregnancy was intrauterine
d. Death of embryo.ans

Q)

Q)Mc SE with MTX therapy for ectopic a. Transient pelvic pain 3-7 d after starting Rx
b. Stomatitis
c. Bone marrow suppression
d. Gastritis
Q) Prenatal diagnosis at 16 w can be performed
using all of the foll, except: a. Amniotic fluid
b. Maternal blood
c. Chorionic villi
d. Fetal blood..ans

Q)

Ans B

Q) A 29 y G1 at 8 w with triplet gestation presents for


prenatal care. She is sure she does not desire to have invasive
testing as a first step but desires to have some information
regarding her risk for fetal DS. What option do you
recommend?
A.

Integrated screen
B. Sequential screen
C. Quad screen..ans
D. Nuchal translucency only
Q)

Q)

Q) 33 year old women presents to clinic for infertility


investigations. She has a significant psychiatric history. Her
blood tests reveal a raised Prolactin. Which of her medications
below is NOT known to cause this - A. Amitriptyline
B. Zopiclone..ans
C. Chlorpromazine
D. Risperidone
E. Cocodamol
Q) 72-year-old woman undergoes a total abdominal
hysterectomy. She has chronic obstructive pulmonary disease.
Postoperatively, she is difficult to extubate and has a
prolonged stay on the intensive care unit.Which of the
following is the most important direct stimulus to respiration?
A. Decreased arterial pH
B. Decreased arterial p02
C. Decreased arterial pCO2
D. Increased H concentration of CSFans
E. Increased pCO2 of the CSF
Q) Half Life of heparin - A)1 hour ans
B) 2 hrs
C)1 day
D) 3 hrs
E)1 week

Q) Regarding gametogenesis, what is the end product of one


germ cell at the end of meiosis 2 in male and female?
a. male : 4 spermatozoa, female : 4 ovum
b. male : 2 spermatozoa+2polar bodies , female : 2 ovum+2
polar bodies
c. male : 4 spermatozoa , female : 1 ovum+ 3 polar
bodies..ans
d. male : 1 spermatozoa + 3 polar bodies, female : 4 ovum
e. male : 4 spermatozoa , female : 2 ovum+2 polar bodies

Q) Chromaffin cells produce which of the

following hormones?
A. Dopamine.ans
B. Corticotropin-releasing hormone
C. Somatostatin
D. Prolactin
E. Vasopressin
Q) Which of the following is incorrectly matched?
a. trisomy 21 - decreased alpha fetoprotein
b. trisomy 18 - increased NT
c. trisomy 13 - mainly by meiotic non dysjunction
d. Klinefelter - increased NT ..ans
e. Klinefelter - infertility
Q) Which of the following statements regarding
thyroid function testing in pregnancy is true?
A. TSH levels increase in pregnancy
B. TSH levels remain constant through the
trimesters
C. Total T3 levels drop in pregnancy
D. Free T3 levels drop during
pregnancy.ans
E. Thyroid hormone binding globulin levels drop
during pregnancy
Q) Anti hypertensive that can cause neonatal
hypoglycemia
A)methyl dopa
B)hydralazine
C)labetalol.ans

D)captopril
E)none above

Q)

Q)

Ans B
Q)

Ans E
Q) Drug which bind to alpha 1 glycoprotein - A)salicylate
B)warfarin
C)ansaids
D)beta blocker..ans
E)all above
Q) At what week in pregnancy is testing for gestational
diabetes (GD) advised
A. 24-28 weeks of pregnancy if past history of GD
B. 24-28 weeks of pregnancy if no history of GD
C. 16-18 weeks of pregnancy if no history of GD
D. 16-18 weeks of pregnancy if past history of GDans
E. At booking appointment regardless of past history

Q) Which of the following women would you advise to take


400mcg of folic acid during the first 12 weeks of pregnancy?
A)25 year old type 1 diabetic
B)26 year old recently diagnosed with coeliac disease
C)31 year old taking sodium valproate
D)24 year old with sickle cell anaemia
E)None of the above..ans
Q) Regarding PE in Pregnancy which of foll statements is
true?
PE is the most common cause of maternal death in the UK
PE accounts for approximately 25% of maternal deaths in the
UK
PE mortality is 0.79/100,000 pregnancies in the UK ....ans
Being overweight (BMI 25-30) does not increase VTE risk
Age >30 is a positive risk factor for VTE when considering
thromboprophylaxis
Q) With respect to pre-natal diagnosis
A. Screening programs for Downs syndrome do not need to
include karyotyping.ans
B. The results of rapid pre-natal diagnosis tests are usually
available in 7-10 days
C. Karyotype is essential for an effective pre-natal diagnosis
programme
D. The results of rapid pre-natal diagnosis tests are usually
available in 2-3 weeks

E. The results of rapid pre-natal diagnosis tests are usually


available within 24h
Q) Regarding health risks associated with PCOS which of the
following statements relating to PCOS patients are true (as
advised by the RCOG)?
A. It is good practice to recommend treatment with gestogens
to induce a withdrawal bleed at least every 3 to 4
months.ans
B. Due to the increased risk of ovarian cancer patients with
PCOS
require additional surveillance.
C. Due to the increased risk of breast cancer patients with
PCOS
require additional surveillance.
D. Insulin-sensitising agents are licensed in the UK for use in
patients without diabetes
E. Bariatric surgery may be an option for morbidly obese
women with PCOS (BMI of 35 kg/m2 or more or 30 kg/m2 or
more with a high-risk obesity-related condition)
Q) A 31 year old patient undergoes an elective c-section
delivery. You estimate blood loss has reached 1000ml and you
suspect uterine atony is the likely cause. Following bimanual
uterine compression what pharmacological intervention is
advised?
A)Ergometrine 0.5mg by intramuscular injection
B)Carboprost 0.25 mg by IM injection

C)Syntocinon 5u by slow intravenous injection..ans


D)Direct intramyometrial injection of carboprost 0.5 mg
D)Misoprostol 1000 micrograms rectally
Oxytocic Agents Key Points
Prophylactic oxytocics should be offered routinely
in the management of the third stage of labour in
all women as they reduce the risk of PPH by
about 60%.
Oxytocin
1st choice for prophylaxis PPH in the third stage
of labour.
1st choice if delivering by caesarean section
Syntometrine
May be used in the absence of hypertension
(for instance, antenatal low haemoglobin) as it
reduces the risk of minor PPH (500-1000 ml)
Compared to Oxytocin 5 fold increase risk of
side effects inc nausea, vomiting and raised BP
Q) A patient attends clinic with a vaginal prolapse. On
examination the vaginal prolapse is visible 1.5cm above the
plane of the hymen. According to the POPQ classification
what grade is this prolapse? Grade 0
Grade 1.ans
Grade 2

Grade 3
Grade 4
Q) Regarding the pubic symphysis, what type of joint is it
Fibrous
Condyloid
Synovial
Primary Cartilaginous
Secondary Cartilaginousans
Q) Which of foll contraceptives primary MOA is inhibition of
ovulation?A)Norgeston
B)Cerazetteans
C)Micronor
D)Femulen
Traditional POP main mode of contraceptive
action: thickening of cervical mucus
Desogestrel-only POP main mode of
contraceptive action is inhibition of ovulation
Cerazette is the only Desogestrel-only POP in
the options above. Other desogestrel brands
include:
Aizea
Cerelle
Nacrez
Q) Which of the following is with regard to
perinatal mortality in the United Kingdom?

A. It is approximately 7 per 10000 births


B. It is associated with low birth weight (less than
2.5 kg) babies in over 60% of cases
C. It is defined as all stillbirths and all deaths in
the first 28 days after birth
D. It is higher in boys .ans
E. It is lower in babies of mothers who are
primiparous
Q) An 18 year old patient comes to see you in clinic. Her BMI
is 25.0 and her BP is 122/80. She is a non- smoker and there is
no personal or family history of VTE or migraine. She would
like to start the pill for her acne. She has used topical Zineryt
in the past but still has moderate acne. What is the most
appropriate option?
A. Cerazette (Desogestrel)
B. Marvelon (Ethinylestradiol/Desogestrel)ans
C. Dianette (Co-cyprindiol)
D. Norimin (Ethinylestradiol/Norethisterone)
E. Yasmin (ethinylestradiol / drosperinone)

Q)

Q) How many lobules are in each testis? 25


250..ans
25,000
250,000
2.5 million
Ans 200-300.... so 250

Q)

Q)

Q)DIC - PT ,APTT Inc, low platelets,low fibrinogen


and high fibrin degradation products
Q) Which of the following statements regarding the Vaginal
artery is typically TRUE?
It arises from the Internal iliac artery..ans
It arises from the External iliac artery
It arises from the Abdominal Aorta
It arises from the Ovarian artery
It arises from the Uterine artery
The Vaginal artery, like the Uterine artery is
typically a branch of the Internal Iliac artery. It
can sometimes arise as a branch of the Uterine

artery so it is important to read the question (if


the stem stated it CAN arise from the Uterine
artery then that would be true)

Q) Where are the proximal and distal centriole located in a


spermatozoa?
Head
Neckans
Middle Piece
Tail
End Piece
Q) What is the first immunoglobulin to be
synthesised by the neonate? IgA
IgG
IgM . ans
IgD
IgE
Q) Plasma typically accounts for what percentage of body
weight? 4%.......ans
14%
20%
40%
80%
Q) Which one is not a side effect of warfarin
A)purpura fulminans
B)purple toe syndrome
C)embryopathy
D)skin necrosis
E)Steven John sons synd.ans
Q) You are asked to review the serology results of a 27 year
old women. She was noted to have had deranged liver

function tests. The results are as follows:


Marker
Result
HBsAg
Negative
Anti HBs
Positive
Anti HBc
Negative
IgM Anti HBc
Negative
What does this indicate regarding her hepatitis B status?
Acute Infection
Chronic Infection
Immune due to past infection
Immune due to vaccination.ans
Susceptible
This patient has Anti HBs antibodies - this suggests immunity.
Anti HBc persists for life after infection so is a marker of past
infection. In this case anti HBc is negative suggesting
immunity by vaccination rather than past infection. Hepatitis
B Marker Description Interpretation HBsAg Hepatitis B
surface Antigen Indicates current infection either acute or
chronic Anti HBs Hepatitis B surface Antibody Indicates
immunity either due to infection or vaccination Anti HBc
Hepatitis B core Antibody Indicates either current or past
infection IgM Anti HBc IgM antibody to Hepatitis B core
Indicates recent infection
Q) You are asked to review the full blood count of a 61 year
old patient in pre-op clinic. The results are as below: Hb 9.0
g/l
MCV 118 fl
Platelets 155 * 10 /l

WBC 9.4 * 10 /l
You note her past medical history records a diagnosis of
vitiligo 4 years ago but nil else.
She takes no regular medications. You note routine bloods
done by the GP 1 month earlier
show normal thyroid function, urea and electrolytes and
HBA1C. What is the likely
diagnosis? Iron deficiency anaemia
Gastrointestinal bleed
Endometrial bleed
Hashimotos
Pernicious anaemia.ans
Q) Hyponatraemia is a recognised complication of which of
the following?
(Please select 1 option) A) Carbenoxolone therapy
B) Cerebral contusion ans
C) Diabetes insipidus
D) Polyuric phase of acute renal failure
E) Major burns
Q) What is the additional risk of miscarriage if amniocentesis
is performed for genetic screening? a. 1%.......ans
b. 5%
c. 10%
d. 20%
e. 40%

Q) When consenting a patient for TAH(for a benign


condition) what would you advise regarding the risk?
A)Haemorrhage requiring transfusion approximately 1%
B)Injury to Ureter or bladder approximately 1%.............ans
c)Overall risk of serious complications 1%
D)Pelvic abscess approximately 1%
E)Injury to Bowel 1%
The RCOG states injury to bladder and/or ureter at 0.7%. This
is for abdominal hysterectomy for benign conditions. Note
abdominal hysterectomy carries higher risks when performed
for non-benign conditions and overall risk of ureter injury is
1%. Abdominal Hysterectomy According to the RCOG
consent advice for abdominal hysterectomy for benign
conditions. The following risks are quoted: Overall Risk
serious complication 4% Haemorrhage requiring blood
transfusion 2.3% Bladder and/or ureter injury and/or longterm disturbance of bladder function 0.7% Return to theatre
(e.g. because of bleeding/ wound dehiscence etc) 0.7% VTE
0.4% Pelvic abscess/infection: 0.2% Bowel injury: 0.04% (4
in 10 000) Risk of death within 6 weeks, 0.03%
Q) The juxtaglomerular apparatus (JGA) lies within which
part of the kidney? Renal Cortex.ans
Renal Medulla
Minor Calyces
Major Calyces
Renal Pelvis

Q) The superficial inguinal ring is an aperture in which


structure? Pubic tubercle
Aponeurosis external oblique..ans
Aponeurosis internal oblique
Conjoint tendon
Transversus abdominus
Q) In regards to varicella zoster immunization, which of the
following is correct?
a. vaccination can be given during pregnancy
b. in seronegative patient, vaccination can be given
postnatally and mother can safely breastfeedans
c. in seronegative patient, vaccination can be given postnatally
and breast feeding is contraindicated

d. vaccination should not be given during prepregnancy


e. pregnant mother with Ig G positive was exposed to a
community with chicken pox. she should be given IVIG.
-mothers with Ig G positive do not require IVIG as they are
immune to VZV
- latest recommendation by RCOG is to offer seronegative
mothers, vaccination during prepregnancy or post natally. and
it is save to breast feed
Q) What testicular cell type secretes testosterone? Myoid
Intersitial macrophages
Sertoli
Leydigans
Germ

Q) Male infertility in a patient with cystic fibrosis is likely to


be due to which condition?

Congenital absence of testes


Congenital absence of vas deferens..ans
Hypothalamic failure
Oligospermia
Testicular failure
Q) 1st meiotic division occurs in - Spertogonium
Prim spertocytes .ans
Sec spermstovytes
Spermatids
Q) Which phase is best to visualise chromosome? a. prophase
b. metaphase.ans
c. G2 phase
d. G1 phase
e. S phase

Q)

Q) epidural anesthesia is given at? L 3/4

Q) The testis receive innervation from which spinal segment T10.ans


T12

S1
S2
S3
Q) Regarding Acute PID, which of foll is TRUE?
a. patient presents with superficial dyspareunia
b. it is strongly recommended to remove insitu-IUD once
diagnosed of having PID
c. the absence of infection on high vaginal swab excludes PID
d. Absence of pus cell give a good -ve predictive value .ans
e. the presence of pus cell gives a good positive predictive
value
Q) A patient was diagnosed of having PID and was treated
with appropriate ABic. What is your further plan?
a. see back in 8 wks to assess adequate response to Rx
b. see in 4 weeks to repeat testing for growth in all patient
c. see back in 6 weeks to ensure compliance of antibiotic
d. see back in 2 w to screen and treat sexual partner ans
e. discharge
A,C and D need to be done during follow up...but within 2 to
4 weeks.

Q) Nv supply to perineum

Q) A pregnant patient who is needle phobic has her nuchal


translucency (NT) scan but refuses serum markers. You advise

her the False Positive Rate of the scan is 5%. What would you
advise the mother regarding the detection rate of DS using NT
alone? 40%
50%
70%...........ans
90%
95%

Q) Levator ani p body insertn

Q) Regarding Turner syndrome which of the


following statements is true?
A)Complete monosomy is rare accounting for
<10% of cases of Turners

B)Turner syndrome occurs in approximately 1 in


every 20,000 live female births
C)Only 1% of affected fetuses will survive to
term.ans
D)Turners is thought to affect 0.1% of all
conceptuses
E)Long fingers are a recognised clinical feature
Turners may be complete monosomy of the sex
chromosomes where the karyotype is termed 45
X or show a mosaic pattern with variable
penetration of cell types with the single X
chromosome. Complete monosomy accounts for
50% of cases. Turners syndrome is common in
utero affecting 1-2% of all conceptuses however
99% of these will miscarry and only 1% will
survive to term. Turners occurs in 1 in 2000 live
births. Short stature, dysmorphism including
short fingers, pectus excavatum and webbed
neck are features. As are cardiac abnormalities
(coarctation aorta, bicuspid aortic valve, aortic
aneurysms and more), urogenital abnormalities
(horseshoes kidneys, double collecting system
and more) and behavioural problems.

Q)

Ans C
Q) A patient arrives on LBW she is 37 w . Her last preg ended
with delivery via uncomplicated LSCS 4 y ago. Contractions
are 5 mins apart and on examn and Cx is 5cm. She wants to
know the risk to baby of proceeding with VBAC. What is
additional risk the baby will have resp problems after VBAC
compared to elective repeat C-section (ERCS)?
A. No difference
B. Risk 1 to 2% greater with VBAC
C. Risk 1 to 2% greater with ERCSans
D. Risk 4 to 5% greater with VBAC
E. Risk 4 to 5% greater with ERCS

Q) A 25-y presents with a symmetrical arthropathy affecting


her hands. On examn she has synovitis of 2nd & 3rd MCP jts.
What type of HLA allele is most assoc with this condition?

A. HLA DR3
B. HLA A3
C. HLA DR4..ans rheumatoid arthritis
D. HLA DR2
E. HLA B27
Q)Fontanelle closure - Ant by 18 months
& post by 3 months
Q) Bal is anti apoptosis
P53 cause apoptosis
Q) What cells in the spermatogenesis process can
undergo mitotic division? Primary spermatocytes
Secondary spermatocytes
Spermatids
Spermatogonia.ans
Spermatozoa
Q) antigen presenting cell then langerhan
and if it is phagocytic antigen presenting then
neutrophil
Q) The rectus sheath is formed by which of the following?
External oblique and rectus abdominus aponeuroses
Internal oblique and rectus abdominus aponeuroses
Rectus abdominus and pyramidalis aponeuroses
aponeuroses of internal and external oblique
aponeuroses of transversus abdominis, external and internal
oblique.ans

Q) The round ligament leaves the pelvis via what? Greater


sciatic notch
Lesser sciatic notch
Superficial inguinal ring
Deep inguinal ring..ans
Lesser sciatic notch

Q) After surgery the patient is having difficulty in


adduction of thigh...which nerve involved.?
Obturator
Q) A 40-year-old woman presents at 14 weeks of gestation.
She opts to have antenatal screening. Her blood tests show an
increase in -hCG and low PAPP-A. At her dating scan there
is a raised nuchal translucency noted. What is the most likely
explanation for these results? Down syndrome
Edwards syndrome..ans
Normal pregnancy
Patau's syndrome
Twin pregnancy

Q) At what stage in the cell cycle is mitosis arrested if there is


a chromosomal abnormality? G2
Q) You have been asked to perform a pudendal nerve block on
a patient by your consultant. The pudendal nerve is formed
from which spinal segments? S1 and S2
L3,L4,L5 and S1
L5,S1 and S2
S2,S3 and S4.ans
S3,S4 and S5

Q) The post scrotal artery is a branch of which artery? Internal


Pudendal.....ans
External Pudendal
Inferior Rectal
Ilioinguinal
Superior rectal

Q) The metabolic response to trauma causes a


decrease in which of the following? A. Antidiuretic hormone
B. Glucagon secretion
C. Growth hormone
D. Insulin secretion ans
E. Urine osmolality
Q) With regard to the cell cycle. In which part of
the cycle do Chromatids form?A. G0
B. G1
C. G2
D. Sans
E. M

Q)

Ans D

Q) Letrozole - MOA

Q) You are called to see a women after a prolonged labour


with failed instrumental delivery converted to c-section. She
is unable to dorsiflex her right foot and complains of pins and
needles to the foot and lower leg. What is the likely
diagnosis?
Right L5 nerve root compression
Right S1 nerve root compression
Right Saphenous nerve root compression
Right common peroneal nerve root compression.ans
Right superficial peroneal nerve root compression

Q) In the third trimester of pregnancy what is the daily


calcium requirement of the fetus?
50 mg
100 mg
200 mg - This is the correct answer
500 mg
800 mg Q) The ureters receive autonomic supply from which spinal
segments? T10-12
T11-L2.ans
L1-L3
L2-L5
S1-S3
Q) Regarding the rectus sheath which of the following
statements are true?
Above the arcuate line the internal oblique divides into two
lamellae..ans
Below the arcuate line the internal oblique divides into two
lamellae
Below the arcuate line the external oblique divides into two
lamellae
Above the arcuate line the external oblique divides into two
lamellae
Below the arcuate line the transversus divides into two to
encompass the rectus abdominus
Q) Following a water birth, a woman elects not to have
oxytocics for the
management of the third stage of labour. Thirty minutes later,

she is brought to the consultant unit with a postpartum


haemorrhage owing to an atonic uterus. If she had received
standard oxytocic management for the third stage of labour,
by what amount would she have reduced her risk of a
postpartum haemorrhage? A. 10%
B. 20%
C. 30%
D. 60%............ans
E. 90%
N) At the proximal end of the urethra (prostatic urethra) there
is transitional cell epithelium in continuation with the bladder.
The epithelium changes to stratified columnar and then to
stratified squamous near the urethral orifice.
Q) Implantation of the embryo occurs at which stage?
Blastocyst.ans
Morula
Pronuclear phase
Trophoblast
Zygote
Correct
The answer is Blastocyst. Within 24 hours of fertilisation, the
zygote undergoes cleavage. It then further subdivides into
blastomeres. By the third day, the embryo contains 12 cells.
By the fourth day, it is comprised of 16-32 cells and is called a
morula. On day 5, the blastocyst hatches from the zona
pellucida and implants into the endometrium. For in vitro
fertilisation, transfer of the embryo with blastocyst has been

shown to be more effective and is recommended with a single


embryo transfer.
Q) In earliest phase of wound healing platelets
held together by A)fibroblasts
B)fibrin..ans
C)PGE 1
D)type 1 collagen
E)type 2 collagen
Q) The average functioning adult kidney contains approxly
how many nephrons? 100
1,000
10,000
100,000
1,000,000.ans
Q)

Ans C
Q) Which two nerves provide the primary cutaneous sensory
innervation to the labia majora? Ilioinguinal and inferior rectal
Ilioinguinal and pudendal..ans
Pudendal and perineal

Anterior femoral and genitofemoral


Pudendal and iliohypogastric

Q) Fetal sex differentiation at week 9..ans


10
11
12
Q) A baby with shoulder dystocia suffers a brachial plexus
injury. The mother asks you if this will be permanent. What
percentage of babies will have permanent neurological
dysfunction as a result of brachial plexus injury secondary to
shoulder dystocia?<10%..............ans

15%
25%
35%
50%
Q) Which complement protein recruits other
complement proteins to form MAC (membrane
attack complex)? C3a
C3b
C3
C5bans
C
Q) In females the pudendal nerve branches are?
Inferior rectal, superior rectal and ilioinguinal
Perineal, ilioinguinal and dorsal clitoral
Inferior rectal, ilioinguinal and perineal
Perineal, inferior rectal and dorsal nerve of clitoris .ans
Perineal, superior rectal and posterior nerve of clitoris
Q) Which of foll muscles does NOT receive innervation from
pudendal nerve? Internal anal sphincter..ans
External anal sphincter
External urethral sphincter
Bulbospongiosus
Levator ani
Q) What is the average oblique diameter of the pelvic inlet
according to the RCOG? 7.5 cm
9 cm
10.5 cm

12 cm..ans
13.5 cm
Q) A 26 years old, primigravida who has had a previous
exposure to VZV outbreak when she was 12 weeks of
gestation. At that time, she was warded and IVIG was
administered. Currently, she is 18 weeks of gestation and had
another exposure to VZV. What would be your management?
a. counselling and reassurance
b. admit for observation
c. put her under isolation
d. give her 2nd dose of IVIG.ans
e. start her on acyclovir
Q) Which of foll muscles is NOT a constituent of pelvic floor
(diaphragm)?iliococcygeal
Piriformis..ans
Puborectalis
Pubococcygeus
Coccygeus
Q) The lumbar plexus is derived from which spinal segments?
T10-L5
L2-L4
L1-L3
T12-L4..ans
L1-L5

Q)In male urethra where do ducts of bulbourethral (Cowper's)


glands enter? Posterior wall prostatic urethra
lateral walls prostatic urethra
Membranous urethra.ans
Spongy urethra
Cavernous urethra
Q) Capsule of developing Griffian follicles ? A
theca interna
B theca externaans
C Zona pellucida
D Zona granulesa
E lamina propria
Q) Bladder neck closure and relaxation of the bladder is
mediated by? Sympathetic Fibres L1,L2ans

Parasympathetic Fibres S2, S3,S4


Sympathetic Fibres S2,S3,S4
Parasympathetic Fibres T11,L1,L2
Sympathetic Fibres L3,L4,L5
Q) A 25-year-old diabetic woman has been
morbidly obese for past 5 y.In this patient, which
one of foll hormones would decrease the appetite
as levels increase? A. Leptinans
B. Thyroxine
C. Ghrelin
D. Adiponectin
E. Insulin
Q) Sertoli cells contain receptors to which hormone? Inhibin
Oestradiol
Testosterone
LH
FSHans
Q) Innervation of vagina upper2/3 by inf.hypogastric plexus
and lower1/3 by pudendal nerve..T
Q) What is the average AP distance of the female pelvic
outlet? 7.5 cm
9 cm
10.5 cm
11.5 cm
13 cmans

Q) Regarding lymph drainage of Cx where does the majority


of lymph drain to? Internal iliac nodes
External iliac nodes..ans
Para-aortic nodes
Lateral aortic nodes
Ingiunal nodes
Q)

Q)

Q) According to the 1967 Abortion Act, the foll


are all indications for TOP if pregnancy were to
continue EXCEPT:
a The woman's life is likely to be endangered
b The womens physical health is likely to be
endangered
c The womens mental health is likely to be
endangered
d The partners physical health is likely to be
endangered

e The child is likely to suffer a physical


handicap..ans
Q)In male urethra where do ducts of bulbourethral (Cowper's)
glands enter? Post wall prostatic urethra
lateral walls prostatic urethra
Membranous urethraans
Spongy urethra
Cavernous urethra
Q) Which one of foll options best describes TGs concns in
preg? (Please select 1 option)
A) The increase in triglycerides results from decreased hepatic
lipase activity and decreased lipoprotein lipase activity
B) The increase in triglycerides results from decreased hepatic
lipase activity and increased lipoprotein lipase activity
C) The inc in triglycerides results from inc hepatic lipase
activity and dec lipoprotein lipase activity..ans
D) The increase in triglycerides results from increased hepatic
lipase activity and increased lipoprotein lipase activity
E) There is no change in triglyceride concentrations
Serum total cholesterol and triglyceride
concentrations increase markedly during
pregnancy.
The large increase in triglycerides appears to be
due to two factors:
Increased hepatic lipase activity resulting in
enhanced hepatic triglyceride production and

Reduced lipoprotein lipase activity leading to


reduced catabolism of adipose tissue.

Q) Which nerve arises near ant sup iliac spine


below inguinal ligament?

Q)Hysteroscopic classication of submucosal


fibroids is:
0= totally in
endometrial cavity,
1= more than 50% protrude in endometrial
cavity and
2= less than 50% protrude in endometrial cavity

Q) Hypothal locatn diencephalon

Q)

Q) urea absorption through placenta? Passive diff


Q) Chromosomes are detected in metaphase by Geimsa
stainingT
Q) What is average oblique diameter of pelvic inlet according
to RCOG? 7.5 cm
9 cm
10.5 cm
12 cm
13.5 cm

Q)

Q) Which one is purine - Cytosine


Guanine..ans
Thymine
Uracil
Q) In the case of hypocalcaemia, what ECG
changes typically occur? The presence of Q
waves
ST elevation
ST depression

Short QT interval
Long QT intervalans
Q) You have been asked to review an asymptomatic patient in
the early pregnancy unit. Ultrasound scan is negative for
pregnancy. hCG levels over 48 hours are 550 and 350, and
serum progesterone levels are 17 nmol. What is the most
likely finding? A. Ectopic pregnancy
B. Failing pregnancy
C. High risk of ectopic pregnancy needing intervention for
treatment
D. Non-viable pregnancy with the possibility of spontaneous
resolution..ans
E. Viable pregnancy
Q) Which Ar is a direct branch of aorta? A. Inferior vesical
B. Internal iliac
C. Ovarian ..ans
D. Uterian
E. Vaginal
The answer is ovarian. The ovarian artery is a
branch of the aorta. It arises anterolaterally just
below the renal artery, running retroperitoneally
to leave the abdomen by crossing the common or
external iliac artery in the infundibulopelvic fold.
It crosses corresponding ureters and supplies
twigs to it but does not supply to abdominal
organs. The internal iliac artery arises from the

common iliac and its inferior branch further


supplies to the pelvis.
Q) A 65-y underwent vaginal surgery for which she was in the
lithotomy position for 2 hours. Postoperatively you have
noticed parasthesia of the lateral side of the leg, foot and foot
drop. The compression of which nerve is likely to be
responsible? A. Common peroneal nerve
B. Femoral nerve
C. Great saphenous nerve
D. Obturator nerve
E. Pudendal nerve
The answer is common peroneal nerve. The
injury is thought to be secondary to compression
of the nerve between the lateral head of the
fibula and the bar holding the legs. When the
stirrups are used, special attention must be paid
in order to avoid compression. Injury to the
femoral nerve will cause hypoesthesia in the
anterior and anteromedial area on the thigh
below the inguinal ligament.
Q) Ribosomes are located in which part of cell?
A. Golgi complex
B. Lysosomes
C. Mitochondria
D. Rough endoplasmic reticulum .ans
E. Smooth endoplasmic reticulum

Q)Which part of cell cycle is noted for sister chromatids


separating and moving to oppo sides of cell?
A. Anaphase .ans
B. Metaphase
C. Prophase
D. Synthesis phase
E. Telophase

Q) Regarding Dimethyl Sulfoxide which is true???


Is administered directly into bladderans
Is a nor adrenaline reuptake inhibitor
Is a 5HT reuptake Inhibitor
Has anticholinergic effects on bladder
Results in symptomatic improvement in stress
incontinence
Q) An aromatic amino acid decarboxylase
Inhibitor - Enapril
Methyldopa.ans
Captopril
Hydralazine
Labetolol

Q)

Ans E
Q) A 62 years old female presented to the emergency
department with a right neck of femur fracture. No history of
trauma. You suspected her of having vitamin D deficiency.
What blood investigation would you request to support your
diagnosis?a. serum calcitriol
b. serum calcidiol..ans
c. 1 alpha hydroxylase enzyme
d. 25 hydroxylase enzyme
e. serum calcium and phosphate level
it is because calcidiol is the major circulatory
form for vitamin D, thus the surrogate marker for
Vitamin D status.
Q) Which of the following is responsible for Aquaporin-2
protein channel openings in the collecting duct? Angiotensin
Angiotensin II

ADH.ans
Aldosterone
Renin
Q) Aldosterone promotes water retention via which of the
following mechanisms?
Up-regulation of Aquaporin protein channels in the collecting
duct
Up-regulation of urea transport proteins in the collecting duct
Down-regulation of urea transport channels in the collecting
duct
Up-regulation of sodium/potassium pumps in distal tubule and
collecting duct.ans
Stimulation of Potassium resorption in the tubular lumen
Q) Breast milk is produced by the secretions of the epithelial
cells of the acinar. Which of the following is an accurate
estimate of mature breast milk composition?
Fat 4%, Protein 1%, Sugar 7%............ans
Fat 10% Protein 4% Sugar 20%
Fat 10% Protein 10% Sugar 30%
Fat 18% Protein 5% Sugar 5%
Fat 18% Protein 15% Sugar 1%

Q) Which structure is the primary mechanism for shunting


blood away from the fetal pulmonary circulation? Foramen
Ovale.ans
Ductus Arteriosis
Arteria Umbilicalis

Ductus Venosus
Umbilical Arteries
Blood enters the right atrium of the fetal heart and most
passes through the foramen ovale into the left atrium. From
there it is pumped through the aorta. The foramen ovale is the
major structure for bypassing the fetal pulmonary circulation.
Some of the blood in the right atrium does enters the right
ventricle and then into the pulmonary artery however most of
this passes through the ductus arteriosus into the aorta thus
bypassing the fetal pulmonary circulation.
Q) Which one of the following structures is the origin of the
renal tubules?(Please select 1 option) A) Genital tubercle
B) Mesonephric duct
C) Metanephric blastema..ans
D) Paramesonephric duct
E) Urethral folds

Q) Which of the following options describes lymph group to


which the vulva drains?A Deep inguinal lymph nodes
B Internal iliac lymph nodes
C Para-aortic lymph nodes
D Superficial inguinal lymph nodesans
E Superior mesenteric lymph nodes
The vulva

Q) 99% of body calcium is in what form? Calcium


Bicarbonate
Calcium Gluconate
Calcium Phosphate.ans
Calcium Carbonate
Calcium Hydroxide

Q) A 14-year-old child presents to the adolescent gynaecology


clinic. She has a history of virilisation after undergoing
pubertal changes. The karyotype reveals 46XY. An ultrasound
scan does not show the presence of a uterus and ovaries.
Which enzyme deficiency may be associated with these
clinical features?
A. 5-alpha-reductase deficiency ..ans
B. Complete androgen insensitivity syndrome
C. Kallmann syndrome
D. Polycystic ovary syndrome
E. Turner syndrome
The answer is 5-alpha-reductase deficiency. Turner syndrome
is 45 XO. Polycystic ovarian syndrome has a normal female
karyotype. The child described above is genetically male.
However, testosterone is not converted to dihydrotestosterone
in target tissues. Five-alpha-reductase deficiency prevents
conversion of androgen to estrogen. The child may have been
born with ambiguous genitalia and raised as a female. At
puberty, elevated levels of androgen lead to masculinisation
including virilisation. Complete androgen insensitivity is not

the answer because increased androgens are converted to


estrogen and do not show any virilisation. It involves
phenotypic females and an unresponsiveness to
androgens.Kalman's syndrome is a form of hypogonadotropic
hypogonadism and anosmia.
Q) What is the action of cytochrome P450
enzymes?
a. Always acts an isomerase
b. Always acts as a transferase
c. Always adds an alcohol group
d. Always adds a terminal electron
e. Always catalyses hydroxylations..ans
Q) Which marker can be used to assess the
functional status of the fetal adrenal gland?
a. estradiol
b. DHEA
c. estrone
d. estriol.ans
e. testosterone
placenta does not express CYP17A enzyme which
is necessary to convert progestin to
progesterone. so it works with fetal adrenal. fetal
adrenal produces
16alphahydroxyandrostenedione and through
aromatization, estriol is produced by the
placenta. Estriol is a unique steroid which is only
synthesized by the placenta. Thus , estriol is a

surrogate marker to assess the functional status


of the fetal adrenal gland.
Q)

Ans FRC
Q) You are discussing radiation doses for an abdominal CT.
The radiation dose of an abdominal CT is equivalent to which
of the following? 40 Chest X-rays
1 year natural background radiation
400 Chest X-rays.ans

10 years natural background radiation


40 Days natural background radiation
An andominal CT is equivalent to 400 chest
XRAYs or 2.7 years background radiation.
Q) hypervitaminosis C associated with: 1-oxalate
stone.ans
2-sensory neuropathy
3-motor neuropathy
3-iron def anemia
Vitamin C toxicity is a rare case as Vitamin c is
water soluble, excess amount will be excreted via
kidneys. But still there is a chance of vitamin c
toxicity and death.
Anything in excess can kill you. for an example,
water toxicity can kill a person. If you exceed the
normal requirement of vitamin c, you are running
a risk of vitamin c.
The upper limit for vitamin C intake is 2000
mg/day.Common side-effects vitamin c toxicity
1.
2.
3.
4.
5.
6.
7.

Indigestions
Diarrhea
Nausea
Vomiting
Headache
Fatigue
Disturbed sleep

8. Skin rashes
Possible side-effects of vitamin toxicity
1. Iron poisoning as vitamin c enhances the
absorption of iron.
2. Heamochromatosis
3. Heamolytic anemia can be precipitated if the
patient is enzyme glucose-6-phosphate
dehydrogenase (G6PD) deficient one.
4. Kidney stone
5. May precipitate abortions
Q) Which of the following is a branch of the
femoral artery?
A Peroneal artery
B The ascending genicular artery
C The deep epigastric artery This is the correct
answer
D The deep external pudendal artery
E The inferior gluteal artery
Q) About octreotide one is false - A)is somatostatin analogue
B)used v treatment of carcinoid syndrome
C)used in treatment of esophageal varices
D)can cause biliary calculi
E)none aboveans
Q) Steroid is lipophilic. It exerts its action by free access into
cell and binds to intracellular receptor. Prostagladin is also

lipophilic and synthesized from fatty acid. How does it exerts


its action? a. intracellular receptor
b. G protein coupled receptor..ans
c. ion channel receptor
d. throxine kinase linked receptor
e. receptor with intrinsic enzyme activity
although chemical property same as steroid... prostaglandin
and steroid differ in many ways. one of it is their interaction
with cellular molecules. steroid via intracellular signalling,
prostagladin via G couple protein
other difference
1. steroid takes hours or days to exerts its action. prostagladins
takes minutes or seconds to exert its action
2. steroid is from cholesterol. Prostagladin is from arachidonic
acid fatty acid)
3. steroid responsible for sex hormones functions.
prostagladin for initialtion of labour.
Q) TTP associated with all except - A)Inc BT
B)Dec platelets
C)Inc PT.ans
D)shistocytes on peripheral film
E)none above
Q) Homologue of fallopian tube - A)appendix
testes..ans
B)appendix epidydimis
C)epoopheros

D)prostate
E)paraoopheros
Q) Ergometrine malete-which is true?
Causes an increased central venous
pressure..ans,causes tetanic contractn
Prostaglandin analogue
Causes spasmodic contractions of uterus
Causes arterial vasodilation
Is a cycli oxygenase 2 inhibitors
Q) Which of the following is the drug of choice for
the treatment of Chlamydia trachomatis infection
during pregnancy? A Amoxicillin .ans
B Cephazolin
C Clindamycin
D Metronidazole
E Tetracycline
Chlamydia infection in the nonpregnant state is
usually treated with a tetracycline
(doxycycline100mg BD for 7 days), or with
azithromycin 1g in a single dose. Erythromycin
and ofloxacin can be used if the first line
treatments are contraindicated.
During pregnancy, tetracycline therapy is
contraindicated because of its incorporation into
fetal bones and teeth. Treatment options are
therefore erthromycin or amoxicillin or
azithromycin.

Q) Which of foll is the primary opsoniin the coplememt


system? C3a
C3b..ans
C4b
C5a
C5b
Q) Calcium is ECF 12000 times than ICF
In ECF it bound to
Plasma proteins phosphate and bicarbonate
And 45% in ionized form and 55% bound form
Q) Rate/ chances of transmission of hepatitis HBsAg positive
mother?
If mother positive HBsAg and HBeAg percentage is 70-90%
If just HBsAG positive it is 10%
Q) You answer an emergency call for a postpartum
haemorrhage. The midwife estimates that the patient has lost
approximately 1 l of blood. What should you be your first
action?
A. Assess the patients airway, breathing and circulation and
administer oxygen at a rate of 15 l/min ans
B. Bimanual compression of the uterus
C. Catheterise the bladder
D. Obtain blood for cross match of 4 units
E. Site two large bore intravenous cannulae

The answer is assess the patients airway, breathing and


circulation and administer oxygen at a rate of 15 l/min. In an
acute emergency, one should always assess the airway,
breathing and circulation before addressing the secondary
treatment. In practice, this may be talking to the patient and to
see if they respond.
Q) A primigravid woman presents in spontaneous labour at 39
weeks of gestation. At 18:00h, her cervical dilatation is 6 cm.
A further vaginal examination at 22:00h reveals that cervical
dilatation is still at 6 cm. At 02:10h, the fetus is in the
occipitoposterior position and uterine activity is present. What
is the most appropriate action?
A. Amniotomy .ans
B. Commence intravenous oxytocin
C. Membrane sweep
D. Repeat vaginal examination after 2 hours
E. Repeat vaginal examination after 4 hours
The answer is amniotomy. This case demonstrates slow/no
progression during the first stage of labour and malpositioning
of the fetus. The most appropriate initial intervention would
be an amniotomy (artificial rupture of the membranes).
Q) What percentage of pregnancies are comolicated by
GDM ? A)1%
B)1-5%.............ans
C)10-15%
D)20%
E)25%

N) Restriction fragment length polymorphism


characterised by-Is used for DNA fingerprinting
Is used to identify difference in protein
expression
Requires the use of DNA polymerase
Is used to identify difference in gene expression
Doesn't require the use of DNA probes
Q) The doppler effect refers to which of the following
The change in wave direction as it passes from one medium to
another.
The freq shift of reflected sound waves assoc with movement
of an object in respect to the transducer ans
The effect when sound waves are greater than the structure
they come into contact with causing uniform amplitude waves
in all directions with little or no reflection returning to the
transducer.
Apparent bending of waves around small obstacles
The decreasing intensity of a sound wave as it passes through
a medium
Q)

Extra cellular is 55%


40% bound to albumin
10 % bound to bicarbonate, phosphate, lactate
Q) Where in the body is ATP found?
a. Both intracellular and extracellular
b. Only extracellular
c. Only intracellularans
d. Only within high energy output cells
e. Only within mitochondria
ATP is produced in the cytoplasm (anaerobic) and
the mitrochondia (aerobic) which are both
intracellular.
Q) Where in the body is calcidiol produced? Kidneys
Liverans
Parathyroid
Skin
Spleen
Calcidol is 25 hydroxycholicalcideril which is formed in the
liver by 25 hydroxylation then converted to 1,25 DHH by 25
hydroxylation in the kidney
Q) At what gestation does the fetal heart first become
detectable on ultrasound?4 weeks
6 weeks..ans
7 weeks
8 weeks
9 weeks

Q) Which mediator that is responsible for the drop of blood


pressure in pregnancy?a. eNOSans
b. iNOS
c. bNOS
d. prostacyclin
e. prostaglandin
endothelium nitric oxide synthase e ..endothelial
i... inducible
b... brain
Q) woman with history of multiple intercourse had ulcer in
cervix first line investigation:
a) pap smearans
b) cervical biopsy
c) vaginal douch and fellow up after 4 weeks
N) Transfusion of blood n blood products:
Immuno supression is a recognized complication
of transfusion
Stored red blood cells have a high conc of 2,3
bisphosphiglycerate
Stored red cells have lower oxygen affinity
compared to freshly donated red blood cells
Hypercalcemia is recognized complication
Plasma conc of pottasium in stored blood is lower
then in freshly donated blood
Q) Not transfusion transmissible infection - A)
salmonella

B) brucella
C) CJD
D) Hep A
E) streptococci.ans
Q) What is the main buffer in urine? a. Ammonia
b. Bicarbonate
c. Chloride
d. Haemoglobin
e. Phosphate.ans
Q) A 32-year-old Asian woman presents at 36 weeks of
gestation with abdominal discomfort, 2+ proteinuria and a
blood pressure of 140/90 mmHg. She has blood tests in
accordance with the NICE guideline for the management of
hypertension in pregnancy. The midwife asks you to review
the following blood results. What do these results suggest?
Urea 2.8 mmol/l
Creatinine 67 micromol/l
Sodium 138 mmol/l
Potassium 4.2 mmol/l
Urate 0.37 mmol/l
Albumin 32 g/l
Alkaline phosphatase 198 iu/l
Alanine transferase 33 iu/l
Bilirubin 5 mmol/l
The answer is Normal blood results for 36 weeks
of pregnancy.
Q) An Asian woman books in for her third pregnancy at 12
weeks of gestation. She has recently moved to the UK from

Thailand to be with her new husband. After pre-test


counselling, with the aid of an interpreter, she agrees to
hepatitis B virus (HBV) screening. The results return as
follows. What is the significance of these results?
HBsAg POSITIVE
Anti-HBc POSITIVE
Anti-HBc IgM NEGATIVE
HBeAg NEGATIVE
Anti-HBe POSITIVE
HBV DNA 203 iu/ml
The answer is Chronic infection (immune control
phase). HBsAg is a marker of infectivity. Its
presence indicates either acute or chronic HBV
infection. Anti-HBc (IgG antibody to core antigen)
usually remains positive for life following HBV
infection. Anti-HBc IgM is found in high
concentrations in acute infection, gradually
declining and complementing the rise in Anti-HBc
IgG. The presence of Anti-HBe suggests a low
viral titre and a low degree of infectivity. HBV
DNA is a dynamic parameter in chronic HBV.
Below 20 000 iu/mlthere is a relatively low
likelihood of hepatic damage.
Q) A woman attends for a pre-operative review in preparation
for gynaecological surgery. Routine blood tests are performed,
including a full blood count. One of the clinic nurses remarks
that the patient looks mildly jaundiced. Later that day the
laboratory informs the following results:

Haemoglobin 7.6 g/l


White cell count 2.5 x 109/l
Platelets 275 x 109/l
MCV 109 fl
MCHC 30 fl
Haematocrit 21%
Reticulocytes >15%
The answer is Haemolytic anaemia. Pernicious
anaemia is the potentially difficult differential
diagnosis in this situation but reticulocytes are
not generally increased in pernicious anaemia
until treatment is commenced. Less than 2% of
circulating red blood cells are reticulocytes.
Nucleated red blood cells (normoblasts) are not
normally seen in the peripheral circulation.
Q) A healthy 34-year-old woman had a total abdominal
hysterectomy for cervical disease. Routine urea and
electrolytes are measured one day after surgery and are as
follows:
Sodium 121 (reference range 135145 mmol/l)
Potassium 4.3 (reference range 3.55.0 mmol/l)
Urea 2.8 (reference range 2.56.7 mmol/l)
Creatinine 74 (reference range 70150 micromol/l)
What is the most likely cause to explain the blood test results?
A. Excessive intravenous dextrose .ans
B. Nephrogenic diabetes insipidus
C. Primary aldosteronism

D. Undiagnosed diabetes mellitus


E. Ureteric damage
The answer is Excessive intravenous dextrose.
Hypernatraemia is common after excess IV normal saline but
hyponatraemia can occur with excessive 5% dextrose IV.
Q) When do the corona radiata cells appear?
A) at birth
B) 12 days..ans, 12 d after ovulatn
C) 28 days
D) 1 day
Q) Polyglactin sutures are used extensively in surgical
procedures, particularly to ligate vessels. What are the key
features of polyglactin sutures?
A. Braided, absorbable and synthetic ans
B. Braided, non-absorbable and synthetic
C. Non-braided, absorbable and natural
D. Non-braided, absorbable and synthetic
E. Non-braided, non-absorbable and natural
The answer is braided, absorbable and synthetic. Polyglactin
sutures are used to ligate pedicles and close the uterus during
a caesarean section. To achieve this, the sutures are braided to
prevent the knots from slipping.
Q) A nulliparous woman presents with spontaneous rupture of
membranes at 41 weeks of gestation. At 18:00h, her cervical
dilatation is 3 cm. A further vaginal examination at 22:00h
reveals that her cervical dilatation is still 3 cm. At 02:10h, the

fetus is in the occipitoposterior position and uterine activity is


present. What is the most appropriate action?
A. Administer prostaglandin per vaginam
B. Caesarean section
C. Commence intravenous oxytocin .ans
D. Membrane sweep
E. Repeat vaginal examination after 4 hours
The answer is commence IV oxytocin. The membranes have
ruptured already so amniotomy is not required. No
progression has been made during the first stage of labour.
Therefore, the patient should be administered intravenous
oxytocin.
Q) which does not cross placenta ? a. heparin..ans
b. morphine
c. naloxone
d. warfarin

Q)

Ans A
Q) which antihypertensive drug is typically
associated with tolerance on long term use??
Labetolol
Captopril
Enapril
Methyldopa
Hydralazine..ans
Q) Regarding FISH which one following is correct?
Is used to detect mosaics
Can be used to detect unbalanced translocation
Can be used to detect gene mutation
..ans

Has false positive 1:4000


Can be used to detect balanced translocation
Q)

For infection
First see at HBsAg ..positive or negative
If positive that means infection Ist step
Now acute or chronic

In acute IgM core antibodies will positive


In chronic it will be negative plus in chronic
sometimes data can give presence of Hep b
DNA ..highly indicative of chronic infection
Q) A 68-year-old woman presents with two episodes of
postmenopausal bleeding. She has a BMI of 23 and is
otherwise healthy. An ultrasound shows that her endometrial
cavity is 4 mm thick, and an endometrial pipelle sample is
taken that yields a small volume of tissue. The pathology
report suggests a neoplasm. What is the most likely diagnosis?
A. Leiomyoma
B. Endometrial hyperplasia
C. Endometrial polyp
D. Endometrioid adenocarcinoma
E. Serous carcinomaans
The answer is serous carcinoma. Serous
carcinomas are typically seen in postmenopausal
women. The development of these carcinomas is
not associated with a raised BMI, diabetes or
hypertension. The uterine tumour can be very
small (even in the presence of extra uterine
spread), and therefore, results from an
ultrasound and even hysteroscopy can appear
normal.
Q)Fibroid degen Hyaline is most common
Red degeneration is specific to pregnancy

Q) A CTG is performed in labour following normal


pregnancy. The reading has a baseline rate of 120 bpm, a
variability of 5 bpm, no decelerations and no accelerations. A
fetal blood sample has been performed and the pH is 7.25.
What is the most appropriate management?
A. Delivery is indicated
B. Do not repeat the fetal blood sample unless the CTG
deteriorates ans
C. If the CTG remains the same repeat the fetal blood sample
in 30 minutes
D. Maternal oxygen therapy and repeat the fetal blood sample
in 20 minutes
E. Repeat the fetal blood sample in 30 minutes regardless of
the CTG
The answer is Do not repeat the fetal blood
sample unless the CTG deteriorates. The CTG is
reassuring according to NICE guidelines so the
primary fetal blood sample (FBS) was not
needed. The FBS result is also normal.
Q) which foll drug suppress pituitary LH
production,hepatic SHBG production and use to
treat Hirsutism?Spirinolectone
GnRH Analogue
Flutamide
Gestrinone
COCP..ans
Q) Which of the following vessels is a branch of the anterior
trunk of the internal iliac artery? A Middle rectal artery

B Median sacral artery


C Obturator artery
D Ovarian artery
E Uterine artery..ans

Q) Which one of the following nerve roots mediates the anal


reflex?
A C5, C6
B C7, C8
C C8, T1
D L5, S1
E S3, S4.ans
Q) ureters are derived from? Mesonephric duct
Q) Which GFR calculator is recommended by NICE? Shwartz
Formula
CKD-EPI Formula
MDRD formulaans
Mayo Quadratic Formula
PANG Formula
eGFR is estimated GFR calculated by the abbreviated MDRD
equation : 186 x (Creat / 88.4)-1.154 x (Age)-0.203 x (0.742 if
female) x (1.210 if black)
Q) A midwife asks the obstetric registrar to review a
primigravida in labour who has progressed to 6 cm dilatation
with an abnormal CTG. She has been monitored by
continuous electric fetal monitoring because of suspected fetal
growth restriction. The CTG has been normal up to 30
minutes previously.
The registrar reviews the patient and confirms that the CTG
has a baseline heart rate of 150 bpm with a baseline variability
of 4 for the last 30 minutes. There were no accelerations but

variable decelerations were present. What would be


appropriate management with these CTG findings?
A. This is a normal CTG and no intervention required
B. This is a suspicious CTG and requires continued obstetric
review ..ans
C. This is a pathological CTG and fetal blood sampling is
required
D. This is a pathological CTG and urgent delivery is required
E. This is a suspicious CTG and fetal blood sampling is
required
The answer is This is a suspicious CTG and requires
continued obstetric review. This is because there is just one
abnormal feature variable decelerations. The decreased
variability and lack of accelerations have only been present
for 30 minutes, not yet long enough to classify as abnormal
features
Q) serum lactate value indicative of severe sepsis
> 4 mmol/l
Q) A middle aged woman who is currently 12
weeks oregnant is referred for a routine us scan.
The following are all commonly detected at this
time except:
Anencephaly
Oligohydramnios
Cystic hygroma
Bladdwr outflow obstruction.ans
Abdominal wall defects

Q) The following serum markers increase during pregnancy


except - T3
Protein
Alkaline phosphatase
T4ans
TBG
Q) During childbirth, anesthesia is administered
into epidural space of spinal column. Where is
the epidural space located?
A. Between supraspinous and interspinous
ligaments
B. Betw wall of vertebral cavity and dura
mater.ans
C. Between arachnoid and dura mater
D. Between arachnoid and pia mater
E. Between pia mater and spinal canal
Q) In early pregnancy at what gestation does the Embryonic
pole become visible on transvaginal ultrasound? 4 weeks
4 weeks + 3 days
5 weeks
5 weeks + 3 days..ans
7 weeks
Q) most imp anion in urine - a. albumin
b. phosphate
c. chloride..ans
d. bicarbonate
e. nitrate

Q) Direct lymphatic drainage of the breast does not occur via


which of the following? A Abdominal lymph nodes
B Axillary lymph nodes
C Mandibular lymph nodes..ans
D Parasternal lymph nodes
E Trochlear lymph nodes

Q) Which cells are not used for PIGD?


Polar body from oocyte
Inner cell mass ..ans
Tropoectoderm cell
Polar pody from zygote
Blastomere from cleavage stage embryo

Q)

Q) In fetal circulation, blood passes from IVC to left ventricle


through??
1) ductus arteriosus
2) foramen ovale..ans
3) ductus venousus
Q) What makes dimple in gluteal region? A)
ischial spine
B) post sup. iliac spine ..ans
C) post inf. iliac spine
D) sacroiliac ligament
Q) In fetal circulation, umbilical venous blood
before entering the inf vena cava passes through
which structure? 1) foramen ovale

2) ductus venousus ans


3) ductus arteriosus
Q) Karyotype for complete mole. - A.69xxy
B.69xxx
C.46xx ..ans
D.46xy
Q) An MRI examination is NOT allowed under any
circumstances when the following is present A. A pacemaker
B. A hip/knee joint replacement
C. An intracranial aneurysm clip
D. A metallic heart valve
E. A first-trimester pregnancy
Ans A,C
Q)Absolute CI of HRT is ? Thrombosis ..ans
fibrocystic disease
fibroadenoma
haemorrhage
Q) first heart beat seen on USG - day 20
day 22
day 28
day 32 . Ans
dont confuse first heart beat in embryo is on day
21
first heart beat dectetd in usg is day 32

Q)

Ans C
Q)Prevalence 0.5 hep B and 0.3-0.7 hep C
N)Following cross placenta, regards as teratogenic :- Varicella
Rubella
HSV
Toxoplasmosis
Syphilis
CMV
Q) HRT is helpful in all of the following except ? vaginal
atrophy
flushing
osteoporosis
coronary heart disease . Ans

Q) Bifurcation of aorta at this level Uppermost edge of iliac crest . Ans,this level
corresponds to L 4
Anterior superior iliac spine
Anterior inferior iliac spine
Sciatic notch
Ischial spine

Q) A registrar is asked to review a primigravida in labour who


has progressed to 6 cm dilatation and has an abnormal CTG.
The registrar reviews the case and confirms that the CTG has
a baseline rate of 155bpm, a baseline variability of eight, no
accelerations and variable decelerations. What is the correct
categorisation of this CTG?
A. Non-reassuring CTG
B. Normal CTG
C. Pathological CTG
D. Pathological CTG with reassuring features
E. Suspicious CTG.ans
The answer is Suspicious CTG. A suspicious CTG is defined
as a CTG where one of the features is non-reassuring and all
other features are reassuring.
Q) Gestational age of CRL of 30mm - 6 weeks
8weeks
10weeks .. ans
12weeks

Crl+42 days=weeks
30+42days=weeks
72days7=weeks
10.02weeks=10weeks
Q) According to WHO , Hb in pregnancy shouldn't be less
than -8
10..ans
9
11
13
Q) DNA duplication occur in - G1
Sans
G2 phase
Q) Mc uterine immune cell - A)macrophage
B)natural killer .ans
C)dendritic
D)B cell
E)T cell
Q) all of the foll appear to decrease hot flushes in
menopausal women except ? androgens
raloxifene .ans
isoflavones
tibolone
Treatment of vasomotor symptoms SSRIs:
fluoxetine and paroxetine. SNRI: venlafaxine.

Clonidine (-agonist): once mainstay treatment,


but now shown ashaving limited effect.
Q)

Ans C

Q) A 36-year-old woman who had supraspinous


ligament fixation 2 days ago complains of pain
over right mons pubis, right labia, and the
perineum. Which nerve is most likely to be
injured?A. Iliohypogastric nerve
B. Ilioinguinal nerve
C. Genitofemoral nerve
D. Pudendal nerve .ans
E. Posterior femoral cutaneous nerve
Q) potassium is absorbed in? PCT

Q) A 35-year-old married woman presents to her


GP. She complains of feeling unwell with nonspecific joint pains and perianal discomfort. On
examination you note evidence
of wart like lesions on her perineum. She admits
to being unfaithful to her husband for several
years. What is the most likely diagnosis? a
Primary syphilis
b Secondary syphilis.ans, wart like
lesion= condylomata lata,at secondary stage of
Syphilis
c Latent syphilis
d Tertiary syphilis
e Congenital syphilis
Q) The following statements are all true with
regards to labour EXCEPT:
a Engagement is defined as less than or equal to
two-fifths of the fetal head palpable above the
pelvic brim
b In maximum flexion the posterior fontanelle
can be palpated vaginally
c Internal rotation involves rotation of the fetal
head from the occipito transverse to the left
occipital transverse position
d Restitution is where the fetal head reverts to
the transverse position
e Descent tends to occur just before the onset of
labour in Afro-Caribbean women..ans

Q) what is the nerve supply to the skin of perineum


inf gluteal
inf rectal..ans
superficial perineal
deep perineal
obturator
Q) At a booking (12 w) appointment a 24-y primi complains
of fainting episodes and notices her hands and feet feel
warmer than usual.On examn her observations are normal
(i.e., BP and P). Auscultation of heart reveals an ESM in 2nd
left IC space.Which of foll would explain these findings?
(Please select 1 option) A) A rise in central venous pressure
B) Cardiac output drops in pregnancy
C) Peripheral vasoconstriction in pregnancy leads to an
increase in vascular resistance
D) The pulmonary vascular resistance increases in pregnancy
E) The stroke volume is increased in pregnancyans
Q) A 25 -year-old woman presents to A&E. She is currently
41 weeks pregnant. She is referred to the obstetric registrar on
call who feels she is a candidate for immediate surgical
induction of labour. What is the most appropriate Bishops
score required prior to such induction? a 1
b2
c4
d5
e 7 and aboveans

Q) Fetal hydrope is associated with. - A . toxoplasmosis


B.parvovirusB19..ans
C.Rubella
D.treponema pallidum
E.CMv
Q) A patient with 2 prev c/s presents at 35 w with a painless
vaginal blood loss of 400 ml. She is unbooked, that is, she
has not presented for any previous AN appointments so far in
this pregnancy.Clinical finding are as follows: BP 120/75
Pulse 78 bpm
No clinical e/o hypovolaemia
Urinalysis Normal
Examn Abdominal mass is soft, not tender, with a Tr lie
Cardiotocography (CTG) Normal
What is the most likely cause of the blood loss?
A. Cervical fibroid degeneration
B. Likely abruption
C. Likely placenta praevia .ans
D. Likely vasa praevia
E. Show
Q) A 24-year-old woman presents to A&E
following an episode of unprotected intercourse.
She complains of generalised abdominal pain and
irregular bleeding.Speculum examination reveals
evidence of a discharge. Bimanual examination
demonstrates evidence of cervical excitation.
What is the most likely aetiological cause for
such symptoms? a Neisseria gonorrhoeae.ans

b Treponema pallidum
c Chlamydia trachomatis
d Gardnerella vaginalis
e Haemophilus influenzae
Q) A 71-year-old woman has presented with recent onset of
vomiting. She vomits up the content of every meal
approximately two hours after eating.On examination she has
a palpable mass to the right of the midline in the epigastrium.
Which one of the following is the most likely electrolyte
disturbance in this scenario?(Please select 1 option)
A)High chloride, low bicarbonate
B)High potassium, high sodium, high bicarbonate
C)Low chloride, low sodium
D)Low K, low CL-, high HCO3 ..ans,contraction alkalosis
E)Low potassium, low sodium, low chloride, low bicarbonate
Q) A patient who is 36 weeks pregnant comes to see you as
she has developed tingling to the right lateral thigh over the
past 3 weeks. On examination there are no skin changes and
no muscle weakness. What is the likely diagnosis? Obturator
nerve entrapment
Pudendal nerve entrapment
Femoral Nerve entrapment
Meralgia Paraestheticaans
Shingles

Q) Which one give maximum radiation during


checking tubal patency

Ans B
Q)

Ans C
Q) Crypytococus falls in to which fungi category - A) mould
B) yeast like
C) diamorphic
D) true yeastans

Q) The following are all common contraindications to the use


of the Mirena coil EXCEPT: a Pregnancy
b Liver failure
c Renal failure..ans,rest are all CI
d Mechanical heart valve
e Ovarian carcinoma
Q)

Q) How many copies of AA sequence present in


TRH precursor? A.2
B.4
C.6..ans
TRH is synthesized as a 242-a a precursor
polypeptide that contains 6 copies of the
sequence -Gln-His-Pro-Gly-, flanked by Lys-Arg or
Arg-Arg sequences.
Q) Which one is true for congenital hip dislocation ?
A m.c in African population
B m .c in first born female babies ans
C m.c in first born in male babies

D needs x-Ray for confirmation


E only surgically treated
Q) Epilepsy and pregnancy true/ false
Combination of multiple low dose drugs recommended.F
Risk of congenital malformation is increased by 2 to 3
fold..T
Cognitive deficits in older children have been reported with
use of valproate in utero.T
Oxcarbemezapine is highly toxic to be used in
pregnancy.F

Q) Prostratic utricle

Q) The epithelia of the amnion ? 1. Squamous


2. Cuboidal ..ans
3. Transitional
4. Simple
Q) Are the following true regarding the fallopian tube? True /
False
A) Has a thick muscle layer in the isthmus
B) Is actively motile

C) Is covered by peritoneum
D) Lies anterior to the round ligament.F
E) Possesses a cilial lining
D is F rest are T
Q) Are the following statements about the
lymphatic drainage of the genital tract true ? True
/ False
A) Drainage from the corpus uteri goes partly to
the superficial inguinal nodes
B) Drainage from the oviducts is mainly via the
para-aortic nodes
C) Ovarian drainage is directly to the para-aortic
nodes
D) The lymphatics of each side of the vulva
communicate with each other
E) The middle third of the vagina drains to the
superficial inguinal nodes.F
A-D are true
Q) Lithium and malformations true/false
Ebsteins anomaly is the displacement of the tricuspid valve
towards the apex of the right ventricle
It gives rise to a large right atrium
Occurs about 4 weeks after conception
Lithium is associate with ASDs and VSDs
The use of lithium during the last trimester is problematic
All true

Q) Vasectomy is assoc with which one of above


complications?A An inc risk of coronary heart disease
B An increased risk of prostate cancer
C A 10% risk of developing anti-sperm antibodiesF,75 %
D An increased risk of epididymo-orchitisans
E A failure rate of 1:20,000
Q) Which drug trial phase primarily assesses the
safety of a drug? A. Phase IV
B. Phase III
C. Phase II
D. Phase I ..ans
E. Phase 0
IV - Post-launch safety surveillance
Phase III - Randomised control trial
Phase II - Assesses how well the drug works as
well as continued safety
Phase I - Assesses safety
Phase 0 - First human trial - does it behave in
humans as expected
Q) A first-line treatment option for a pregnant
woman at 6 weeks of gestation who has seasonal
allergic rhinitis and no history of drug allergies or
adverse drug reactions is
A) loratadine.
B) chlorpheniramine..ans
C) nasal beclomethasone.

D) oral prednisone.
E) all of the above.

Q) What has the following FISH analysis most


likely identified (del22)?A. Wolf harrison
syndrome
B. Angelman syndrome
C. Prader willi syndrome
D. Digeorge syndrome.ans
E. Cri du chat syndrome

Examples of syndrome due to structural chromosomal


abnormalities caused by deletion :
Wolf harrison syndrome 4q15
Cri du chat syndrome 5q15
Digeorge syndrome 22q11
Angelman syndrome 15q11-13 (maternal deletion)
Prader willi syndrome 15q11-13 (Paternal deletion)
Q) Deficiency of which vitamin cause paresthesia
A)B1
B)k
C)B2
D)B3
E)B5ans

Q) Which of the following blood clotting factors


activates fibrinogen in common pathway?a.
Factor VIII
b. Factor XIII
c. Kallikrein
d. Kinin
e. Prothrombinans

Q) Your consultant has asked you to send a urea


and electrolytes test (U&E) for a postoperative
woman who has undergone a prolonged and
difficult transcervical resection of fibroids. What
electrolyte disturbance can occur after this
operation?
a. Hypernatremia
b. Hypokalaemia
c. Hyponatraemia..ans
d. Hypovolaemia
e. Increased serum osmolality
TURP syndrome It is characterized by
hyponatremia, hypervolumia leading to cerebral
edema and hypoosmolarity of serum.

Q) A patient arrives on labour ward she is 37 weeks pregnant.


Her last pregnancy ended with delivery via uncomplicated
lower segment C-Section 4 years ago. Contractions are 5
minutes apart and on examination and the cervix is 5cm
dilated. She wants to know the risk to the baby of proceeding
with vaginal delivery (VBAC). What is the additional risk the
baby will have respiratory problems after VBAC compared to
elective repeat C-section (ERCS)?
No difference
Risk 1 to 2% greater with VBAC
Risk 1 to 2% greater with ERCS..ans
Risk 4 to 5% greater with VBAC
Risk 4 to 5% greater with ERCS
The Green-top guidelines regarding Vaginal Birth After Csection (VBAC) state the following risks with VBAC:
2-3/10,000 additional risk of birth-related perinatal death
8 in 10,000 infant developing hypoxic ischaemic
encephalopathy
22-74 in 10,000 Risk of uterine rupture (previous lower
segment c-section)
1% additional risk of either blood transfusion or endometritis
VBAC reduces the risk of:
Reduces risk the baby will have respiratory problems after
birth: rates are 2 to 3% with planned VBAC and 3 to 4% with
ERCS
Q) A primigravida in the first trimester of pregnancy was
found to be sputum positive for acid fast bacilli. There is no

prior history of tuberculosis. What is the treatment of choice


for this patient?
A. Category I DOTS..ans
B. Category II DOTS
C. Category III DOTS
D. Start ATT after delivery
DOT direct observed therapy

Q) Minute ventilation in pregnancy increases due to which of


the following?
Effects of gravid uterus on diaphragm
Dliutional anaemia
Increased renal excretion of bicarbonate
Increased circulating oestragen
Increased circulating progesterone.ans
This is thought to be the result of increased
circulating progesterone. Progesterone is known
to directly stimulate ventilation by sensitizing the
CNS respiratory centres to CO2.
Increased minute ventilation "blows off" CO2 and
as a result pCO2 is reduced. pH homeostasis is
maintained via increased renal excretion of
bicarbonate.
Q) a child born with multiple congenital defects including
cleft palate , neural tube defect , ASD and microcephaly ,
which of the following drug was probably used by mother ?

a. erythromycin
b. isotretinoin..ans
c. ibuprofen
d. metronidazole
Q)

Ans C
Q) All the following therapies would be
appropriate for the acute treatment of migraine
in a pregnant woman except
A)
sumatriptan..ans
B) codeine.
C) acetaminophen.
D) propranolol.
Q) With regard to cardiac cycle, what is the definition of
stroke volume?A. Cardiac output/body surface area
B. End diastolic volume- end systolic volume..ans
C. End systolic volume-end diastolic volume
D. End systolic volume+ end diastolic volume
E. End diastolic volume+ end systolic volume

stroke volume= end diastolic volume-end systolic


volume
Q) Which progesterone is chemically related to
spironolactone?
A. Norethisterone
B. MPA
C. Dydrogesterone
D. Drosperidone.ans
E. Levonorgestrel
Q) In normal pregnancy:
Capacity of uterus is 4 lt at term
Blood flow to uterus is 5-8l/min
Uterus weighs about 3 kg at term
20-30% of cardiac output is directed to the uterus
Pre pregnancy uterus capacity is 10 ml and at
term is 5 lt.ans
Q) Mechanisim of PID & endometriosis is?
A.loss of ciliary action of fallopian tube
B.narrowing of fallopian tube
C.fibrosis
D.Scarring
E.all of these.ans
Q) The primary stimulus for ADH (anti-diuretic hormone
AKA Vasopressin) release is? Low serum osmolality
High serum osmolality.ans
Low blood volume

High Blood volume


Reduced ECF volume
Q) Which of foll tests is used to detect antibodies or
complement bound to red blood cell antigens in vivo?
Direct Coombs..ans
Indirect Coombs
Guthrie
Mantoux
Heaf
Direct Coombs detects antibody/complement attached to
antigens in vivo used to test RhD and ABO incompatibility
Indirect Coombs detects low levels antibody in serum in vitro
used in cross matching
Q) Regarding prenatal exposure to teratogen,
choose the correct option:
A. Large doses should be used for the shortest
time
B. All organs are sensitive for the same period of
time
C. There is no risk of teratogenicity after 8 weeks
D. Up to day 17 , any teratogenic effect results in
miscarriage..ans
E. Known teratogenic anticonvulsants should be
stopped in pregnancy

Q)

Ans C

Q) which one is potassium sparing diuretics??


Bendeoflumethiazide
Captopril
Doxazosin

Furosemide
Triamterene..ans

Q) Fertilisation leads to haploid number of


chromosomesF
------------------------------------------------------------Q) 31- Isolated diathermy machines
A- Are earth referenced
generators .................................................F

B- Operated in a frequency range of 400-600


kHz .............................F
C- Isolated machines are inherently
safer ........................................F
D- Diathermy can only pass back to the generator via the
patient plate.....T
E- Produce unavoidable large earth leakage
currents.........................F

Modern diathermy machines are isolated (unearthed)


generators as opposed to the old earth referenced generators.
The old earth referenced generators tend to produce higher
frequency current over a wider range than the narrow range of
the isolated generators. Current will only pass back to the
generator with no pathway back to earth i.e. a small area of
skin touching a metal contact (eg drip stand) will not result in
a burn. If the plate is omitted current will not flow. The
sophisticated electronics in the isolated generator ensures a
considerable reduction in earth leakage currents compared to
the earth referenced generators.
32- Non-disjunction of chromosomes during meiosis is a
cause of:[/u
A- Edward's

syndrome ....................................................................T
B- Turner's
syndrome .....................................................................F
C- Cri du chat
syndrome ................................................................F
D- Patau's
syndrome ......................................................................T
E- Laurence-Moon-Biedl
syndrome ....................................................F
Non-dysjunction (the failure of replicated chromosomes to
segregate during Anaphase II) during meiosis is responsible
most typically for Down's syndrome (trisomy C21), Patau's
syndrome (trisomy C13), Turner's syndrome (XO),
Klinefelter's (XXY) syndrome and Edwards syndrome
(Trisomy C18). Cri du Chat is due to a deletion of a portion of
C5.
33- Campylobacter jejuni:
A- attack rates are highest in the
elderly ..........................................F
B- infections are treated with
ciprofloxacin ........................................T

C- is a recognised pathogen in domestic


animals ................................T
D- is readily isolated in stool
culture .................................................F
E- causes
colitis ..............................................................................T

a-Young adults and children.


b-Cipro and Erythromycin, but most are self-limiting.
c-Transmitted to humans by milk or water infected by wild
and domestic animals and poultry.
d-Requires special conditions: 42C, micro-aerobic
atmosphere on blood agar with antimicrobials added.
e-Proctocolitis and enterocolitis may be due to sexually
transmitted agents such as CampylobacterEntamoeba
Lymphogranuloma venereumand may be clinically
indistinguishable from non-infective causes.
34- Mast cells:
A- Contain
heparin .......................................................................T
B- Degranulation releases lytic enzymes and inflammatory
mediators from storage granules ......F

C- Are lipophilic cells involved in inflammatory and immune


responses ...F
D- Cross-linkage of surface IgA molecules by antigen may
cause an anaphylactic reaction.......F
E- An excess of circulating mast cells causes
mastocytosis ...................T

Mast cells are basophilic cells (not lipophilic) in the


connective and subcutaneous tissues, which are involved in
inflammatory and immune responses. They contain storage
granules that contain lytic enzymes (e.g. tryptase) and
inflammatory mediators, e.g. histamine, heparin, 5-HT,
leukotrienes, platelet aggregating factor, leucocyte
chemotactic factor and hyaluronidase. Release of these
mediators occurs during mast cell degranulation, which can be
triggered by: tissue injury; drugs; complement activation; and
foreign antigenic material. An anaphylactic reaction occurs
when a previously sensitised individual is re-exposed to the
antigen. It is an IgE mediated immune response (not IgA).
Mastocytosis occurs when excess mast cells are present in the
circulation or as tissue infiltrates.
35- The following is true of diathermy safety features :
A- The person who applies the diathermy plate is responsible

for its correct application......F


B- Only isolated diathermy machines alarm when switched on
if the plate is not connected to the machine.........F
C- The patient plate is applied to ensure the current is moving
away from the electrocardiogram electrodes.......T
D- The area under the plate should have a good blood
supply.............T
E- Always shave the skin in contact with the diathermy
plate...............T
The surgeon using the diathermy has overall responsibility for
it and should check the alarm, wiring and plate before use.
Isolated and earth referenced monopolar diathermy machines
will alarm when switched on if the plate is not connected to
the machine, but only a few will alarm if the plate is not
attached to the patient. The plate should be applied close to
the operation site with the broad side placed perpendicular to
a line drawn from the operation site to the plate. A good
supply is necessary to dissipate any heat generated. The skin
should be shaven in all patients to ensure good contact
between the skin and the plate.
36- Which of the following statements regarding meiosis
is/are true/false ?

A- In meiosis II, whole chromosomes


separate..................................F
B- In spermatogenesis, meiosis begins at
puberty..............................T
C- Exchange of paternal and maternal DNA takes place in
meiosis II....F
D- Anaphase lag leads to numerical chromosome
aberrations..............T
E- Pairing of X and Y chromosomes in spermatogenesis is end
to end....T
In meiosis there is a separation of the chromosomes and
halving of karyotype to form germ cells. Meiosis (I and II)
comprises two successive nuclear divisions with only one
round of DNA replication.
37- The urinary system
A- develops from intermediate
mesoderm ........................................T
B- During intra uterine life 3 overlapping kidney systems are
formed ...T
C- The mesonephros develops as the metanephros

regresses ..............F
D- Bowman's capsule develops in the
metanephros ............................T
E- The glomerulus forms part of the
mesonephros ...............................F
The urinary system develops from the intermediate
mesoderm. During development of the fetus there are 3
overlapping kidney systems - the pro, meso, and metanephric
systems. The metanephros forms the permanent kidney.
Bowman's capsule and the glomerulus develop as part of the
metanephros.
38- The ureter:
A- Passes into the pelvis over the bifurcation of the internal
iliac artery...F
B- Is seen lying on the tips of the transverse processes of the
lumbar ...T
C- Has the genitofemoral nerve lying anterior to
it ...............................F
D- Is surrounded by Waldeyers sheath as it passes through
the bladder wall .....T

E- Lies anterior to the renal artery at the hilum of the


kidney ...............F
The ureter passes caudally lying on the psoas muscle and
crosses into the pelvis over the bifurcation of the common
iliac artery. It is seen lying on the tips of the transverse
processes of the lumbar vertebrae on an intravenous urogram.
the genitofemoral nerve lies on psoas hence lies posterior to
the ureter. Waldeyer's sheath is an investment of muscle
surrounding the ureteral opening in the bladder wall. The
ureter lies posterior to the renal artery at the hilum of the
kidney
39- The herpes group of viruses include:
A- Varicella-zoster
virus ....................................................................T
B- Papilloma
virus ............................................................................F
C- Rabies
virus ................................................................................F
D- Epstein-Barr
virus ........................................................................T
ECytomegalovirus .......................................................................

...T
The Herpesviridae family of viruses are dsDNA viruses that
include herpes simplex, varicella zoster, CMV and EBV. The
papilloma virus, a small dsDNA virus, is a member of the
Papovavius family. Rabies, a ssRNA virus, is a member of the
Rhabdovirus family.
39.The structures at risk of damage while cannulating the
subclavian vein include:
A- phrenic
nerve ..........................................................................T
B- subclavian
artery .....................................................................T
C- ansa
cervicalis .........................................................................F
D- anterior ramus of first thoracic
nerve ........................................T
Epleura .....................................................................................T
The subclavian artery lies inferior to the suclavian vein and
may be inadvertently cannulated in an attempt to cannulate
the suclavian vein. This may be of serious consequence as it is

not possible to put pressure on the subclavian artery to arrest


bleeding given its anatomical position. The apical pleura is
inferior and caudal to the subclavian vein and pleural
puncture, with, or without, pneumothorax are recognised
consequences of subclavian vein cannulation.
40.In acute allergic reaction
A- there is an increase in
bradykinins ...............................................T
B- T-helper cells are
involved ..........................................................T
C- there is an increase in the products of the 5-lipoxygenase
pathway ....T
D- the gene for allergy is located on chromosome
12 .........................F
E- may be triggered by acute complement
activation ..........................T
c) Leukotrines
d) The gene is located on chromosome 6
e) Anaphylatoxins
41.The amnion arises from epithelial cells between the
trophoblast

A- and ectodermal
disc ..................................................................T
B- The amnion has 5
layers ...........................................................T
C- the chorion has 4
layers ..........................................................T
D- the main layer of the chorion is the cellular
layer .......................F
E- the chorion is surrounded by the
blastocyst ................................F

The amnion is a layer of epithelial cells between the chorion


and the cell mass. It has 5 layers compared to the 4 layer
chorion. The chorion surrounds the blastocyst.
42.Concerning grief reactions
A- angry outbursts are a typical feature of the natural grieving
process ......T
B- hallucinations of the deceased can be a feature of the
normal grief reaction ......T
C- grief reactions may occur many years after the death of the

person ....T
D- suicidal ideations are a common feature of a grief
reaction .............F
E- Antidepressants is the most appropriate initial
therapy ...................F
Grief reactions are typically mapped along the lines of anger
denial and guilt. Delayed grief is said to occur if it
commences two weeks after bereavement. Grief reactions
often occur on anniverseries such as weddings, births etc. The
most appropriate treatment is conselling and antidepressants
should not be routinely used. Suicidal ideations are also
abnormal.

43.Which of the following are tumour suppressor genes?


Ap53 ...........................................................................................
T
BBRAC1 ......................................................................................
T
CAPC .........................................................................................T

D- bcl2 .......................................................................................F
E- cmyc .......................................................................................F
Mutations in Tumour suppressor genes have been implicated
in malignancy as down regulation of these genes can result in
the unrestricted growth of cells and hence predispose to
malignancy. These genes include VHL (von Hippel Lindau),
p53, Rb1(Retinblastoma), BRAC and APC (adenomatous
polyposis coli) genes. Oncogenes, on the other hand are
associated with the promotion of cell division and include
myc, erb, ras and ret.
44.Trinucleotide repeat sequences are seen in
A- cystic
fibrosis .............................................................................F
B- Duchenne muscular
dystrophy ....................................................F
C- myotonic
dystrophy ...................................................................T
D- fragile X

syndrome .....................................................................T
E- Leber's optic
atrophy ..................................................................F
also Huntington's chorea
In genetics, anticipation is a phenomenon whereby the
symptoms of a genetic disorder become apparent at an earlier
age as it is passed on to the next generation. In most cases, an
increase of severity of symptoms is also noted. It is common
in trinucleotide repeat disorders like Huntington's disease,
myotonic dystrophy and fragile X syndrome, where a
dynamic mutation in DNA occurs. All of these diseases have
neurological symptoms.
45.Which of the following drugs are teratogenic
AWarfarin ..................................................................................T

B- Oral
contraceptive ...................................................................F
CMetformin ..............................................................................F
DSimvastatin ...........................................................................T

ERanitidine ...............................................................................F

There is no evidence that ranitidine, metformin or the OCP is


teratogenic although it was once believed that aspirin and the
OCP were, studies indicate otherwise. Similarly, metformin is
often used in PCOs to induce fertility through reduction in
insulin resistance. Warfarin is associated with CNS and
skeletal abnormalities if foetal exposure occurs in the first
trimester, plus foetal haemorrhage is more likely. Statins also
are associated with teratogenicity.
46.In haemophilia A
A- dental extraction bleeding can be controlled with DDAVP
if factor VIII concentration is 2-5% of normal ..............F
B- sons of an affected man will be
normal ........................................T
C- Only about 90% have bleeding before one year of
age ...................T
D- hepatitis B virus is the most common cause of deranged
LFTs .........F
E- there is a normal amount of factor VIII-related

antigen ...................T
Only in mild cases (5-20% activity)is bleeding controlled by
DDAVP.
Haemophilia A is X-linked recessive. Sons of affected males
will inherit a normal Y chromosome, and will be unaffected.
Hepatitis C or E would be commoner as a cause of abrnomal
LFTs in these patients, as hepatitis B is more easily screened
for.
Factor VIII related antigen is reduced in von Willibrand's
disease.
90% present with bleeding by their first birthday. Only 30%
present with bleeding after circumcision.
47.Which of the following are Tocolytic:
ASalbutamol ...............................................................................
T
BSuxamethonium .......................................................................
F
CPropofol ..................................................................................F
DProgesterone ...........................................................................T

EGTN .........................................................................................
T
Tocolytic drugs, inhibitors of uterine contraction, include
GTN, alcohol, magnesium sulphate, ritodrine, salbutamol,
nifedipine and NSAIDs. Progesterone in high concentrations
also has some tocolytic activity and promotes the relaxant
effects of more conventional tocolytics.
48.Metronidazole:
A- Inhibits dihydrofolate
reductase. .................................................F
B- Has 80% bioavailability if given
rectally. ......................................T
C- Has harmful effects with
alcohol. .................................................T
D- Discolours the
urine....................................................................T
E- Causes peripheral
neuropathy......................................................T
Reduced to active derivative which binds to DNA and inhibits

acid synthesis. Disulfiram reaction with alcohol. Some types


of Warfarin. Metallic taste, hypotension, peripheral
neuropathy and also causes a darkening of the urine.
49.Polymerase chain reaction:
A- takes several days to
complete ....................................................F
B- DNA or RNA can be used as the
template ....................................T
C- helps in diagnosis of
infection ......................................................T
D- in diagnostic PCR the exact sequence at both ends of the
target region must be known .......T
E- Polymorphisms in the viral genome may result in
amplification failure .....T

The polymerase chain reaction is a rapid technique which


produces a result in only a few hours. It is therefore extremely
useful for rapid diagnosis of conditions such as tuberculosis
where traditional culture methods can take several weeks.
DNA is the standard template but viral RNA sequence can
also be amplified if the enzyme reverse transcriptase is used.
Primers used in the reaction must be complementary to the

nucleic acid sequence surrounding the region to be amplified,


therefore these sequences must be known. In HIV and perhaps
other viruses sequence polymorphism may prevent binding of
primers and result in failure of amplification. Primers which
were developed for amplification of the predominant clade B
strain found in Europeans and Americans have proved
unreliable for amplification of other HIV clades from Africa
and Asia.
50.The following are true of genes
A- The rate of DNA replication is directly under the control of
enhancer sequences.............F
B- Mitochondrial genes are inherited from the
mother..........................T
C- Transcription factors are mainly made of
RNA................................T
D- Introns are the portions of a gene which code for
protein................T
E- Most of the human genome encodes
polypeptide.............................F

A gene is a length of DNA that carries information to make a


single peptide chain. However, it is estimated that only a few

percent of DNA comprising the whole human genome


consists of genes encoding protein. Introns are intervening
sequences of unknown function in mammalian genes.
Regulatory elements of gene transcription include promoters
(regions of DNA to which RNA polymerase bind and initiate
transcription), enhancer sequences (modify activity of genes
on the same chromosome) and transacting proteins (modify
genes on both pairs of homologous chromosomes). mRNA are
transcripts of DNA, which are attached to transfer RNA and
then translated in ribosomes to protein, which then undergo
considerable post-translational changes. Mitochondrial
inheritance is exclusively maternal, as none of the
mitochondria from sperm survives fertilization.
51.The following mechanisms are involved in the mediation
of insulin action:
A- adenylate cyclase
activation ........................................................F
B- cell membrane receptor
interaction ..............................................T
C- inhibition of hepatic glucose
release .............................................T
D- receptor tyrosine kinase
activity ..................................................T
E- hormone receptor DNA

binding ....................................................F

Insulin acts through a disulfide-bonded heterotetrameric cell


surface receptor comprised of an extracellular alpha subunit
coupled via disulfide bonds to a transmembrane and
intracellular beta subunit. Insulin inhibits gluconeogenesis and
promotes glycogen synthesis. Signaling through the insulin
receptor occurs through an intracellular tyrosine kinase
domain and resultant phosphorylation of the receptor.
52.The seminal vesicles:
A- Contain
spermatids ...................................................................F
B- Contain
spermatids .................................................................F
C- Secrete acid
phosphatase .........................................................F
D- Secrete
fructose ......................................................................T
E- Secrete
prostaglandins .............................................................T
Seminal vesicles secrete a significant proportion of the fluid

that ultimately becomes semen. About 60% of the seminal


fluid in humans originates from the seminal vesicles. The
seminal vesicles do not contain sperm cells. Seminal vesicle
secretions contain proteins, enzymes, fructose,
phosphorylcholine and prostaglandins.

53.Regarding the human chromosomes:


A- There are 23 pairs of autosomal
chromosomes...............................F
B- The Y chromosome is larger than the X
chromosome......................F
C- Cells containing YO chromosome are not compatible with
life...........T
D- Barr body is caused by the presence of an inactive X
chromosome.....T
E- Barr body is only found in people who are phenotypically
female........F
There are 22 pairs of autosomes, and one pair of sex
chromosomes. The Y chromosome is smaller than the X
chromosome, no YO individuals have been identified, not
even aborted foetuses. It has been suggested that there is

something fundamental on the X chromosome that is needed


for life. The Barr body is visible during interphase and
chromosomes are too tenuous to be stained and seen by light
microscopy. However, a dense, stainable structure, called a
Barr body (after its discoverer) is seen in the interphase nuclei
of female mammals. The Barr body is one of the X
chromosomes. Its compact appearance reflects its inactivity.
People with XXY or XXXY karyotypes are males (because of
their Y chromosome), and display the features of Klinefelters
syndrome. The phenotypic effects of the extra X
chromosomes are mild because, just as in females, the extra
Xs are inactivated and converted into Barr bodies
54. Typical adverse effects of combined oral contraceptive
preparations include:
AMigraine ...................................................................................
T
BHyperprolactinaemia .................................................................
F
CDepression ...............................................................................
T
D- Breast

tenderness ....................................................................T
E- Loss of
libido ............................................................................F

Menstrual migraine: First migraines associated with a


hormonal event: menarche, birth control pills, pregnancy,
and/or postpartum Worse headaches occur near menstruation
Birth controls often make headaches worse; discontinuation
may bring some relief. Headache typically occurs the week
off birth control pills, Absence of migraine noted in second &
third trimesters of pregnancy. Presence of other premenstrual
complaints: mood swings, depression, food cravings, fluid
retention and breast pain. Side-effects include depression or
irritability. The effect on libido is variable. In many women
relief from fear of pregnancy removes a restraint on sexual
activity and enjoyment; in a very few there will be a loss of
libido. The other side effects of greatest concern with the
combined OCP are vascular events including DVT and stroke
(v rare).
55. The menopause
A- Implies cessation of menstrual flow for one year or
more ................T
B- Is synonymous with the
climacteric ..............................................F

C- Symptoms may occur long before menstruation


ceases..................T
D- Occurs on average at age 45.5
years ..........................................F
E- Is associated with
mennorhagia ..................................................F
The menopause is the cessation of normal menstruation.
Conventionally a woman has to stop menstruating for 12
months before she is considered to be postmenopausal.
Strictly speaking, menopause is defined as cessation of
menses for a minimum of 6 months because of inadequate
ovarian follicular development and waning oestrogen
production. The climacteric is an extended period of gradually
declining ovarian function often beginning years before and
lasting years after menopause itself. The average age of
menopause in the UK is 51 years, with a large majority of
women experiencing menopause between the ages of 45 and
55. The cessation of periods, can occur suddenly or may be
preceded by light and infrequent periods. The climacteric
(rather than the menopause) although typically associated
with light periods may be heralded by menorrhagia.
56. The benefits of adding androgen to menopausal hormone
replacement therapy include which of the following:

A- Increase in high-density lipoprotein cholesterol


levels......................F
B- Increase in
libido.........................................................................T
C- Decrease in triglyceride
levels......................................................T
D- Decrease in bone mineral
density.................................................F
E- Decreased cardiovascular
risk .....................................................F

Androgens are known to increase libido and may help to


protect bone mass. However, lipid profiles change when
androgens are added to HRT. The HDL fraction decreases
with androgen therapy; however, triglyceride concentrations
decrease significantly in patients given oestrogen-androgen
combination therapy.
57. Growth Hormone:
A- secretion is inhibited by somatostatin
analogues ............................T

B- is used in the treatment of proliferative


retinopathy .......................F
C- therapy causes an elevation of IGF-1
concentrations .....................T
D- is approved by NICE for the treatment of adult
hypopituitarism ......T
E- therapy is associated with an increased incidence of breast
carcinoma .............F

GH therapy produces an elevation of IGF-1 and therapy is


monitored through measuring these concentrations. GH
therapy is approved for the treatment of adult hypopituitarism
and there is no evidence to suggest that it causes an oncreased
risk in any malignancy. Treatment is contra-indicated in any
active malignancy and indeed proliferative retinopathy. GH
secretion is suppressed by somatostatin analogues - eg
octreotide, which are used therefore in acromegaly.
58. Proteolytic enzymes are released by the following
organisms:
A- Neisseria
meningitides ................................................................F

B- Salmonella
typhi .........................................................................F
C- Streptococcus
pyogenes .............................................................T
D- Mycobacterium
tuberculosis ........................................................F
E- Clostridium perfringens
(welchii) .................................................T

Virulence factors are important in the bacterial survival in


vivo. In this sense bacterial extracellular proteolytic enzymes
can be recognized as the legitimate target for this approach
since they are involved either in direct or indirect destruction
of an infected/colonized tissue and in dysregulation of many
host defense pathways. The best example of the last is an
effect of bacterial proteinases on fibrinolytic, kallikrein-kinin
and complement cascades, as well as degradation of
immunoglobulins, inactivation of endogenous proteinase
inhibitors, and dysregulation of cytokine network system.
Proteolytic enzymes are responsible for the virulence and
activity of organisms such as Strep Pyogenes, Staphylococcus
aureus, E coli and Clostridium welchii enabling the necrolytic
effects on the skin in cellulitis and gangrene

59. The thyroid gland:


A- Is stimulated by posterior pituitary
hormones ...............................F
B- Functions from the 12th week of fetal
development ......................T
C- Actively traps inorganic iodine from
plasma ..................................T
D- Stores colloid outside epithelial
cells .............................................T
E- Enlarges during normal
pregnancy ................................................T

Thyroid hormone production is stimulated by the anterior


pituitary hormone TSH and secretion begins from
approximately the 12th week of gestation. T3 and T4 are
manufactured within the thyroid cells through iodination of
tyrosine. The synthesized T3 and T4\ are then stored within
the colloid at the centre of the thyroid follicles. The thyroid
like most other endocrine organs moderately enlarges during
pregnancy.

60. The human testis secretes:


AAndrostenedione .......................................................................
T
B- Luteinising
hormone ...................................................................F
COestradiol ..................................................................................
T
DInhibin .......................................................................................
T
EFructose .....................................................................................
F

The testis is responsible for secreting testosterone,


androstenedione, Oestradiol, inhibin as well as a small amount
of progesterone. The anterior pituitary produces LH and FSH.
Fructose and prostaglandins that nourish the spermatozoa are
secreted by the seminal vesicles.

61. Staphylococcus epidermidis:


A- is coagulase
positive .................................................................F
B- on microscopy are Gram positive cocci in
chains .......................F
C- are usually sensitive to
penicillin ...................................................F
D- grown in blood cultures are due to contamination and
should be ignored ...........F
E- are destroyed by povodine
iodine ...................................................T

Staph. epidermidis is part of the normal skin flora. As for


Staph. aureus, Gram positive cocci in bunches are seen on
microscopy.
It is usually insensitive to penicillin.Although it is often a
contaminant in blood cultures, it is associated with line
infections. Central venous catheters are an especially likely
site of infection even in the absence of overt exit site sepsis.
They should not be ignored but interpreted in the clinical
context, and usually the blood cultures should be repeated. In
patients with central lines, blood cultures should be taken both

peripherally and from the central line as semi-quantitative


microbiological techniques are available that may point to the
central line as the principle site of infection.
62. The following hepatitis viruses are RNA viruses:
AHAV ...........................................................................................
T
BHBV ..........................................................................................
.F
CHCV ..........................................................................................
T
DHDV ..........................................................................................
T
EEBV ..........................................................................................
F

Hepatitis A is an RNA virus spread via the faecal-oral route.

Whilst both HBV and HCV are transmitted through blood and
blood products, HBV is a DNA virus and Hepatitis C is an
RNA flavivirus. Hepatitis D is an incomplete RNA virus,
existing only with H epatitis B, spread is with co-infection or
super-infection with Hepatitis B. Epstein Barr Virus is a DNA
virus, very similar to the other herpes viruses. It is shed in
pharyngeal secretions and transmission occurs via close oral
contact.
63. Regarding Immunoglobulin:
A- Immunoglobulins are secreted from Tlymphocytes. ......................F
B- An immunoglobulin G (IgG) comprises 2 antigen-binding
sites and a site for the binding of complement.........T
C- The molecular structure of IgG is a Y
shape ...................................T
D- IgG constitute approximately 25% of all immunoglobulis
in a healthy individual........F
E- IgMs can cross the placenta to the
foetus........................................F

Immunoglobulins (antibodies) are secreted from Blymphocytes (plasma cells) in response to a specific antigen.

Both antigen-binding sites are identical and consist of a long


and heavy chain. IgGs constitute approximately 75% of all
immunoglobulis in a healthy individual. Only IgGs can cross
the placenta. This is important as they provide immune
protection for the newborn in the first few months of life.
64. Disinfectant solutions may become contaminated with:
A- Enterobacter
species ...................................................................T
B- Streptococcus
species .................................................................F
C- Escherichia
coli .........................................................................F
D- Pseudomonas aeruginosa
(pyocyanea) .......................................T
E- Staphylococcus
pyogenes ...........................................................F

Disinfectant solutions have variable bactericidal and


fungicidal properties but are usually but there are reports of
contamination with Pseudomonas spp, Enterobacter, Serratia,
actinomyces and Fungi.

65. Use of the combined oral contraceptive pill is associasted


with an increased risk of:
A- Ovarian
carcinoma .....................................................................F
B- Breast
Carcinoma ......................................................................T
C- Endometrial
carcinoma ..............................................................F
D- Hepatic
carcinoma ......................................................................T
E- Cervical
carcinoma ....................................................................T

The risks of some cancers are associated with use of the OCP.
In particular, it appears that breast cancer risk is increased
and, use of the OCP for more than 5 years is associated with
increased risk of cervical carcinoma. Hepatoma risk may also
be increased with use of the OCP. Use may be protective for
ovarian carcinoma.
66.Unopposed oestrogen therapy for the postmenopausal

female
A- Improves the urethral
syndrome ...................................................T
B- Decreases urinary calcium
excretion .............................................T
C- Reduces the incidence of myocardial
infarction ..............................F
D- Causes an increased incidence of endometrial
carcinoma ..............T
E- Cause
hypertension ....................................................................F
Low levels of natural oestrogen around and after menopause
diminish the body's ability to absorb calcium and to
metabolize vitamin D. Low oestrogen levels lead to a thinning
of trabecular bone and eventually osteoporosis. This leads to
an increased risk of fractures of the hip and wrist and
compression fractures of the vertebrae resulting in a dowager
hump. Rather than the original belief that postmenopausal
HRT reduces CV risk, studies like WHI and HERS show an
increased CV mortality and morbidity associated with HRT.
Neoplasia of the endometrium may follow unopposed
oestrogen; the risk increases with the duration of use:x 3-6
after five years of use. x 10 after ten years. That is why

unopposed oestrogens are no longer used in subjects with an


intact uterus.
67. The following are precursors of oestradiol
ACholesterol ................................................................................
..T
BStilboestrol ................................................................................
..F
C- Arachidonic
acid ...........................................................................F
DDehydroepiandrosterone ...........................................................
..T
ETestosterone ...............................................................................
.T
The pathways whereby oestrogens are synthesized in the
ovary
Cholesterol to Pregnenolone then via D4 pathway:
Progesterone to 17-hydroxyprogesterone (17-OHP) to
Androstenedione (D4) Testosterone to Oestradiol-17b (E2)

or via D5 pathway: 17-hydroxypregnenolone to


Dehydroepiandrosterone (DHA) to Oestrone (E1)
68. Prostaglandins are
A- Synthesized from
cholesterol ......................................................F
B- Are small
polypeptides ...............................................................F
C- Secreted by the pituitary
gland ...................................................F
D- Secreted by the prostate
gland ...................................................T
E- Associated with gastrointestinal side
effects ..................................T

Prostaglandins are long-chain hydroxy fatty acids derived


from arachidonic acid, which is released from cell membrane
phospholipids and catalysed by the enzymes cyclo-oxygenase
and endoperoxidase. Prostaglandins are produced locally. The
original source was the prostate. GI side effects include
diarrhoea and abdominal pains.
69. The Barr body

A- arises from the Y


chromosome .................................................F
B- is present in subjects with Turner's
syndrome ...........................F
C- is present in 20% of cells in the
male ........................................F
D- appears as a dense rod close to the cell
wall .............................F
E- is present in males with Klinefelter's
syndrome .............................T
The chromatin nodule is the second (inactive) X chromosome.
Normal women are thromatin positive. Normal men are
chromatin negative. In polymorphonuclear leucocytes, a tiny
nodule of chromatin (the Barr body, or nuclear drumstick) can
be seen near the nuclear membrane of many cells in normal
females, but not in normal males. Turner's XO does not have a
Barr body. In Klinefelter's syndrome the sex chromosomal
structure is XXY, and in them a chromatin nodule is seen.
70. Androgens:
A- Exert a feedback inhibitory effect on FSH
production .....................T

B- Produce a release of LH from the


ovary ........................................F
C- Are produced in the
ovary ..........................................................T
D- Are secreted by the female adrenal
cortex ...................................T
E- Are formed in the Leydig cells of the
testis ....................................F

Testosterone, like oestradiol, feeds back at the hypothalamus /


pituitary to inhibit GnRH secretion and switch off both LH
and FSH secretion. In the female, testosterone is synthesied in
small amounts, probably in the adrenals, but a weak androgen,
androstenedione is formed as a step in the metabolism of
progesterone. LH stimulates testosterone production from
Leydig cells of the testis.
71. IgA:
A- Is involved in mucosal
immunity....................................................T
B- Has 4 distinct sub-

groups...............................................................F
C- Activates complement via the classical
pathway...............................F
D- Is manufactured in lymph
nodes....................................................F
E- Crosses the
placenta. ...................................................................F

IgA is the major antibody produced by plasma cells near


mucosal surfaces, and is found in tears, sweat, lung, gut,
urine. IgA avoids digestion by the presence of the secretory
piece which is added as it is secreted onto the mucosa. It is an
important defence against surface binding of microorganisms. There are 2 sub-types, IgA1 and IgA2. Only IgG is
transferred across the placenta. Complement proteins
circulate, therefore, they rarely come into contact with IgA.
72. Circulating anticoagulants have been described in
A- Systemic Lupus
Erythematosis ......................................................T
B- postpartum ................................................................................T

Chomocystinuria ..........................................................................
F
Dhaemophilia ...............................................................................
T
E- the
elderly ..................................................................................T
Circulating anticoagulant, usually IgG, interfere with
coagulation reactions. The main laboratory feature are
prolonged PT and PTT which persists if normal plasma is
added.
a-These are non-specific inhibitors which prolong haemostasis
by binding to phospholipids. Anti-factor VIII antibodies may
also be seen.
b,d and e-Anti-factor VIII antibodies.
c-Thrombotic tendency.
d-haemophiliacs who have had plasma transfusions.
73. A karyotype
A- may be prepared from chorionic villi cells of the
placenta................T
B- helps in the diagnosis of chromosome
disorders.............................T

C- is made from cells arrested at telophase of


mitosis.........................F
D- from a Patau syndrome patient shows an extra
chromosome No. 18.....F
E- helps in the identification of the Philadelphia chromosome,
in chronic myeloid leukaemia......T

Karyotype is the chromosomal composition of cells- normal


karyotype is 46XX or XY. Patau syndrome is associated with
trisomy of Chromosome 13.
74. Which of the following organelles have their own self
replicating DNA?
Alysosomes ..................................................................................
.F
B- Golgi
body ..................................................................................F
Cmitochondria .............................................................................
.T

Dnucleolus ...................................................................................
.F
E- rough
ER ....................................................................................F

Mitochondria are well recognised to contain DNA and give


rise to maternally inherited diseases such as the mitochondrial
myopathies - red ragged fibres - DIDMOAD syndrome,
Wolfram disease
74. Immunoglobulin G (IgG)
A- constitutes the natural haemagglutinins (anti-A and antiB) ............F
B- is the predominant immunoglobulin in normal bronchial
secretions ....F
C- freely crosses the
placenta ..........................................................T
D- has a molecular weight of 350,000
daltons ...................................F
E- bears only 2 antigen-binding

sites ................................................T
a) IgM,
b) IgA,
c) the only Ig to do so,
d) 150,000
75. Which of the following statements regarding DNA is/are
true/false ?
A- Attached to the 2' position of the sugar ring is one of four
bases.......F
B- The bases lie stacked on each other 3.4 Angstroms
apart................T
C- During transcription each daughter DNA contains one
newly synthesised strand.........F
D- The guanine - cytosine bonds are made of two hydrogen
bonds. .......F
E- The DNA from a single cell is nearly 2 metres
long..........................T
Individual nucleoside units (bases) are joined together in a
nucleic acid in a linear manner, through phosphate groups
attached to the 3' and 5' positions of the sugar (deoxyribose).
Hence, the full repeating unit in a nucleic acid is a 3',5'-

nucleotide.
Guanine - cytosine bonds consist of three hydrogen bonds.
--------------------------------------------------------Q) Are the following true or false regarding the developmwnt
of the urinary system?
( True / False)
A) Bowman's capsule develops in the metanephros..T
B) Develops from intermediate mesoderm..T
C) In intra uterine life 3 overlapping kidney systems are
formedF
D) The glomerulus forms part of the mesonephrosT
E) The mesonephros develops as the metanephros
regressesF

Q)

--------------------------------------------------Q) 1.
A cone biopsy specimen of a woman who
requested that her menstrual function be preserved

shows a carcinoma in situ with complete excision. She


should be advised to

a.

Be discharged from follow up.

b. Continue to attend yearly follow up for Pap smear


tests.
c.

Requires hysterectomy.

d.

Requires Wertheims hysterectomy.

e.

Referred for prophylactic RT.

2.

In a pregnant lady with diabetes mellitus

a. Insulin requirement decreases because of the fetal


pancreas.
b.

Fetal mortality occurs in early pregnancy.

c.

Should be monitored by labstix.

d.

All of the above.

e.

None of the above.

E.

3. A positive fern test of the cervical mucus in the early


part of the menstrual cycle followed by a negative fern
test premenstrually means

a.

Estrogen is present.

b.

Progestrogen is present.

c.

Ovulation has occurred.

d.

All of the above.

e.

None of the above.

C.

4. Cervical carcinoma is best treated by Wertheims


operation

a. In the presence of 1st trimester intrauterine


pregnancy.

b.

In the presence of PID.

c.

In early stage II.

d.

All of the above.

e.

None of the above.

C.

5.

Seminar fluid

a.

pH is 4.6

b.

Volume is 10ml.

c.

Contains PGE2.

d.

Produced mainly by seminiferous tubules.

e.

None of the above.

C.

6.

In NUH basic investigation for subfertile couples

a.

Analysis of seminal fluid.

b.

Hormonal analysis of the male.

c.

Post coitus test.

d.

All of the above.

e.

None of the above.

A.

7. What is not the advantage of early booking in


pregnancy?

a.

Gestational age assessment.

b.

Congenital abnormalities detected.

c.

Early abortion when pregnancy is contraindicated.

d.

IUGR in the 1st trimester detected.

e.

Pelvic pathology in pregnancy detected and treated.

D.

8. In a gynecological examination, a per-rectal is done


in:

a.

Carcinoma of the cervix.

b.

Endometriosis.

c.

Unmarried females.

d.

All of the above.

e.

None of the above.

D.

9. Which one does not help to indicate whether the


pelvis is adequate for delivery?

a.

Birth weight of last baby.

b.

Duration of previous labor.

c.

Length of last child.

d.

Method of delivery of previous labor.

e.

Indications for previous operative deliveries.

C.

10. The clitoris doesnt contain

a.

Erectile tissue.

b.

Corpus spongiosum.

c.

Involuntary muscles.

d.

A glans.

e.

2 crura.

B.

11. Which of the following isnt a reason for doing a


vaginal examination in the 1st trimester of pregnancy?

a.

To exclude pelvic tumor.

b.

To assess size of uterus.

c.

To assess pelvic size.

d.

To assess fetal viability.

12. Definite signs of labor

a.

Leakage of aminotic fluid.

b.

Show.

c.

Painful contractions.

d.

Painful contractions with cervical changes.

e.

Effaced cervix with 3cm dilatation.

D.

13. When a women comes for labor

a.

Oxytocin drip is given at 1st stage labor.

b.

Oxytocin drip is given at 2nd stage labor.

c.

Oxytocin drip is given at 3rd stage labor.

d.

Drip without oxytocin given at 1st stage.

e.

None of the above.

E.

14. In normal delivery

a.

Optimal cord cutting time is 30 60 seconds.

b.

Optimal residual cord length is 6 10cm.

c. Should milk as much of the cord blood into the


baby.
d.

All of the above.

e.

None of the above.

15. Moderate uterine bleeding associated with ovulation


is

a.

The constitutional variant of the individual.

b.

Usually seen in young women.

c. Associated with a reduction in circulating estrogen


levels.
d.

All of the above.

e.

None of the above.

D.

16. What is not important in menstrual history?

a.

1st day of LMP.

b.

Duration.

c.

Pain.

d.

Color.

e.

Passage of clots.

D.

17. Which is not found in asymmetrical growth


retardation?

a.

Hypoxia.

b.

Oligohydramnios.

c.

Normal H/A ratio.

d.

Abnormal H/A ratio.

18. In severe pre-eclampsia

a.

The plasma volume is reduced.

b.

The extracellular fluid is increased.

c.

There is placental insufficiency.

d.

All of the above.

e.

None of the above.

D.

19. In polycystic ovarian syndrome, the following is not


found in

a.

Wide carrying angle.

b.

Hirsuitism.

c.

Obesity.

d.

Raised LH.

e.

None of the above.

A.

20. Amenorrhoea

a.

Is a feature of anterior pituitary destruction.

b. May occur with continuous administration of


estrogen and progesterone.

c.

May occur with anorexia nervosa.

d.

Is always present in Turners syndrome.

e.

All of the above.

21 Eclampsia management

a. Control convulsions and hypertension and deliver


immediately regardless of fetal maturity.
b. Intubate patient to ensure airway is not obstructed
and deliver by LSCS.
c.

Delivery depends on urinary output.

d. If fetus is premature, control fits and hypertension


and wait.
e.

A.

None of the above.

22. What is the least likely presentation of twins?

a.

Cephalic-cephalic.

b.

Breech-cephalic.

c.

Transverse-transverse.

d.

Breech-breech.

e.

Cephalic-transverse.

C.

23. Which of the following is not associated with twin


pregnancy?

a.

Polyhydramnios.

b.

Anemia.

c.

Premature labor.

d.

Urinary tract infections.

e.

All are associated with twin pregnancy.

E.

24. What (if any) are the contraindications of induction


of labor?

a.

Cardiac disease in pregnancy.

b.

Breech presentation.

c.

Short stature (< 150cm).

d.

All of the above.

e.

None of the above.

B.

25. ARM is mediated through

a.

Prostaglandin.

b.

Leukotrienes.

c.

Prostacycline.

d.

Thromboxane.

e.

None of the above.

A.

26. Pyepagus is a state of conjoined twins whereby


there is joining of the

a.

Heads.

b.

Thoraces.

c.

Abdomen.

d.

Limbs.

e.

Back and buttocks.

27. Incidence of anemia is pregnancy is reduced by

a.

Spacing of children.

b.

Give iron supplements.

c.

Give oral contraceptives.

d.

Better nutrition.

e.

All of the above.

E.

28. Maternal mortality in Singapore (1977 1979)

a.

2.5 per 100.

b.

7.2 per 10 000.

c.

1.4 per 100,000.

d.

10 per 100,000.

29. Choriocarcinoma presentation

a.

Purplish spots in the vagina.

b.

Uterine enlargement.

c.

Irregular vaginal bleeding.

d.

Hemoptysis.

e.

All of the above.

E.

30. Partial mole

a.

Commonly associated with CNS malformation.

b.

< 10% of pregnancy.

c.

Commonly associated with severe pre-eclampsia.

d.

All of the above.

e.

None of the above.

31. Dysmaturity are infants who are

a.

Post term 42/52.

b.

Infant of diabetic mother.

c.

SGA.

d.

All of the above.

e.

None of the above.

A.

33. Most common etiologic factor for heart disease in


pregnancy

a.

Syphilis.

b.

Congenital heart disease.

c.

Rheumatic heart disease.

d.

Ischaemic heart disease.

e.

None of the above.

34. What is not seen in buffy coat film in megaloblastic


anaemia?

a.

Hypersegmentation of PMN.

b.

Macropolycytes.

c.

Normoblasts showing abnormal hemoglobinism.

d.

Howell Jolly bodies in RBC.

e.

All the above is seen.

D.

35. What is not routinely done in KKH for a patient


whose hemoglobin is 8.2g%?

a.

Stool for ova and cysts.

b.

PBF.

c.

Erythrocyte electrophoresis.

d.

Bone marrow film.

e.

Serum iron and TIBC.

36. Normal pelvic dimensions are

a.

Anteroposterior 13cm.

b.

Transverse 11.5cm.

c.

Interspinous 10.5cm.

d.

All of the above.

e.

None of the above.

D.

37. Physiological changes in pregnancy are

a.

Increased gastric secretion.

b.

Increased small intestine mortality.

c.

Relaxation of the gastro-esophageal sphincter.

d.

All of the above.

e.

None of the above.

C.

38. Which is untrue?

a. Overlapping of fetal skull before onset of labor


supports the diagnosis of intrauterine death.
b.

Buddha position in X-ray suggests hydrops fetalis.

c. Women thought to have multiple pregnancies need


X-ray or ultrasound scan.
d.

Renal agenesis is associated with polyhydramnios.

e.

Fetal skeleton is visible on X-ray by 20th week.

D.

39. The following conditions should be present before


application of forceps except

a.

Must have no cephalopelvic disproportion.

b.

Head at pelvic brim.

c.

Head in occipito-anterior.

d.

Cervix fully dilated.

e.

Bladder and rectum emptied.

40. Relevant investigations in young primigravida with


severe hypertension at 34 weeks

a.

Renal creatinine clearance.

b.

PBF.

c.

Optic fundoscopy.

d.

UFEME.

e.

All of the above.

41. A 20-year-old primigravida in established labor for


24 hours. Labor is slow, cervix remains half dilated over
6 hours, no cephalopelvic disproportion. Patient has
severe sacral pain and some tenderness over lower
segment. Resting tone of uterus is high. She is likely to
be having

a.

Hypotonic uterine inertia.

b.

Hypertonic uterine inertia.

c.

Obstructed labor.

d.

All of the above.

e.

None of the above.

B.

42. Following treatment should be given for the above


condition

a.

Intravenous dextrose.

b.

Intravenous oxytocin.

c.

Delivery by vacuum.

d.

LSCS.

e.

Controlled anaesthesia.

D.

43. Secondary amenorrhoea is not associated with

a.

Oral contraceptives.

b.

Sheehans syndrome.

c.

Genital tuberculosis.

d.

Hypothyroidism.

e.

Ginari-rommech syndrome.

E.

44. Which of the following statements about


choriocarcinoma is not true?

a.

Always preceded by mole.

b.

Urinary gonadotrophin raised.

c.

Cytotoxic drugs give increased survival.

d.

Secondary deposits in the liver.

e.

May be heralded by onset of hemoptysis.

A
45. Which is usually not associated with failure to
conceive? a. Cervical incompetence.
b.

Sperm count of < 5 million / ml.

c.

Tuberculosis salpingitis.

d.

Adrenogenital syndrome.

e.

Stein-Leventhal syndrome.

A.
46. Which statement is wrong?
a.

Infertility may be associated with acute gonorrhoea.

b. Cryptomenorrhoea may be associated with acute


retention of urine.

c. Negative pregnancy test always excludes ruptured


ectopic pregnancy.
d. Presence of chorionic villi in curetting exclude
ectopic pregnancy.
e. Sterilization by tubal ligation is regarded as
irreversible.

B.

47. Which statement about genital prolapse is wrong?


a. Transverse ligaments take part in supporting
vaginal vault.
b.

Post-natal exercises are of no value.

c.

Cervix is often elongated.

d.

Rectocoele may cause intestinal colic.

e. Overflow incontinence is a common syndrome of


urethrocoele.
B
48. Which of the following statements about carcinoma
of the cervix is untrue?
a.

Anaemia is a common terminal event.

b.

Risk of carcinoma cervix is related to parity.

c.

Post-coital bleeding is a common symptom.

d.

Histologically usually adenocarcinoma.

e.

Common site for mets are obturator nodes.

D.

49. Which of the coupled signs and diseases are


wrong? a.
Ascites and ovarian carcinoma.
b. Thickened uterosacral ligament and carcinoma of
the corpus uteri.
c.

Discrete white patches on vagina and monilia.

d.

Enlarged inguinal nodes and Bartholins abscess.

e.

Thickened uterosacral ligament and endometriosis.

B.

50. Congenital abnormalities of the uterus are


associated with the following except

a.

Habitual abortion.

b.

Normal reproductive ability.

c.

Renal abnormalities.

d.

Polycystic kidneys.

e.

Cryptomenorrhoea.

D.
51. Right ureter lies in relation to the following:
a.

Bifurcation of common iliac vessels.

b.

Infundibular pelvic ligaments.

c.

Inferior mesenteric artery.

d.

Uterine artery.

e.

Paracervical tissue.

52. Young girls may get vaginal discharge due to the


following except

a.

Monilia.

b.

Ovarian dysgerminoma.

c.

Gonorrhoea.

d.

Ectopic ureters.

e.

Sarcoma.

B.

53. 50-year-old woman having irregular periods is first


managed by

a.

Hormones.

b.

Oral Fe and ergometrine.

c.

D & C.

d.

Hysterectomy.

e.

Inducing menopause with radium.

C.

54. Absolute contraindication to use of oral


contraceptives

a.

Cervical erosions.

b.

Carcinoma of breast.

c.

Cholecystitis.

d.

Hyperemesis gravidarum.

e.

PTB.

B.

56. Post menopausal bleeding is caused by the


following except

a.

Cervical prolapse.

b.

Senile vaginitis.

c.

Ectopic pregnancy.

d.

Stilboesterol.

e.

Endometrial carcinoma.

C.

58. APGAR score assess the state of the following


systems in the newborn except

a.

Respiration.

b.

Locomotor.

c.

Cardiovascular system.

d.

Central nervous system.

e.

All.

59. Witches milk results from

a.

Prolonged breast feeding.

b. Excessive endogenous stimulation in female infants


by estrogen.
c. Increased levels of placental hormones in fetal
plasma.
d.

Abnormal lactation.

60. Respiratory quotient in a newborn is

a.

1.0.

b.

0.7.

c.

1.2.

d.

0.8.

61. Meconium is composed of the following except

a.

Mucopolysaccharides.

b.

Vernix.

c.

Upper respiratory epithelium.

d.

Bilirubin.

e.

Mucoprotein.

62. Labor is said to be established when contractions

a.

Are painful.

b.

Occur 1 in 30 minutes.

c.

Occur 1 in 5 minutes.

d.

There is a show and leaking liquor.

C.

63. Tubal insufflation is contraindicated under the


following conditions

a.

Just before mid-cycle.

b.

At onset of menstruation.

c.

Day after coitus.

d.

Gross chronic cervicitis.

64. Which of the following sperm counts are abnormal?

a.

250 million / ml.

b.

60 million / ml.

c.

20 million / ml.

d.

1 million / ml.

65. Endometrial carcinoma stage 1 must be treated by

a.

Increased dosage of progesterone therapy.

b.

Cytotoxic drugs.

c.

Total hysterectomy.

d.

Cobalt therapy.

e.

Combined radiotherapy and surgery.

C.

66. Immunotherapy must be employed in

a.

Treatment of Rh immunized pregnancy.

b.

Prevention of Rh isoimmunization.

c.

Treatment of Barts disease.

d.

As an adjunct to chemotherapy in choriocarcinoma.

B.

67. The ovaries may not be conserved during


hysterectomy under the following circumstances

a.

> 38 years.

b.

For pelvic endometriosis.

c.

For persistent PID.

d.

Extensive fibroids.

B.

68. In the normal menstrual cycle, estrogen is produced


from the

a.

Ovum.

b.

Ovarian stroma.

c.

Graafian follicle.

d.

Endometrium.

e.

Corpus luteum.

C.

69. Complications of IUCD include

a.

Placenta praevia.

b.

Perforation of the uterus.

c.

PID.

d.

Chronic cervicitis.

e.

Thrombophlebitis.

B.

70. Bleeding may be caused by vaginal examination


when there is

a.

Fibroid prolapse.

b.

Cervical carcinoma.

c.

Adenomyosis.

d.

Urethral carbuncle.

e.

Ovarian carcinoma.

B.

71. Young girls may develop vaginal discharge due to

a.

Moniliasis.

b.

Presence of foreign body.

c.

Threadworms.

d.

Ovarian dysgerminoma.

e.

Ectopic pregnancy.

D.

72. Secondary dysmenorrhoea

a. Means painful menstruation in those who have had


at least 1 child.
b.

May precede menstrual period.

c.

Is relieved by antispasmodics.

d.

Usually associated with organic disease.

e.

Is often improved by dilatation of the cervix.

B.

73. Heavy irregular periods in a female aged 52 may be


correctly managed by

a.

Hormonal therapy.

b.

Taking a cervical smear.

c.

Inducing the menopause with radium.

d.

Administration of oral iron and ergometrine.

e.

D&C.

E.

74. Most common cause of early abortions

a.

Physical trauma.

b.

Uterine retroversion.

c.

Abnormality of conceptus.

d.

Incompetent cervix.

C.

75. Ovarian artery runs through

a.

Parametrium.

b.

Ovarian ligaments.

c.

Infudibulopelvic ligament.

d.

Round ligament.

e.

Urachus.

C.

76. Turners syndrome is characterized by all except

a.

Webbed neck.

b.

Short stature.

c.

Primary amenorrhoea.

d.

Chromosome haplotype XO.

e.

Vaginal atresia.

77. Hydatiform mole have all but the following

a.

Uterine size > period of amenorrhoea.

b.

Threatened abortion.

c.

Produce HCG.

d.

Usually have a fetus.

78. Hydatiform mole have all but the following

a.

Uterine size > period of amenorrhoea.

b.

Features of missed abortion.

c.

Produce normal HCG titres.

d.

Irregular vaginal bleeding.

e.

Theca lutein cyst of ovaries.

C.

79. Which of the following is commonly associated with


cervical carcinoma?

a.

Dyspareunia.

b.

Dysmenorrhoea.

c.

Post coital bleeding.

d.

GIT symptoms.

e.

Prolapse.

C.

80. Which of the following is not associated with failure


to conceive

a.

Cervix incompetence.

b.

Sperm count of 10 million / ml.

c.

TB salpingitis.

d.

Fibroid.

e.

Bicornuate uterus.

A.

81. Complications of abdominal hysterectomy with


conservation of ovaries may be any of the following
except:

a.

Apareunia.

b.

Vaginal discharge.

c.

Excessive weight gain.

d.

Hirsutism.

e.

Urinary incontinence.

D.

82. Post-menopausal bleeding is not caused by:

a.

Cervical polyp.

b.

Ectopic pregnancy.

c.

Cervical carcinoma.

d.

Endometrium carcinoma.

e.

Senile vaginitis.

B.

83. Cystic swelling of the anterior vaginal wall may be


due to the following except:

a.

Ganters duct cyst.

b.

Urethrocoele.

c.

Urethral diverticulae.

d.

Ureterocoele.

e.

Cystocoele.

D.

84. Stress incontinence is characterized by:

a.

Loss of posterior urethro-vesical angle.

b.

Recurrent urinary tract infection.

c.

Utero-vaginal prolapse.

d.

All of the above.

e.

None of the above.

85. Method of preventing conception must consider:

a.

Time of ovulation.

b.

Availability of unfertilized ovum.

c.

Viability of sperm.

d.

All of the above.

e.

None of the above.

D.

86. ECV is contraindicated when all of the following


conditions are present except:

a.

APH.

b.

Hypertension.

c.

Grossly contracted pelvis.

d.

Previous LSCS scar.

e.

Polyhydramnios / oligohydramnios.

B.

87. Recognized complications of term pregnancy


include:

a.

Malpresentation.

b.

Anemia.

c.

Premature labor.

d.

Diabetes mellitus.

e.

Hydramnios.

C.

88. In physiological anemia of pregnancy, there is:

a.

Fall in total RBC mass.

b.

Increase in blood volume.

c.

Fall in the serum iron levels.

d.

Increased plasma volume.

e.

Fall in hemoglobin volume.

D.

89. Placental insufficiency can be suspected in all of the


following except:

a.

Falling estriol level.

b.

Fetal movements are not vigorous.

c.

Amnioscopy shows clear liquor.

d.

Uterus less than dates.

e.

History of bleeding in early pregnancy.

C.

90. Pruritus vulvae is a feature of the following except:

a.

Diabetes mellitus.

b.

Varicose veins of the vulvae.

c.

Round worm infestation of the gut.

d.

Leukoplakia vulvae.

e.

Vulval carcinoma.

C.

91. Second stage of labor begins when:

a.

Cervix is effaced.

b.

Membranes ruptured.

c.

Caput distending the perineum.

d.

Cervix is fully dilated.

e. Uterine cavity and vagina form a continuous


cylindrical tube.

D.

92. Management of patient in labor with fetus in


transverse lie:

a.

Caesarean section if cervix is not fully dilated.

b. Uterine version and breech extraction if cervix is


half dilated.
c.

Anticipated spontaneous delivery.

d.

Rupture membrane, set oxytocin drip.

e.

Perform decapitation operation if fetus is absent.

A.

93. Recurrent early trimester abortion due to


incompetent os is best treated by:

a.

Cervical suture.

b.

Progesterone therapy.

c.

Stilbesterol.

d.

Diazepam.

e.

Isoxupurine.

A.

94. Krukenburg tumor is ovarian tumor which:

a.

Is primary to the ovary.

b.

Secondary to any gastrointestinal tract cancer.

c.

Shows characteristic mucinous epithelial changes.

d.

In association with ascites.

e.

None of the above.

B.

95. Ovarian blood supply comes principally from:

a.

Uterine artery.

b.

Ovarian artery.

c.

Long artery of Hishaelis.

d.

Internal iliac artery.

e.

Common iliac artery.

B.

96. OC pill is contraindicated in:

a.

Ca breast.

b.

Past history of jaundice.

c.

Past history of thrombosis.

d.

All of the above.

e.

None of the above.

D.

97. Imperforate hymen may present with:

a.

Acute retention of urine.

b.

Primary amenorrhoea.

c.

Suprapubic pain.

d.

All of the above.

e.

None of the above.

D.

98. Treatment of choice for Bartholin cyst:

a.

Incision and drainage.

b.

Excision of gland.

c.

Marsupialisation.

d.

Antibiotics.

e.

Glycerol magnesium sulphate.

C.

99. All are true of missed abortions except:

a.

Clotting defect sets in after missed abortion.

b.

Requires urgent evacuation of uterus.

c.

Uterus usually greater than period of amenorhroea.

d.

Pregnancy test is negative.

e.

Risk of infection is higher in missed abortions.

C.

100.Fetal distress is characterized by:

a. Fetal heart beat type II dips and scalp blood pH


7.15.
b.

Passage of meconium stained liquor.

c.

Fetal bradycardia or tachycardia.

d.

None of the above.

e.

All of the above.

E.

101.Commonest post-op compliactions of LSCS in:

a.

Deep vein thrombosis.

b.

Pneumonia.

c.

Urinary tract infections.

d.

Rupture of scar.

e.

Haemorrhage.

E.

102.The smallest diameter of fetal skull is:

a.

BPD.

b.

Bregma diameter.

c.

Occipitomental diameter.

d.

Suboccipitofrontal diameter.

e.

Subparietal diameter.

A.

103.Iron deficiency anemia in pregnancy is


characterized by all except:

a.

Microcytic hypochromic RBC.

b.

Serum folate iron is 50 ug/100ml.

c.

Hypersegmented nucleus in white blood cell.

d.

TIBC of 500 up/100ml.

e.

Increased incidence of multiple pregnancy.

C.

105.The following radiological findings indicate fetal


death except:

a.

Haloes area on fetal skull.

b.

Absence of femoral epiphysis.

c.

Spalding sign.

d.

Gas bubbles in heart vessels.

e.

Ball sign.

E.
106.The normal sequelae of events in acute PID is:
a. Abscess formation.
b.

Hydrosalpinx.

c.

Atrophy of ovary.

d.

Pseudocyst formation.

e.

Resolution.

107.What is the method of choice in the termination of


16 week pregnancy? a.
Hysterectomy.
b.

Vacuum aspiration followed by uterine curretage.

c.

Intra-amniotic prostaglandins.

d.

Buccal oxytocin.

C.
108.In a 54 year old woman with vaginal prolapse, the
treatment of choice is:
a.

Vaginal hysterectomy with pelvic floor repair.

b.

Perineal exercises.

c.

Leforts operation.

d.

Ring pessary.

A.
110. Prophylactic forceps delivery should be ruled out in:
a.

Cardiac disease.

b.

Previous caesarean section.

c.

Toxaemia of pregnancy.

d.

None of the above.

e.

All of the above.

D.
-------------------------------------------------------------Q) The following radiological findings indicate fetal death
except:a. Haloes area on fetal skull.
b. Absence of femoral epiphysis.
c. Spalding sign.
d. Gas bubbles in heart vessels.
e. Ball signans
Q) consequence of maternal use of cocaine is ?
a. hydrops
b. sacral agenesis
c. cerebral infarction.ans
d. hypertrichosis

Q) In reducing the risk of blood transfusion in pregnancy and


labour which of the following strategies is recommended?
A Hb of less than 10.5g/l should prompt haematinics and
exclusion of haemaglobinaopathies..ans
Cell salvage is recommended for all women undergoing csection with a Hb of 10.5g/l or less
Anaemia not due to haematinic deficiency should be managed
with parenteral iron
Recombinant human erythropoietin (rHuEPO) should be used
for anaemia correction where oral iron isn't tolerated and
parenteral iron is unacceptable to the patient.
Pre-autologous blood deposit taken and stored for later use

Q) As regard to hormone replacement therapy(HRT)


testosterone has no role in HRT....false as it increases libido
Tibolone is an artificial form of estradiol...false Tibolone is a
synthetic steroid which is chemically inert untill its absorbed
where it becomes active
Q) What are the two major steroid hormones produced by
corpus luteum?
Estradiol and hcg
Estradiol and progesterone.ans
Estradiol and testosterone
Progesterone and hcg
Progesterone and testosterone
Q) Dilatation of the uterine cervix at parturition is primarily
achieved by:

Muscle relaxation under the influence of oestriol


Prostaglandin E1 mediated Interleukin release
Constriction of cervical small vessels
Macrophage mediated collagen degradation
Degradation of type I collagen by interstitial
collagenaseans
Cervical ripening refers to the softening of the
cervix prior to labour. It is a complex process that
involves the following processes (the key point is
type 1 collagen breakdown by collegenase):
Increased activity of metalloproteinases 2 & 9
that degrade extracellular matrix proteins.
Cervical collagenase and elastase also increase
and degradation of collagen increases, leading to
decreased collagen content in the cervix.
Increased oestrogen leads to increased
collagenase activity - Increase COX2 causing
increased prostaglandin E2 (PGE2) in the cervix
PGE2 leads to:
Increase in collagen degradation
Increase in hyaluronic acid
Increase in chemotaxis for leukocytes, which
causes increased collagen degradation
Increase in stimulation of interleukin (IL)8 release
Prostaglandin F2-alpha is also involved in the
process via its ability to stimulate an increase in
glycosaminoglycans.
Q) Regarding blood volume in pregnancy which of the
following statements is TRUE?

Blood volume remains constant


Blood Volume increases by approximately 5%
Blood Volume increases by approximately 10-15%
Blood Volume increases by approximately 20-25%
Blood Volume slowly increases by 40-50%..............ans
Q) Regarding the use of Erythropoietin
(recombinant human erythropoietin AKA rHuEPO) in
pregnancy which of the following statements is correct?
rHuEPO has been shown to be have adverse maternal effects
when used antenatally
rHuEPO has been shown to be have adverse fetal effects when
used antenatally
rHuEPO has been shown to be have adverse neonatal effects
when used antenatally
The use of rHuEPO is only recommended for non-end-stage
renal anaemia
None of the above..ans
The Green Top Guidelines state the following:
Recombinant human erythropoietin (rHuEPO) is mostly used
in the anaemia of end-stage renal disease. rHuEPO has been
used both antenatally and postpartum in women without endstage renal disease without any adverse maternal, fetal or
neonatal effects.
Use rHuEPO in clinical practice for non-end-stage renal
anaemia is still to be established and should only be used in
the context of a controlled clinical trial.

Important Points on Erythropoietin


Currently should be used in end-stage renal anaemia only
No evidence to suggest harmful to mother, foetus or neonate
However, It is NOT established for use in non-end-stage renal
anaemia and therefore shouldn't be used in these patients
outside of a controlled trial
Q) Upregulation of urea movement through urea transport
proteins in the collecting duct is an effect of which of the
following Angiotensin
Angiotensin II
Aldosterone
Renin
Vasopressinans
Q) A 32 years old woman is admitted to postnatal ward 10
days after emergency cesarean section with a painful swollen
calf. Her observations are stable. Her BMI is 37. You want to
rule out a deep vein thrombosis. Which of the following
clotting factors are increased in normal pregnancy? A. Factor
VII
B. VII, VIII
C. VII, VIII,Xans
D. VII, VIII, X , XI
E. VII, VIII, X ,XI , XIII
Q) What is the typical oxygen consumption in a 75kg nonpregnant women?

5ml/min
15ml/min
50ml/min
150ml/min
250ml/minans
The typical Oxygen Consumption (VO2) is 250ml/min.
In pregnancy this increases by around 20% to 300ml/min
Q) Contraindications UKMEC 3/4 to use depot for
contraception??
Treated Breast Ca with no recurrence after 10 yearsans
Age 37 smoking 3-5 ciger per day
Personal history of DVT
BMI 45 kg/m2
12 weeks post partam breastfeeding
Q) antimalarial drug to be avoided in pregnancy ?
a. chloroquine
b. quinine
c. primaquine..ans
d. anti-folates
e.tetracyclines
Q) What's the earliest gestational age at which the fetal
endocrine system is thiught to be fully functional? 6weeks
10weeksans
16weeks
24weeks
30weeks

Q) Which structure is the primary mechanism for shunting


blood away from the fetal pulmonary circulation?
Foramen Ovale.ans
Ductus Arteriosis
Arteria Umbilicalis
Ductus Venosus
Umbilical Arteries
Blood enters the right atrium of the fetal heart
and most passes through the foramen ovale into
the left atrium. From there it is pumped through
the aorta. The foramen ovale is the major
structure for bypassing the fetal pulmonary
circulation.

Q) A 52 year old patient attends clinic for the results of her


bone density scan. Her T-score is -2.2. She tells you she has
previously fractured her 5th metatarsal and patella following a
fall from a bike. You diagnose her as: Normal bone density
Osteopenic.ans
Severely Osteopenic
Osteoporosis
Severely Osteoporoti

The fracture history here is irrelevant especially


considering neither are fragility fractures.
T-score 0 to -1 Normal bone density
T-score -1 to -2.5 Osteopenia
T-score -2.5 or less Osteoporosis
Q) During wound healing collagen alignment along tension
lines is part of which phase?
Haemostasis
Inflammation
Granulation
Proliferation
Remodelling..ans
Realignmnet of collagen is part of the remodeling
phase. Remodeling is usually underway by week
3. Maximum tensile wound strength is typically
achieved by week 12.
Q) a pregnant mother is treated with oral anticoagulant . the
likely congenital malformation that may result in the fetus is ?
a. long bones limb defect
b. cranial malformation
c. cardiovascular malformation
d. chondrodysplasia punctate..ans
Q) HPV proteins associated with ca. risk? a. E2 and E3
b. E4 E5
c. E6 E7..ans
d. E1 E2

Q) 99% of body calcium is in what form? Calcium


Bicarbonate
Calcium Gluconate
Calcium Phosphateans
Calcium Carbonate
Calcium Hydroxide
Q) Which of the following options describes the lymph group
to which the vulva drains?
A Deep inguinal lymph nodes
B Internal iliac lymph nodes
C Para-aortic lymph nodes
D Superficial inguinal lymph nodes..ans
E Superior mesenteric lymph nodes
The vulva

Q) The haemorrhage considered the 6th cause maternal


mortilityT
Mat mortalioty causes Sepsis
Preeclamsia and eclampsia
Vte
Amniotic fluid embolism
Ectopic
Haemorrage
Anaesthesia

Q) Which of foll statements regarding placenta and fetomaternal circulation are FALSE?
Deoxygenated fetal blood arrives at the placenta via 2
umbilical arteries
Oxygenated blood returns to the fetus via a single umbilical
vein
There is no direct mixing of fetal and maternal blood at the
placenta
At term the placenta receives 70% of uterine blood
flow..ans
In the umbilical vein the pressure is approximately 20 mmHg
At term the placenta receives around 90% of
uterine blood flow
Q) Paraurethral glands skenes present in:
Superficial perineal pouch
Deep perineal pouch

Q) Minute ventilation in pregnancy increases due to which of


the following?Effects of gravid uterus on diaphragm
Dliutional anaemia
Increased renal excretion of bicarbonate
Increased circulating oestragen
Increased circulating progesterone..ans
This is thought to be the result of increased
circulating progesterone. Progesterone is known
to directly stimulate ventilation by sensitizing the
CNS respiratory centres to CO2.
Increased minute ventilation "blows off" CO2 and
as a result pCO2 is reduced. pH homeostasis is
maintained via increased renal excretion of
bicarbonate.

Q) During pregnancy the GFR changes as follows:


GFR remains the same
GFR decreases by 15%
GFR increases by 15%
GFR decreases by 40%
GFR increases by 40%........ans ,inc by 40-50 %

Q) A 67 years old woman is admitted to hospital with


frequently of urination and extreme thirst. Her blood tests
reveal deranged urea and electrolytes. Provisional diagnosis is
diabetes insipidus. Which hormone acts in nephron, to
increase the permeability of the collecting ducts to water?
A. Aldosterone
B. Angiotensin
C. Atrial natriuretic peptide

D. Parathyroid hormone
E. Vasopressin..ans
Q) Which one is a type of chromosome
heteromorphism?
Balanced translocation
Non dysjunction
Gene deletion
Unbalanced translocation
Fragile sites..ans
Q) Which of foll hormones inhibits Galactopoiesis and
Lactogenesis postpartum?Oxytocin
Progesterone
Dopamineans
hPL
Prolactin
Q) At what stage of fetal development does fetal haemoglobin
(HbF) replace embryonic haemoglobin (HbE) as the primary
form of haemoglobin?4-6 weeks
10-12 weeksans
20-24 weeks
32-36 weeks
At Birth
Q) A 23 years old woman develop a fever and has
offensive discharge and abdominal pain. You are
concerned that she is septic and wish to
administer intravenous antibiotics. Prior to
administration you wish to calculate her

estimated GFR in order to dose her appropriately.


Which of the following factors is included when
calculating eGFR? A. Creatinine
B.Diabetic status
C.Height
D.Medication
E.Weight

Q) What is the typical peak change in oxygen consumption in


a pregnant vs a non-pregnant women? 5%
20%..........ans
60%
100%
120%
The typical Oxygen Consumption (VO2) is 250ml/min.

In pregnancy this increases by around 20% (or 50ml/min) to


300ml/min
Q) You review a patient who is 34 weeks pregnant. She
complains of gradually worsening itching over the past 6
weeks particularly to the hands and feet which is worse at
night. You order some bloods. Which of the following would
you normally expect to increase in the 3rd trimester? ALT
ALP..ans
Bilirubin
Albumin
Calcium
Q) A woman with schizophrenia presents with galactorrea
..the cause of hyperprolactinemia is mostly due to
Hypothyroidism
Macroprolactinoma
Microprolactinoma
Renal faliure
Use of phenothiazine..ans
Q)72 y is undergoing lung function tests prior to abdominal
surgery. Which of foll gives the correct lung volume equation?
A. Functional residual capacity= residual volume+ tidal
volume
B. Inspiratory capacity= tidal volume+ expiratory reserve
volume
C. Inspiratory capacity=inspiratory reserve volume+ tidal
volume..ans
D. Total lung capacity=inspiratory capacity+ residual volume

E. Vital capacity=inspiratory capacity+expiratory reserve


volume
Q) Mc cause of secondary dysmenorrhea?Endometrioses
Q) Menarche before 10 years of age is considered precocious
puberty
Q) During wound healing the clotting cascade is activated.
Which of the following activates the extrinsic pathway?
Damaged endothelium
Thromboxane A2
Prostaglandin E2
Tissue Factorans
Fibrin
The clotting cascade can be activated either by damaged
endothelium (Intrinsic pathway) or Tissue Factor (Extrinsic
pathway)

Q) Which of the following cause a shift of oxygen


dissociation curve to the left?
A. Decreased hemoglobin
B. Decreased 2,3- DPG.ans
C. Increased acidity
D. Increased carbon dioxide
E. Increased temperature

Q) What is the average volume of blood loss during the


menstrual cycle
35-40ml.ans
60-65ml
80-85ml
150-200ml
350-400ml
average 35-40 ml
max 80-85 ml
Q) Which of foll prolongs Prothrombin time? Factor V
deficiency
Von Willebrand disease.ans
Hemophilia

Aspirin therapy
Protein C Deficiency
Causes of prolonged Prothrombin time (PT) and also INR
(INR is a ratio of PT):
Factor V deficiency
Warfarin therapy
Liver Failure
DIC
Q) What is the average lifespan of a platelet? 3-4 days
5-9 daysans
14-21 days
30 days
90 days
Blood Cell Lifespans
Red Blood Cells 120 days
Platelets 5-9 days
White blood cells 2-5 days
Neutrophils (up to 5 days)
Basophils (2 to 3 days)
Eosinophls (2 to 5 days)
Monocytes (1 to 5 days)
Lymphocytes (variable)
Q) Lactogenesis at term is stimulated by which hormone?
Oxytocin
hPL

Progesterone
Prolactin.ans
Dopamine
Q) which following drug suppress pituitary LH
production,hepatic SHBG production and use to
treat Hirsutism? Spirinolactone
GnRH Analogue
Flutamide
Gestrinone
COCPans
Q) On TVs 4-5 week gestational sac visible
5 week yolk sac
5-6 embryo
Q) Which one is true for congenital hip dislocation ?
A m.c in African population
B m .c in first born female babies ..ans
C m.c in first born in male babies
D needs x-Ray for confirmation
E only surgically treated
Q)Cardinal signs of inflamm
rubor-redness
calor-heat
dolor-pain
tumor- swelling
functiolessa- loss of functions

Q)Anal canal epithelium : They 3 types


Inner columnarer
Outer : divided by white line non-keratanaized st.sq and
below the line keratinaized st.sq
Q)

Q) Issue in pre-pregnancy counseling of women


with heart disease ??
A. Risk of IUGR
B. Risk of maternal death
C. Maternal and fetal monitoring during labour
D. Reduction in maternal life expectancy
E. Need for frequent hospital attendance and
possible admission..ans
Q) Assuming normal renal function ,by how many
days post Partum will HB start to increase ?
A day 1
B day 3 ..ans
C day 7
D day 10
E day 14
Q) In precocious puberty:which is false
A. Treatment is down regulation of pituitary with
GnRH analogues
B. Pubertal changes occur earlier than normal
C. Mostly idiopathic
D. Sequence of events that occur there after
mimics normal puberty..F
E. When investigating children the exclusion of a
serious tumor is very important
Q) A 28 year old women is seen in the early pregnancy unit.
She has had a positive pregnancy test but is uncertain of her
LMP. Ultrasound doesn't visualise a pregnancy. You perform a

progesterone test. The result is 59 nmol/l. According to NICE


guidelines what level is consistent with viable pregnancy? >5
nmol/l
<15 nmol/l
>25 nmol/l
>60 nmol/l
None of the above..ans
NICE state progesterone levels shouldn't be used
to diagnose viable or ectopic pregnancy.
RCOG suggest levels >60 are consistent with
viable pregnancy and you may get a
progesterone level in a clinical scenario but not
usually without serial hCG measurements.
This may seem like a trick question but highlights
that if you get a question on pregnancy of
unknown location then the clinical picture is of
primary importance followed by hCG
measurements.
Q) Gastrochisis is due to undevelopemnt of
Abd wall
Q) A 16years old female presents to your clinic with per
vaginal discharge. upt is negative. you took a swab and it
revealed microscopic image as below. what would be your
first line treatment for her? a. azithromycin 1g once only
b. doxycycline 100mg bd + metronidazole 400mg tds for 7

days
c. doxycycline 100mg bd for 14days + metronidazole 400mg
tds for 7 days
d. metronidazole 400mg tds for 5 days..ans
e. tinidazole 2g once only
it is trichomoniasis. it is an anaerobic infection.
Tinidazole can also be given but after failure of
metronidazole. therefore, 1st line is
metronidazole.
Q)Labia Mj nv supply ant 1/3 by illioinguinal
nerve
post 2/3 by perineal division of pudendal nerve.
Q) anterior part of labia majora supplied by which
nerve?Hypogastric
Q) Which one is notifiable disease ?
A clostridium difficle
B measles .ans
C MRSA
D pneumococcal pneumonia
E rheumatic fever
Q) Please confirm according to last guideline
Mc direct maternal death is sepsisT , (Sepsis
Eclampsia VTE)
And indirect is cardiac disease.T
Q) What is the lymphatic drainage of the ovaries? A. Common
iliac nodes

B. External iliac and superficial iliac nodes via the round


ligament
C. External iliac nodes
D. Internal iliac nodes
E. Lateral aortic and preaortic nodesans
The answer is lateral aortic and pre-aortic nodes.
It is useful to remember the following: the
bladder drains to the external iliac nodes; the
urethra drains to the internal iliac nodes; the
fallopian tubes and fundus uteri drain to the
external iliac and superficial iliac nodes via the
round ligament; and the cervix drains to the
external and internal iliac, rectal and sacral
nodes and occasionally obturator nodes.
Q) incidence of asymptomatic bacteuria in
pregnant females? 2- 7%
Q) At which cell cycle checkpoint is the cell cycle halted if
the cell's DNA is damaged?
a) G1 S..ans
b) S - G2
c) G2 - M
d) G0 - G1

Q)

ORIGIN
Perineal body (and midline raphe
over corpus spongiosum in male)
INSERTION
Superficial perineal membrane and
dorsal penile/clitoral aponeurosis
ACTION
Male:aids emptying of urine and
ejaculate from urethra. Female:
closes vaginal introitus
NERVE
Perineal branch of pudendal nerve
(S2, 3, 4)

Q) DKA is associated with Hypokalemia


Q) a patient had hypophsectomy for pituitary tumor she has
amenorrhea for 8 months ovulation induction can be done in
her by:
a)clomiphene citrate
b)pulsatile hCG
c)hMG
d)hMG followed by pulsatile hCG.ans
Q)

Ans D
Q) Regarding varicella zoster virus, SBA
1) it is a single-strain herpes virus..ans
2.it contains single stranded DNA
3.the incubation period lasts one week
4.infections are preventable by a killed vaccine
Q) Rubella virus, SBA : 1) Contains double stranded DNA
2)has incubation period of 13-20 days.ans
3) has killed vaccine
4) is treated with antivirals
Q) Regarding Meningococcus: SBA
1)it is not part of the normal flora in human
2)infected people develop a slow spreading
petechial rash
3)people with meningitis typically have
coexisting septicemia
4)the cerebrospinal fluid glucose concentration is

increased
5)treatment is with penicillinans
Q) Guthrie test was done on a newborn and it was positive.
What is the mode of inheritance of this disease? a. autosomal
dominant
b. autosomal recessive.ans
c. x linked dominant
d. x linked recessive
e. none of the aboue
phenylketonuria gives guthrie test positive due to
accumulation of phenylalanine in blood. it is inherited as AR.
Q) Vitamin D undergoes a few process of activation. Where
does the final activation step predominantly occurs? a.
juxtaglomerulus apparatus
b. distal convoluted tubule
c. hepatocyte
d. proximal convoluted tubule..ans
e. loop of henle
Q) Calcitriol is important to maintain a good calcium
homeostasis. what is the effect of its function? a. increase
serum ca, increase serum phosphate
b. increase bone formation
c. both aboveans
d. increase serum ca, decrease serum phosphate
e. reduce kidney reabsorption of phosphate

Q)

Ans D
Q) Which statement is false regarding non herpetic genital
ulcers
1) lymphogranulom venerum is caused by specific serovars of
chlamydia trachomatis
2)lymphogranuloma venerum causes severe proctocolitis
3)chancroid is an infection caused by klebsiella
granulomatis.ans
4)chancroid causes inflamed lymph nodes to weep through
skin
5)granuloma inguinale causes elephantiasis

Q) Defective feedback control of calcium hemostasis may


lead to secondary hyperparathyroidism. It is usually

associated with - a. parathyroid gland tumour


b. chronic liver disease
c. thyroid carcinoma
d. chronic kidney disease.ans
e. SIADH

Q) A 31-year-old woman who is known to be HIV


positive presents following a positive pregnancy
test. Her last menstrual period was 6 weeks ago.
The last CD4 count was 420 * 106/l and she does
not take any antiretroviral therapy. What is the
most appropriate management with regards to
antiretroviral therapy?
A - Check CD4 at 12 weeks and initiate
antiretroviral therapy if CD4 count is less than
350 * 106/l
B - Do not give antiretroviral therapy
C - Start antiretroviral therapy at 20-32
weeksans
D - Start antiretroviral therapy at 10-12 weeks
E - Start antiretroviral therapy immediately
Treatment with a single anti-retroviral drug (zidovudine) may
be considered if your viral load is less than 10000, your CD4
count is more than 350 and you are prepared to have a
caesarean section.Your doctor will usually recommend that
you start the treatment between 14 and 24 weeks of your
pregnancy and continue until your baby is born.

You should be able to have a vaginal delivery, even if you


have had a caesarean section before, if you are taking
HAART, have a viral load less than 50 and a CD4 count more
than 350.
If you are taking HAART and your viral load is between 50
and 399, your doctors may recommend a caesarean section,
usually at 38 weeks. This will depend on the pattern of your
viral load, how long you have been on treatment and your
wishes. You will be advised to have a caesarean section,
usually at 38 weeks, if: {{ you are taking HAART and have a
viral load of 400 or more {{ you are taking zidovudine
alone{{ hepatitis C virus is detected in your blood.
You should be prescribed zidovudine through a drip, which
will be started a few hours before your caesarean section. It
should continue until your baby is born and the umbilical cord
has been clamped.
Q) In which phase of the cell cycle is DNA replicated?
a) G1 phase
b) S phase.ans
c) G2 phase
d) M phase

Q) A new mother who is 4 weeks post partum


presents fir warm red tender patch on the rite
side just lateral to areola this has been getting
worse in last three days and feeding is now
painful she saw a midwife yeaterday helps with
positioning but it is not improved on examination
has mastitis of rt brest no obvious absess what is
most appropriate managment ?
A.co amxiclav, continue breast feeding
B.flucloxacillon, continue Bf .ans
C.flucolxacillin, stop feeding
D.co amociclav , stop feeding
E. Metronodazole , continue feeding
Q) main supply of levator ani muscle is : S1
S2
S3
S4..ans

Q)

Q) Incubation period for varicella infection


5-7days
7-10days
10-14 days
10-20 days..ans
14-21 days
Q) Vasectomy is associated with which one of the above
complications?
A An increased risk of coronary heart disease
B An increased risk of prostate cancer
C A 10% risk of developing anti-sperm antibodies
D An increased risk of epididymo-orchitis..ans
E A failure rate of 1:20,000
Complications of vasectomy
1) Local Anaesthetic complications
Pain, Bleeding, Infection
Ischaemic necrosis - rarely
2) Surgical complications
Intraoperative bleeding
Early post operative - Infection (wound,
epidydimytis), Scrotal haematoma, Post
operative pain
Late post operative - Chronic testicular pain,
Anti sperm antibodies nearly 75% will develop
antibodies
Failure of method rate is 1 in 2,000

Q)

Ans C
Q) MRSA infections may respond to which
antibiotics
.Augmentin
.carbapenams
.cephalosporins
.clindamycin
.vancomycin.ans
Q) 19 year old university student is concerned that she might
have Chlamydia infection.
Which one of the above is not a sign / symptom of Chlamydia
infection?

A Intermenstrual bleeding
B Joint pains
C Cervical excitation
D Exophytic Cervical mass..ans
E Mucupurulent vaginal discharge
70% of women are asymptomatic. If symptoms are present,
they include: Lower abdominal pain, Dyspareunia,
Abnormal vaginal bleeding e.g. postcoital bleeding,
intermenstrual bleeding, Abnormal vaginal or cervical
discharge.
May be mucupurulent. Dysuria.
Rarely, patients can present with right upper quadrant pain
(peri-hepatitis) and joint pains (reactive arthritis).
Signs: The following may be present: Pyrexia, Lower
abdominal tenderness, Cervical excitation, Contact bleeding

Q)

Ans C
Q) Which ovarian tumor is difficult to distinguish
from endometriosis ?
A.chorioacarcinoma
B.mature cyst teratoma..ans
C.yolk sac tumor
D.mucinous tumor
E.serous teratoma

Q)

Ans E,theca produce androgens


Q) Which one of above is not a CI to the use of depomedroxyprogesterone acetate?
A History of stroke
B Active viral hepatitis
C Suspicious vaginal bleeding
D Healthy 38 year old smokerans
E Diabetic retinopathy
Progesterone-only injectables: Contraindications:
UKMEC 3 risks outweighs the benefits of using the method.
Multiple risk factors for arterial cardiovascular risk e.g.
older age, diabetes, hypertension, smoking.
Vascular disease
Current or history of IHD. On anticoagulants.
Current or history of stroke
Past history of breast cancer & NO evidence of current
disease for 5 yrs
Unexplained vaginal bleeding or suspicious before

investigations
Retinopathy, nephropathy, neuropathy or diabetes > 20 yrs
duration
Active viral hepatitis
UKMEC 4 unacceptable health risks or absolute
contraindications
Current breast cancer UKMEC 4
Q) A 33 year old woman has been diagnosed with ovarian
hyperstimulation syndrome. Which one of the above features
would suggest severe OHSS?
A Haematocrit > 45%...............ans
B Ultrasound evidence of ascites
C Ovarian size of 8 12 cm
D Urine output of 650 mls over 24 hrs
E Mild abdominal pain
Severe OHSS
Clinical ascites +/- hydrothorax
Oliguria
Haemoconcentration (haematocrit > 45%)
Hypo-proteinaemia
Ovarian size usually > 12cm
Q) Characteristics of common organisms include
all except
A)Neisseria gonococcus is non motile
B)Mycobacteria are gram +ve
C)Bacillus anthracis is anaerobic..ans,both
aerobic and anerobic.Bacillus species is aerobic

D)Ecoli is motile
E)Gardnerella vaginalis is gram -vegm
variable can be +ve or -ve
Q) Which one of above statements is true?
A Cervical ectropion is a pathological process resulting from
eversion of the lower cervical canal
B Cervical ectropion is a risk factor for developing CIN
C Cervical ectropion in early pregnancy is an indication for
termination
D Cervical ectropion is assoc with adolescence.ans
E Cervical ectropion is pre malignant
Q) Sunlight plays a major role in vitamin D production. In
certain country, during winter, day is shorter than night.
People in those country produce vitamin D during summer
and store them for usage during winter. where is the vitamin D
being storeda. adipocyte..ans
b. hepatocyte
c. red blood cell
d. adrenal
e. dermis
Q) Anencephaly include
A)high AFP and Inc estriol
B)low AFP and normal estriol
C)low AFP and low estriol
D)high AFP and low estriol
E)high AFP and normal estriol

Q) Diagnostic test for cystic fibrosis in blood


a. FISH
b. sweat test
C .immunoreactive trypsinogenans
d. karyotyping
Blood > immunoreactive trypsinogen
Sputum ,throat swab > FISH
Sweat > sweat test
Q) With respect to pre term births,which is true??
A.The majority of preterm are due to
spontaneous pre term labour
B.About a third of preterm births are due to
multiple pregnanciesans
C.Over 75% preterm births due to infection
D.About 50% are Iatrogenic
E.UTI most common cause of preterm births

SPONTANEOUS PRETERM 30%


MULTIPLE PREGNANCIES 30%
GENITO-URINARY INFECTION,PROM 20-25%
ELECTIVE 10-15%
Q) which are the following is correctly matched?
a. chylomicron - transports TAG from liver to tissues
b. VLDL - transports TAG from intestine to liver
c. LDL - transport cholestrol from intestines to tissues
d. HDL- transport cholestrol for storage
e. none of the above.ans
chylo transport TAG from intestine to tissues
vldl transport tag from liver to tissue
ldl transpost cholestrol from liver to tissue
hdl transport cholestrol for excretion
Q) A diabetic women attends your preconception clinic. You
have checked her HBA1C. According to NICE guidelines
what HBA1C level would prompt you to strongly advise this
women NOT to get pregnant due to the signifiant risks it
presents?
>6.5% or 48mmol/mol
>7.6% or 60mmol/mol
>10.0% or 86mmol/mol..ans
>11.1% or 98mmol/mol
>12.6% or 114mmol/mol
Q) Ribosomes are located in which part of the
cell? A. Golgi complex
B. Lysosomes

C. Mitochondria
D. Rough endoplasmic reticulum
E. Smooth endoplasmic reticulum
The answer is Rough endoplasmic reticulum.
Ribosomes 'read' the messenger RNA created
from the nuclear DNA and translate it to create
proteins via transfer RNA. There are free
ribosomes in the cytoplasm of cells but they are
in abundance in the rough endoplasmic reticulum
Q) Which part of the cell cycle is noted for sister chromatids
separating and moving to opposite sides of the cell? A.
Anaphase .ans
B. Metaphase
C. Prophase
D. Synthesis phase
E. Telophase
The answer is Anaphase. Within the cell cycle DNA
replication occurs in the synthesis phase before entering the
Gap 2 phase. Mitosis starts with chromosomes condensing
(prophase) follow by chromatids aligning (metaphase),
following by sister chromatids separating and moving
(anaphase). Finally the cell membrane divides in telophase
Q) Which of the following amino acid bases is not contained
in RNA?
A. Adenine
B. Cytosine
C. Guanine
D. Thymine

E. Uracil
:
The answer is Thymine. In RNA the thymine is replaced by
uracil. The other bases (adenine, cystosine and guanine)
remain the same.
Q) Which of the following molecules generates weak forces
that can attract neutrophils to cell walls? A. Cadhedins
B. Eicosanoids
C. Hemidesmosomes
D. Integrins
E. Selectinsans
Selectins are molecules that are expressed in endometrial cell
walls and also leucocytes. They generate weak attraction
forces. Inflammatory processes via interleukins increase the
amount of selectins present and hence, attract more leucocytes

Q) Causes of DIC include all except


A)amniotic fluid embolism
B)hep E
C)massive blood loss
D)acute fatty liver of pregnancy
E)none aboveans
Q) Notochord from :
1.mesoderm ..ans
2.endoderm
3.ectoderm

Q) Which hormone is made in ovarian granulosa cells before


ovulation occur
A)testosterone
B)progesterone
C)antimullerian hormoneans
D)alpha fetoprotein
E)aldosterone
Q) From which of the following structures are the inferior
parathyroid glands derived?
A-Mesoderm of first pharyngeal arch
B-Mesoderm of second pharyngeal arch
C-Pouch of first pharyngeal arch
D- Pouch of second pharyngeal arch
E-Pouch of third pharyngeal arch..ans
Q) Optimum pressure range for primary trocar
insertion at laproscopy
A) 10-15mmhg
B) 20-25mmhg.ans

C) 20-25 mm water
D) 10-15 mm water
E) 15-18mmhg
Q)

Ans C
Heprin and warfrin is give for 3 to 5 day combine then heprin
withdraw.
Q) RBC no mitochondria,energy needs met by anerobis
respiration

Q) where in the body ketone bodies produced during


starvation??
Brain
Skeletal muscle
Cardiac muscle
Liver..ans
Heart

Q) A 30-year-old woman, who is 8 weeks pregnant, presents


to accident and emergency with PV bleeding and crampy
abdominal pain. She has a past medical history of a rightsided deep vein thrombosis and two previous miscarriages.
She is sent to the early pregnancy assessment unit, where
ultrasound confirms miscarriage. What is the most likely
underlying diagnosis? A. Rheumatoid arthritis
B. Anti-phospholipid syndromeans
C. Sjgrens syndrome
D. Discoid lupus
E. Systemic lupus erythematosus (SLE)
Q) A team wish to audit their departmental results on the use
of anticoagulation in patients with obstetric thromboembolic

disease.Which one of the following options is the most


appropriate next step in the audit cycle?(Please select 1
option)
A) Data analysis
B) Data collection
C) Identify standards
D) Implement change
E) Needs assessment
Q) Which one is notifiable disease ? A clostridium
difficle
B measles .ans
C MRSA
D pneumococcal pneumonia
E rheumatic fever
Q) A diabetic women attends your preconception clinic. You
have checked her HBA1C. According to NICE guidelines
what HBA1C level would prompt you to strongly advise this
women NOT to get pregnant due to the signifiant risks it
presents?
>6.5% or 48mmol/mol
>7.6% or 60mmol/mol
>10.0% or 86mmol/molans
>11.1% or 98mmol/mol
>12.6% or 114mmol/mol

Patients with HBA1C of 10.0% or greater should be strongly


advised to use contraception and improve glycemic control
prior to getting pregnant.
Diabetes In Pregnancy
NICE updated its guidance on management of diabetes in
pregnancy in 2015. The current guidelines advise the
following regarding management including new HBA1C
targets:
Advise women with diabetes who are planning to become
pregnant to aim to keep their HbA1c level below 48
mmol/mol (6.5%), if this is achievable without causing
problematic hypoglycaemia.
Reassure women that any reduction in HbA1c level towards
the target of 48 mmol/mol (6.5%) is likely to reduce the risk
of congenital malformations in the baby.
Strongly advise women with diabetes whose HbA1c level is
above 86 mmol/mol (10%) not to get pregnant because of the
associated risks.
Q)

Q)

Q) Which artery is a direct branch of the aorta?


A. Inferior vesical
B. Internal iliac
C. Ovarian .ans
D. Uterian
E. Vaginal
The ovarian artery is a branch of the aorta. It
arises anterolaterally just below the renal artery,
running retroperitoneally to leave the abdomen
by crossing the common or external iliac artery in

the infundibulopelvic fold. It crosses


corresponding ureters and supplies twigs to it but
does not supply to abdominal organs. The
internal iliac artery arises from the common iliac
and its inferior branch further supplies to the
pelvis.
Q) The risk of vertical transmission of HIV with no
intervention?
1. 10-20
2. 5-10
3. 15-25
4. 25-40
5. More than 50ans
Q) Pre implantation genetic diagnosis is ?
A. Is Done in early stage of human zygote embryo
development.ans
B. Carried out at 10 week of gestation.
C. Should be offer at all women.
D. Is done so that termination can be offered to couple timely.
D. Carries a small risk of miscarriage.
Q) A 38 year old woman attends her GP at 10 weeks gestation.
She is complaining of fever and has pains in her joints. She
developed a rash yesterday. On examination, she has at
temperature of 38, postauricular lymphadenopathy and a
maculopapular rash over her torso.
what is the most likely fetal abnormality to occur as a result of
this acute infection?

a. cerebral palsy
b. failure to thrive
c. limb hypoplasia
d. microcephaly
e. sensorineural hearing lossans
pt is having rubella infection. arthalgia, fever ,
rash and lyphadenopathy is highly likely to be
rubella.
arthalgia is a prominent clinical feature due to
deposition of antigen antibody complex on the
synovial membrane.
rubella infection causes 80% hearing loss, 60%
heart defects , 30 % cataract.
limb hypoplasia is associated with varicella zoster
microcepaly associated with cytomegalovirus

Q) The legal age for prescribing a contraception


to a girl or agree on a request of termination of
preg (TOP) in UK is :
A. 14 yrs
B. 16 yrsans
C. 18 yrs
D. 20 yrs
E. 21 yrs
Q) the following are responsible for higher circulatory fatty
acid except -a. ACTH
b. stress

c. insulin..ans
d. diabetes
e. glucagon
key c. the rest stimulates the hormone sensitive
lipase (HSL) to break TAG to fatty acid
Q) which organism is responsible for the
occurance of the diagram below? a. trichomonas
vaginalis
b. chlamydia trachomatis
c. neisseiria gonorrhea
d. gardrenella vaginalisans
e. hemophilus ducreyi
bacteria vaginosis presents with high ph
discharge due to reduction of lactobacilli. they
have offensive fishy odor discharge and presence
of clue cells on histology

Q) What is the blood flow to the uterus in a term pregnancy?


A. 500 ml/min
B. 600
C. 700..ans
D. 800
The uterus receive 400ml/min from the extra
1500 ml , so those calculated it from the total

Q) Which artery provides main blood supply to breast ?


A axillary artery
B internal mammary artery..ans
C lateral thoracic artery
D superior thoracic artery
E thoracoacromial artery
Q) Which of the following is not a risk factor for surfactant
deficiency?
A. Elective Caesarean section
B. Intrauterine growth restriction.ans
C. Male gender
D. Prematurity
E. Sepsis
Q) Fluorescent insitu hybridization (FISH) analysis
is useful in all the following situations, except

1) Determination of sex in cases of ambiguous


genitalia
(2) Determination of uniparental disomy
ans.
(3) Rapid diagnosis of trisomies
(4) Identification of submicroscopic deletions
Q) Hypophosphatemia shifts the curve Left
Q) In spermatogenesis, at which stage do the sex
chromosomes segregate?(1) Meiosis II, primary
spermatocyte
(2) Meiosis I, primary spermatocyteans
(3) Meiosis II, secondary spermatocyte
(4) Meiosis I, secondary spermatocyte
Q) A number of cytogenetic method can be
employed in prenatal genetic screening.which
screening methods examines INTERPHASE
CHROMOSOME?
Agarose gel electrophoresis
FIuroscent In situ hybridisation .ans
Multiplex ligation dependent probe amplification
PCR
Z-DNA Isolation and amplification
Q) In men, the lower part of gubernaculum
becomes the scrotal ligament, with the upper
part degenerating, In women, the upper part
becomes the suspensory ligament of the ovary,

whilst the lower part become the round ligament.


The wolffian duct degenerates in females but
contributes to the suspensory ligaments.
The mullerian ducts fuse to form the uterus,
cervix and vagina.
The bladder is formed from the cloaca, whilst the
urachus forms the median umbilical ligament.
Q)

Ans C
1.Chloroquine FDA Category C
can be used Causiously (Benefits >harms).
2.Hep A can be given
3.Tdap is recommended to be given specially late
trimester
4.MMR is live attenuated vaccine So
Contraindicated.

Q)

Ans B
Q) Alcoholic pregnant lady with obstetric
cholestasis what you will give -a) vitamin C
b) vitamin K.ans
c) vitamin B1
d) vitamin B6

Q)

Q)

Q)Planned VBAC is associated with an


approximately how much % risk of uterine

rupture - 1%.......ans
2%
3%
10%
Q) What proportion of cytomegalovirus (CMV) infections is
asymptomatic in immune-competent patients?a- 10%
b- 40%
c- 60%
d- 80%
e- 90%...........ans
The answer is 90%. CMV infection in adults is asymptomatic
in 90% of cases. If primary infection occurs in pregnancy,
then there is a 4050% risk of fetal transmission. Between
22% and 38% of the infected fetuses will develop symptoms.
Q) Which of foll enzymes, which may rise in hepatitis and
myocardial infarction, is present in the liver cell cytosol, brain
and myocardium?
A) Alkaline phosphatase (ALP)
B) Aspartate aminotransferase (AST) ans
C) Gamma-glutamyl transpeptidase (GGT)
D) Alanine aminotransferase (ALT)
E) Serum bilirubin
Q) Which of the following is a cause of hyponatraemia?
A) Cirrhosisans
B) Conn's syndrome
C) Cushing's syndrome

D) Diabetes insipidus
E) Hypertonic saline
Q) Of the SGA infants missed by conventional measurements
of SFH, more than half would
have been picked up using customised assessment of
SFHF
Q)

Q) IU pregnancy at 7 weeks what would be


expected Bhcg levels ?
A 5000IU.ans
B 300,000IU
C 120IU
D 50IU
E 300IU

Q) Plasma PH is neutral 7.4 (range 7.35-7.45)

Q) The localization of a genetic locus to a


particular chromosomal region using linkage or
molecular analysis is known as 1) Gene mapping
(2) Gene cloningans
(3) Gene isolation
(4) Gene splicing
Q) Acidosis is associated with hyperkalaemia and
alkalosis with hypokalemia ..T
Q) A 16-year-old adolescent is seen in your clinic.
On physical examination, you note that he has
small testis for his stated age and has poorly
developed secondary sexual characteristics. In
addition, on physical examination, there is
notable gynaecomastia. He is rather tall, with
abnormally long upper and lower limbs. A buccal
smear is obtained and examined microscopically.
How many chromatin positive inclusion bodies
are seen?
(1) 0
(2) 1.ans
(3) 2
(4) 3
Q) All of foll are stone formation inhibitors except
A ) magnesium
B) syprophosphate
C) calciumans

Q) The standard karyotype is performed by


photomicroscopy of cells at which mitotic stage?
(1) Interphase
(2) Prophase
(3) Metaphase.ans
(4) Anaphase
Q) The mc cause of learning disability
Fragile x syndrome
Down syndromeans
Q) SV inc by - a.50%
b.70%
c.20%..........ans
Q)Hormones synthesised by kidney include which of foll?
A) 1,25 dihydroxycholecalciferol .ans
B) Aldosterone
C) Angiotensin I
D) Angiotensin II
E) Cortisol
Q)Your 25-y preg at 36 w. She has an acute UTI. Which of
foll medications is contraindicated in Rx of UTI in this
patient? a. Ampicillin
b. Nitrofurantoin..ans, CI in 3rd trimester Becoz
haemolytic anaemia in new born
c. Trimethoprim/sulfamethoxazole

d. Cephalexin
e. Amoxicillin/clavulanate
Q)20-y - routine well-woman examn. She has a h/o acne, for
which she takes minocycline and isotretinoin on daily basis.
She also has a h/o epilepsy that is well controlled on valproic
acid. She also takes a COCP containing norethindrone acetate
and EE. She is a nonsmoker but drinks alcohol on daily basis.
She is concerned about effectiveness of her birth control pill,
given all medications that she takes. She is particularly
worried about the effects of her medications on a developing
fetus in event of unintended preg. Which of foll drugs has
lowest potential to cause birth defects?a. Alcohol
b. Isotretinoin (Accutane)
c. Tetracyclines
d. Progesterone.ans
e. Valproic acid (Depakote)
Q) bilirubin is bound to albumin in circulation..T

Q) Natural killer cells belong to which cell type?


A) Basophils
B) Eosinophils
C) Lymphocytes.ans
D) Monocytes
E) Neutrophils

Q) A 17-year-old primigravida complains of constipation and


arthralgia at 28 weeks gestation.A number of biochemical
investigations are performed, but which of these is clinically
significant? (Please select 1 option)
A) Detectable urinary human chorionic gonadotrophin
B) Free thyroxine 8.9 pmol/L (9-22)
C) Prolactin of 1000 mU/L (<450)
D) Serum alkaline phosphatase of 350 IU/L (50-110)
E) Serum corrected calcium 2.89 mmol/L (2.2-2.6).ans
Q) A patient presents for prenatal care in the second trimester.
She was born outside the United States and has never had any

routine vaccinations. Which of the following vaccines is


contraindicated in pregnancy?
a. Hepatitis A
b. Tetanus
c. Typhoid
d. Hepatitis B
e. Measles.ans

Q) Decidual reaction occur at : Day 8..ans


Day12

Q)

Q) Which immunoglobulin is key to passive neonatal


immunity? IgD
IgE
IgM
IgA
IgG.ans
Q) Myasthenia Gravis is an example of what type of
hypersensitivity reaction? Type I
Type II
Type III
Type IV
Type Vans
Myasthenia gravis and Graves' disease as well. These are also
classified as type 2 in the older classification

Q) Folliculogenesis 375 d

Q) Cross-over occure in which stage of prophase 1 in


meiosis?
A.Stage 1
B.2
C.3..ans

D.4
E.5

Q) A 33 year old women presents to clinic for infertility


investigations. She has a significant psychiatric history. Her

blood tests reveal a raised Prolactin. Which of her medications


below is NOT known to cause this : Amitriptyline
Zopicloneans
Chlorpromazine
Risperidone
Cocodamol
Drug Cause of Hyperprolactinaemia include:
Atypical antipsychotics eg risperidone
Phenothiazines eg chlorpromazine
Butyrophenones eg haloperidol
Thioxanthenes
Metoclopramide
Dopamine synthesis inhibitors eg -Methyldopa
Catecholamine depletors eg Reserpine
Opiates eg Codeine
H2 antagonists eg Cimetidine, Ranitidine
Amitriptyline
SSRI's eg Fluoxetine
Calcium channel blockers eg Verapamil
Oestrogens
TRH

Q)

Q)

Q) OCP is contraindicated in:


Bronchial asthma
Hyperthyroidism

Wolf Parkinson white disease


Systemic lupus erythematosus..ans

Q) Regarding the Progesterone-nnly Pill (POP) which of the


following statements regarding its mechanism of action is
FALSE?
A. With Desogestrel-only pills the primary mode of action is
via inhibition of ovulation
B. All types of POP thicken cervical mucus
C. Up to 97% of cycles in women using a levonorgestrel-only
pill are anovulatory..ans
D. Blood pressure and weight should be measured prior to
prescribing a POP

E. The only UKMEC 4 criteria for POP prescription is Breast


Cancer within the past 5 years

Q) During normal pregnancy which one is


correct ?
A 20% increase in plasma volume by 32 weeks
gestation
B Decrease in WBC count
C inc in proportion of B to T lymphocytes
..ans
D increase in antithrombin 3
E lupus anticoagulant in circulation
Q)

Ans C
Q) A baby had bradycardia and was delivered by
ventouse.The umbilical artery pH wad7.05, base
excess -4, while the umbilical vein pH was 7.25,
base excess -2.5. What will you tell the
pediatrician about the test : Normal blood

testans
The baby had an acute episode of hypoxia before
delivery
The baby had chronic episode of hypoxia before
delivery
Here Ph 7.05( reduced )base excess -4 ( normal)
So acute hypoxia
Listed below are normal values in an umbilical
arterial sample in a term newborn:
PH: 7.18 7.38
PCO2: 32 66 (mmHg)
HCO3: 17 27 (mmol/L)
PO2: 6 31 (mmHg)
Base excess: -8 0 (mmol/L); (Base deficit: 0 8)
Q)

Ans C

Q) Angle of Louis T 4

Q) Content of spermatic cord include all except :


A)vas deferns
B)genital branch of genito femoral nerve

C)cremasteric artery
D)inferior epigastric arteryans
E)illio inguinal nerve

Q)

Ans C

Q) Renal artery passes behind : A)aorta


B)inf vena cavaans
C)ureter
D)diaphragm
E)psoas muscle

Q) progesterone only pills (POP)


Types of POP
1. Traditional (eg Femulen, Micronor, Norgeston)
Main mode of acton: thickening cervical mucus preventing
sperm entry at neck of womb
May also cause anovulation but this effect variable and
unreliable
2. Desogestrel (eg Cerazette)
Main mode of action inhibition of ovulation.
Also cause thickening of cervical mucus
UKMEC Conditions

UKMEC 1: A condition for which there is no restriction for


the use of the contraceptive
UKMEC 2: A condition for which the advantages of using the
contraceptive method outweigh the risks
UKMEC3: A condition where the theoretical or proven risks
usually outweigh the advantages of using the method
UKMEC 4: A condition which represents an unacceptable
health risk if the contraceptive is used
Note the UKMEC conditions vary between COCP and POP

.
Q) Cystic fibrosis is diagnosed in serum by : Sweat test
Immunoreactive trypsinans
Chloride test
A is confirmatory test

B is screening test
But A is not done in serum it's done on skin

Q)

Ans C
Q) The 2 most commonly used DHFR inhibitors
are:
Trimethoprim
Methotrexate.
Some antimalarials are also DHFR inhibitors.
Q) A patient is on the ward with a mechanical mitral valve.
There is no history of VTE. What is the target INR? 1.5 - 2.5
2.0 - 3.0
2.5 - 3.0
2.5 - 3.5..ans
3.0 - 4.0

With regard to warfarin therapy there are only 3


target ranges that are commonly used:
2.0 - 3.0 Most common used for DVT,PE & tissue
valve replacement treatment
2.5 - 3.5 Used in mechanical mitral valve
replacement & some aortic mech.valve

replacements
3.0 - 4.0 Used in mechanical valve replacement
where PE has occurred despite anticoagulation at
lower range
Q) Hyperthyroidism
Hyperthyroidism in pregnancy occurs in 2 in
1,000 pregnancies in the UK
Management Options
Antithyroid Drugs 1st Line
Propylthiouracil crosses 1st Choice as crosses
placenta less readily than carbimazole
Carbimazole
Radioiodine is contra-indicated
Beta-Blockers
May be used but use should be limited to a few
weeks as may adversely affect fetus
Surgery
Only when absolutely necessary. Patient needs to
be euthyroid prior to surgery
Q) Pharmacokinetics in Pregnancy
Maternal physiology changes during pregnancy include:
Increased maternal fat and total body water
Decreased plasma protein concentrations
Increased maternal blood volume, cardiac output, renal blood
flow and uteroplacental unit, and decreased blood pressure.
Dilutinal anaemia
Increased tidal volume

Delayed gastric emptying and gastrointestinal motility


Altered activity of hepatic drug metabolizing enzymes
Q)

Q)Pearl index

Q) You are called to review a baby with cyanosis when


feeding immediately after birth. Subsequent investigation and
imaging reveals choanal atresia. Upon questioning the mother
reveals she had been getting repeat prescriptions from her GP
in Poland without her UK GPs' or your knowledge. Which of
the following medication was most likely to cause this?
Carbimazole.ans
Ramipril
Ranitidine
Warfarin
Sertraline
Q)S pread of malignancy from pelvic viscera to
vertebral venous plexis system via
A)lateral sacral vesselsans
B)vertebral vein
C)lumber vein
D)inf mesenteric vein
E)post intercostal vein

Q)

Q)

Q) A patient is stated on Warfarin due to a Pulmonary


Embolism. What is the target INR? 1.5-1.9
2.0-2.5
2.0-3.0..ans
2.5-3.5
3.0-4.0
With regard to warfarin therapy there are only 3
target ranges that are commonly used:
2.0 - 3.0 Most common used for DVT,PE & tissue
valve replacement treatment

2.5 - 3.5 Used in mechanical mitral valve


replacement & some aortic mech.valve
replacements
3.0 - 4.0 Used in mechanical valve replacement
where PE has occurred despite anticoagulation at
lower range

Q)

Q) You see a patient who gave birth earlier in the day. She was
taken off warfarin during pregnancy and is currently on
LMWH. She intends to breastfeed for the first 6-8 weeks and

wants advice regarding restarting warfarin. Which of the


following is appropriate?
Recommence warfarin on stopping breastfeeding
Recommence warfarin immediately
Recommence warfarin in 6 weeks regardless of breastfeeding
status
Recommence warfarin in 5-7 daysans
Recommence warfarin 48 hours after breastfeeding has
stopped
There are 2 pieces of knowledge that will help
you answer this question. Firstly Warfarin is safe
to use when breastfeeding so breastfeeding has
no bearing on when to restart warfarin. The main
concern is postpartum haemorrhage and because
of this the advise is to wait 5-7 days after giving
birth before restarting.

Q)

Q) Obstetric conjugate : A) 10.5 cm


B)12cm
C)13cm
D)13.5 cm
E)none above

Q)

Ans b

Q) A 26-year-old second gravid who is work in a nursery


school attends the GPs surgery as she noticed vesicular rash
on her back and abdomen. She is 25 weeks pregnant. Her GP
confirms the diagnosis of chickenpox. What is the most
appropriate action?
A. Reassurance
B. Give a single dose of varicella zoster immunoglobulin
C. Advise serial ultrasound scans
D. Commence oral acyclovirans
E. Commence intravenous acyclovir
Q) Sure sign for sacral hiatus
A)sacral cornu.ans
B)sacral promontory
C)c7 spine
D)ishial tuberosity
E)ischial spine

Q) Uterus pierce
A)uterosacral ligament
B)broad ligaments
C)levator ani
D)pubocervical ligament
E)none above..ans
Q) Warfarin
RCOG updated its Greentop guidelines on
thromboprophylaxis in 2015.
Key Points
* People on warfarin should be converted to
LMWH during pregnancy
* Very few exceptions to above. Mechanical heart
valves main one
* Warfarin causes a characteristic warfarin
embryopathy in 5% of foetuses exposed to

warfare between 6 and 12 weeks gestation


*Warfarin is safe to use when breastfeeding
* Women converted to LMWH from warfarin
during pregnancy should be changed back 5-7
days after delivery
* It inhibits vitamin K dependent clotting factors
(10,9,7,2)
With regard to warfarin therapy there are only 3
target ranges that are commonly used:
2.0 - 3.0 Most common used for DVT,PE & tissue
valve replacement treatment
2.5 - 3.5 Used in mechanical mitral valve
replacement & some aortic mech.valve
replacements
3.0 - 4.0 Used in mechanical valve replacement
where PE has occurred despite anticoagulation at
lower range
Warfarin Embryopathy
Hypoplasia of nasal bridge
Congenital heart defects
Ventriculomegaly
Agenesis of the corpus callosum
Stippled epiphyses
Q) A 26 year old patient known to have Group B
Streptococcus (GBS) on vaginal swab is being admitted for
elective C-section delivery. She is penicillin allergic. What
intrapartum antiobiotic treatment is advised?

No treatment.ans
IV Clindamycin 900 mg 8 hourly
IV Ciprofloxacin 750mg 12 hourly
IV Clarithromycin 500mg 12 hourly
Oral doxycycline 200mg 12 hourly
Antibiotic prophylaxis for GBS is not required for
women undergoing planned caesarean section in
the absence of labour and with intact
membranes.
If Intrapartum antibiotics for GBS are indicated
3g Benzylpenicillin should be administered as
soon as possible after the onset of labour and
1.5g 4 hourly until delivery.
Clindamycin 900mg should be administered to
those women allergic to

Q)

Ans A

Q)

Q) Which paired structure contain the uterian


arteries and veins?
1. Broad lig
2. Cardinal lig ..ans
3. Pubocervical lig
4. Round lig
5. Uterosacral lig

Q)

Sertoli cells serve a number of functions during


spermatogenesis, they support the developing gametes in the
following ways:
Maintain the environment necessary for development and
maturation, via the blood-testis barrier
Secrete substances initiating meiosis
Secrete supporting testicular fluid
Secrete androgen-binding protein (ABP), which concentrates
testosterone in close proximity to the developing gametes
Testosterone is needed in very high quantities for maintenance
of the reproductive tract, and ABP allows a much higher level

of fertility
Secrete hormones affecting pituitary gland control of
spermatogenesis, particularly the polypeptide hormone,
inhibin
Phagocytose residual cytoplasm left over from spermiogenesis
Secretion of anti-Mllerian hormone causes deterioration of
the Mllerian duct[11]
Protect spermatids from the immune system of the male, via
the blood-testis barrier
Q)

Q)

Q) Herpes,chancroid =painful
Syphilis (chancre)=painless
Q) What is the incubation period of Rubella (in days)? 2-5
4-11
7-14
12-23.ans
21-28

Q) A 29 year old women wants to speak to you regarding


infections in pregnancy. Her two year old son has
sensorineural deafness as a result of infection in her previous
pregnancy. She tells you he was born with a "blueberry
muffin" rash. What was the most likely infection?
Toxoplasmosis
Parvovirus B19

Rubella.ans
Varicella Zoster
CMV
if only sensorineural hearing loss --> CMV
sensorineural hearing loss + blueberry muffin rash--> Rubella
Q)

Ans C
Q)

Ans ? E
Q) Which v the following is not a derivative of
vitelline vein

A)inferior mesenteric vein.ans


B)superior mesenteric vein
C)hepatic vein
D)portal vein
E)lower inferior vena cava

Q)

Ans E

Q) Which antiretrovial drug cause lactic


acidosis??

Q)

Q) Regarding shoulder dystocia which one is


true ?
A- recurrence is 50%
B half of all dystocia cases occur when babies
have normal birth weight ans
C- macrosomia can be reliably predicated
antenatally
D early IOL in suspected cases has been shown
to reduce risk of dystocia
E size of pelvic outlet in association with
perineum contributes to mechanism of dystocia
Q)

Ans C
Q) What percentage of infants with congenital CMV infection
are symptomatic?>99%

50%
10%.............ans
1-2%
<1%

Q)Stages of syphilis :
Q)

Q) chicken pox and rubella both got same


incubation period of around 14 days ...
Q) The H1n1 virus : Is influenza b ..ans
Transmitted from pigs to human
Undergo antigenic shift
Doesnt go antigenic shift
Infection doesnt produce antibodies
Q) which doesnt cross placenta? IgG1
igG2.ans
igG3
IgG4
Q) You are asked to review a 65 year old ladies legs preoperatively. You diagnose cellulitis. What is the most common
causative organism?
Staphylococcus Aureus
Staphylococcus Epidermidis

Clostridium Perfringens
Streptococcus Mutans
Streptococcus Pyogenes..ans
Group A streptococcus AKA Streptococcus Pyogenes is the
most common cause of cellulitis.
Staph. aureus is the second most common
Staph. epidermis can form biofilms on catheters/implants
Clostridia Perfringens causes gas gangrene
Strep. Mutans causes tooth decay and dental cavities
Group A Streptococcus can also cause Tonsillitis
(strep.throat), Scarlet fever and Rheumatic fever.
Candidates should also be familiar with Group B
Streptococcus AKA Streptococcus Agalactiae which can cause
neonatal sepsis.
Q) A patient is seen in the sexual health clinic. Speculum
examination reveals a firm 12mm ulcerated lesion with
smooth edges to the cervix. The patient denies any pain. What
is the likely cause?
Herpes Simplex Type 1
Herpes Simplex Type 2
Neisseria gonorrhoeae
Chlamydia Trachomatis D-K
Treponema pallidumans

Q) A 27 year old patient is maintained on warfarin during


pregnancy due to a mechanical mitral valve. She has read
about warfarin embryopathy. Which of the following is a
typical feature? Aplasia cutis
Stippled epiphyses..ans
Ototoxicity
Renal dysgenesis
Omphalocoele
Q) A 35 year old lady is seen in clinic for IVF counselling.
She reports having bloody watery diarrhoea for the past 5
days and a fever up to 39oC. A day or two earlier she had
takeaway chicken that tasted "funny". Which gram negative
rod is likely to be responsible?
Salmonella
Helicobacter
Listeria
Camplylobacter..ans
Escherichia coli
Q) Which one is not associated with cord prolapse
?
A pre term delivery
B fetal abnormality
C OT positionans
D internal podalic version
E ARM

Q)

Ans C

Q) assuming normal renal function ,after how


many weeks plasma volume return to normal ?

Q) Osmolarity of the blood is more or urine .F


Q) Which white blood cell type is elevated in an acute
cytomegalovirus infection?
Neutrophils
Monocytes
Lymphocytes..ans

Eosinophils
Basophils
Q) Which of the following is best explanation for
breast development in apatient with androgen
insensitivity?
A . gonadal production of estrogen.
B . adrenal production of estrogen.
C.Breast tissue sensitivity to progesterone.
D . peripheral conversion of androgen. .ans ?
E . autonomous production of breast specific
estrogen.
Q) Surface marker of T cell -A.CD3.ans
B.Ig
C.CD4
D.CD8
T cell CD 4 for helper
CD 8 for cytotoxic

Q) Mc trisomy in miscarried fetuses ? T 16

Compatible is 21
Non compatible is 16
Q) What is the incubation period of Scarlet Fever (in days)?
1-7.ans
3-11
7-14
14-21
14-28
Q) What is the mode of action of Mefenamic acid?
Inhibits Prostaglandin Synthesisans
Activates Antithrombin III
Inactivates factor Xa
Inhibits fibrin
Inhibits Plasminogen Activation

Q) Luteoplacental shift occurred in :


1. 12 wks
2. 16 wks
3. 8 wks ans,6-8 w
4. 20 wks

Q) side effects of ACE inhibitor and ARB


(angiotensin II receptor blocker) use in pregnancy
- IUGR and renal dysgenesis
Q) Bloody diarrhea without food poisoning
salmonella ...Bloody diarrhea with food
poisoning camphylobacter
Q) All of the following are stone formation inhibitors except
A ) magnesium
B) syprophosphate
C) calcium.ans

Q) A 24 year old patient who is 14 weeks pregnant has her


urine dipped during an antenatal visit. This shows leucocytes
++ and nitrites ++. You suspect a UTI and send a urine sample
for culture. According to NICE guidance which of the
following is most appropriate treatment option?
No treatment until culture result received
Trimethoprim 200mg BD for 3 days
Trimethoprim 200mg BD for 7 days
Nitrofurantoin 50mg QDS for 7 days.ans
Amoxicillin 500mg TDS for 7 days
NICE guidance on UTI in pregnancy was updated in July
2015

The following is advised:


Send urine for culture and sensitivity from all women in
whom UTI is suspected before starting empirical antibiotics
and 7 days after antibiotic treatment is completed.
Prescribe an antibiotic to all women with suspected UTI
(awaiting culture result is not advised)
Although local antibiotic resistance needs to be taken into
account the following is advised in terms of antibiotic
selection:
1. Nitrofurantoin 50 mg QDS (or 100 mg MR BD) for 7 days.
2. Trimethoprim 200 mg twice daily, for 7 days
Give folic acid 5 mg OD if it is the 1st trimester
Do not give trimethoprim if the woman is folate deficient,
taking a folate antagonist, or has been treated with
trimethoprim in the past year.
3. Cefalexin 500 mg BD (or 250 mg 6qds) for 7 days
Q) Which lung volume is decrease in pregnancy ?
A peak flow rate
B tidal volume
C vital capacity
D functional residual capacity ans
E Forced expiratory volume

Q) There are different types of fetal haemoglobin produced


throughout the gestational period. What is the primary
haemoglobin type at 32 weeks of gestation? A-Hb A
B-Hb F .ans
C-Hb Gower 1
D-Hb Gower 2
E-Hb Portland

Q) Nerve root of ilioinguinal and iliohypogastric nerve ?


A T12
B T12 and L1 .ans
C L1
D L2
E L1 and L2

Q) Acidosis is associated with hyperkalaemia and


alkalosis with hypokalemia .T
Mneumonic : Alk Low K+
Q) Sympathetic supply to the bladder is derived from which
lumbar segments? A- L1 and L2
B-L2 and L3
C-L3 and L4
D-L4 and L5
E-S2 , S3 and S4

Q) Changes in the nucleotide sequence of DNA which arent


passed to offspring occur in....? A. Eggs & sperm cells
B. Non-sexual cellsans
C. Diploid and haploid cells
D. All of the above

Q) a woman seek to conceive, she had history of treatment


from Chlamydia before 2 months, now she complain of right
iliac fosse pain . u/s report was: empty normal size uterus
4mm in thickness in upper corner there is 1534 cm (not
sure about number),normal ovaries ,little fluid in right
adenexia. Diagnosis is......sep2013

a-right hydrosalpinx ..ans


b- ovarian cyst
c-ectopic pregnancy
d-uterine carcinoma
e-appendicitis
Q)In the process of fertilisation, what is the correct sequence
for sperm penetration into the oocyte?
A- Corona radiata, zona pellucida, perivitelline space, plasma
membrane..ans
B- Corona radiata, zona pellucida, plasma membrane,
perivitelline space
C-Zona pellucida, corona radiata, plasma membrane,
perivitelline space
D- Zona pellucida, perivitelline space, corona radiata,
perivitelline space
E-Zona pellucida, perivitelline space, plasma membrane,
corona radiata

Q) Kaposi sarcoma is caused by: HSV1


HSV 2
HSV 4
HSV 8ans
Q) Which body fat has a major role in gene
transcription
A eicosanoids
B fatty acidsans
C membrane phospholipids
D phospholipids
E triglycerides
Q) Glandular fever is caused by : HSV 1
HSV 2
HSV 4.ans , EBV is HSV 4 ,Glandular fever is
the same (infectious mononucleosis)
HSV 8

Q) With regard to oxygen dissociation curve


which factor shift the curve to left ? A alkalosis
.ans
B anemia
C heat
D hypercapnia
E hypoxia

Q) 28 year old lady P3+2 presented to the clinic with history


of irregular menstrual cycle for 2 months, no history of postcoital bleeding. She had pap smear which showed high grade
squamous cell intraepithelial lesion (CIN II). The proper
management is?
A. Follow up & repeat pap smear aer 6 months.
B. Laser cone biopsyans
C. Examination under anesthesia & (D&C)
D. Colposcopic assessment & pelvic biopsy.
E. Cryotherapy & antibiotic
Q) Group B Strep. was found in high vaginal swap in 26 week
primi, what is next action: ?
- Antibiotic for 1 week
- Advice antibiotics perinataly to prevent neonatal septicemia
- After birth IV AB for 1 week
- Repeat swap at 37 weeksans
Q) A 36-year-old woman underwent a total
abdominal hysterectomy for uterine fibroids.
Postoperatively she complains of loss of flexion of
her left hip and numbness over her left anterior
and medial thigh. The compression of which
nerve is likely to be responsible? A. Common
peroneal nerve
B. Femoral nerve..ans
C. Ilioinguinal nerve

D. Obturator nerve
E. Pudendal nerve
Q) When does Oocytogenesis complete? 6 weeks gestation
18 weeks gestation
Birth.ans
Puberty
Ovulation

Q) What is the innervation to the 4th part of duodenum? T5T9


T10/T11.ans
T12/L1
L2/L3
L3/L4
foregut --> T5-T9
midgut --> T10 - T11
hindgut --> T12-L1
Q) Hyponatremia is a recognized complication of
which of the following..
Carbenoxolone therapy
Cereberal contusion.ans
Diabetes insipidus

Polyuric phase of acute renal faulure


Mahor burns
Q)

Ans B

Q)

Ans A
Q) You are asked to review a patient. They have attended for a
scan at 13 weeks following a positive pregnancy test. The
patient has had 2 previous pregnancies for which she opted for
termination on both occasions. The scan shows no identifiable
fetal tissue or gestational sac and you note the radiologist has
reported a 'bunch of grapes sign'. What is the likely diagnosis?
Missed miscarriage
Incomplete miscarriage
Partial molar pregnancy
Complete molar pregnancy.ans
Choriocarcinoma

Ultrasound features of :
complete hydatidiform mole:
-Solid collection of echoes with numerous small anechoic
spaces (snowstorm or granular appearance).
-Bunch of grapes sign which represents swelling of
trophoblastic villi.
-Normal interface between abnormal trophoblastic tissue and
myometrium.
-No identifiable fetal tissue or gestational sac.
Ultrasound Features of Partial Hydatidform mole:
-Enlarged placenta with multiple diffuse anechoic lesions
-Fetus with severe structural abnormalities or growth
restriction
-Oligohydramnios or deformed gestational sac
Q) A couple have been referred to the infertility clinic. The
male partner Is 25 and has CF, his semen analysis shows
azoospermia. What is the likely underlying cause?
Epididymal obstruction by thickened secretions due to
chloride channel dysfunction
Oligospermia
Congenital absence of vas deferens..ans
Obstruction of vas deferens by thick secretions due to chloride
channel dysfunction
Congenital absence of epididymal tail
In CF the thickened intraluminal secretions lead
to progressive obstruction and destruction of the

vas deferens in utero leading to congenital


absence of the vas deferens in most cases.
In treating such patients for infertility sperm can
be retrieved from the epididymis.
Q) Which of the following plays an important role in the
adaptive immune system?Basophils
Dendritic cells
Macrophages
Natural Killer Cells
CD4+ T cell.ans
Q) Which type of ovarian cancer is responsible for the
majority of cases of ovarian cancer in the UK and also the
most cancer deaths? Epithelial ovarian cancers
Germ cell tumours
Borderline ovarian tumours
Teratoma
Dysgerminoma
Malignant primary ovarian tumours can be
broadly classified into 3 types:
1. Epithelial
2. Germ Cell
3. Sex Cord and Stromal
Epithelial Ovarian Cancers (EOCs) are the most
common type with high grade serous ovarian
carcinomas the most common subtype. EOCs

comprise:Serous (68%)
Clear cell (13%)
Endometrioid (9%)
Mucinous (3%)
5 year survival is 43%
Lifetime risk is 1 in 70
Germ cell tumours account for 1-2% of ovarian
cancers
Sex Cord and stromal cancers are rare.
Q) Following are DNA viruses ? A hep A
B hep B
C HIV
D HRV 8
E VZV
Ans BDE,rest are RNA
Q) Cabergolin: Has long half life.ans
Is an effective antiemetic
Is used during pregnancy
May cause parkinsonian SE
Q) Cell cycle : S is the nucleic acid synthesis
S is the dna replication
The apoptotic cell is removed from macrophages
Apotosis is downregulated by p53
Bax family promotes apoptosisans
Bax family promotes apoptosis..
Bcl 2 family inhibits..

Q) Catecholamines from adrenal medulla increase when :


Aprox 1 hr following a MI
During sleep
Following increase in blood sugar
When nerves to adrenal gland are stimulated .ans
Following episode of hypertension
Q) Krebs cycle Produce : 1) 2 ATP 6 NADH 2 FADH
2) 2 ATP, 8 NADH, 4 FADH.ans
3) 4 ATP, 8 NADH, 2 FADH

Q)

Ans C, glycogenolysis, gluconeogenesis and ketone


generation

Q) Which of foll is a Gm -ve obligate anaerobe?


Bacteroidesans
Clostridia
Chlamydia
Escherichia coli
Salmonella
Examples of obligately anaerobic bacterial
genera include Actinomyces, Bacteroides,
Clostridium, Fusobacterium, Peptostreptococcus,
Porphyromonas, Prevotella and Veillonella.

Q) Which complement protein is the product of all 3


activation pathways--C20
C7
C3bans
C1a
C2a

Q)

Ans A
Ovulation occurs 36 h after LH surge ans 1626 h
after peak of LH
Q)
1. Q) Miscarriage
1. Fetal loss before 20 weeks Gestational age
2. Early Stillbirth
1. Fetal loss between 20-27 weeks Gestational age (or
fetal weight 350 grams or less)
3. Late Stillbirth
1. Fetal loss between 28-36 weeks Gestational age
4. Term Stillbirth
1. Fetal loss between 37-40 weeks Gestational age

Q) What epithelium cell type lines the endometrium?


Columnarans
Cuboidal

Stratified Squamous
Pseudostratified Squamous
Transitional

Q)

Ans c
Q) Fetal lung maturation takes place in A)stage one
B)stage two
C)stage three
D)stage fourans
E)stage five
5 stages of lung development 1st embryonic with early
buds,second pseudoglandular upto 17 weeks, third canalicular
upto 25 weeks surfactant production starts,fourth saccular
upto 36 weeks surfactant present mature structure of lungs,last
alveolar it continues to birth and in early childhood

Q) Chromosome 21 which one is true ? A-2nd largest


chromosome
B- is submetacenteric

C carries gene for growth hormone


D includes in group B of chromosomes
E has a nuclear organizer.ans
Genes for human growth hormone, known as
growth hormone 1 (somatotropin) and growth
hormone 2, are localized in the q22-24 region of
chromosome 17

Q)

Q) Which of the following is in direct contact with


maternal blood in lacunae of the placenta?
a. Cells of the cytotrophoblast

b. Extraembryonic mesoderm
c. Fetal blood vessels
d. Cells of the syncytiotrophoblast..ans
e.Amniotic cells
In the developing fetus, the maternal blood is in
direct contact with the syncytiotrophoblast.
During implantation, the syncytiotrophoblast
invades the endometrium and erodes the
maternal blood vessels. Maternal blood and
nutrient glandular secretions fill the lacunae and
bathe the projections of syncytiotrophoblast.
Primary villi consist of syncytiotrophoblast with a
core of cytotrophoblast cells. In secondary villi,
the cytotrophoblast core is invaded by mesoderm
and subsequently by umbilical blood vessels in
tertiary villi.

Q) A baby had bradycardia and was delivered by


ventouse.The umbilical artery pH wad7.05, base
excess -4, while the umbilical vein pH was 7.25,
base excess -2.5. What will you tell the
pediatrician about the test.Normal blood test
The baby had an acute episode of hypoxia before
delivery
The baby had chronic episode of hypoxia before
delivery.ans
Q) Nerve pierce Rectus sheath?Subcostal

Q)

Ans C,avoid in 1st trim

Q) A 29 year old women wants to speak to you regarding


infections in pregnancy. Her two year old son has
sensorineural deafness as a result of infection in her previous
pregnancy. She tells you he was born with a "blueberry
muffin" rash. What was the most likely infection?
Toxoplasmosis
Parvovirus B19
Rubella..ans
Varicella Zoster
CMV

Q) The yolk sac reaches its maximum diameter at what week


of gestation?6 weeks
10 weeks..ans
16 weeks
24 weeks
38 weeks
Q) Macrophages are derived from what type of white blood
cell? Natural Killer Cells
T-cells
B-Cells
Monocytesans
Neutrphils

Q) SLE is an example of what type of hypersensitivity


reaction? 3

Q) You see a patient in antenatal clinic who is concerned that


she has never had chicken pox and may catch it during
pregnancy. You check her Varicella status and she is nonimmune. She asks you about vaccination. What type of
vaccine is the varicella vaccine?
Attenuated..ans
Inactivated
Subunit
Polysaccaride
Toxoid

Q) An MRI examination is NOT allowed under any


circumstances when the following is present
A. A pacemaker..ans
B. A hip/knee joint replacement
C. An intracranial aneurysm clip
D. A metallic heart valve
E. A first-trimester pregnancy

Q)

Q) endometrium -from endoderm


myometrium- from paramesonephric duct
Myometrium, uterus and tubes are formed of Intermediate
mesoderm -> paramesonepheric ducts

Q) TYPES OF FIBROID DEGENERATION


Fibroids degeneration occurs when fibroids overgrow their
blood supply and slowly die afterwards. These degenerative
changes of the fibroids tissue are common consequence of the
rapid growth, pregnancy, trauma, and postmenopausal
atrophy.
1. Hyaline Fibroids Degeneration
Hyaline degeneration is the most common type of fibroids
degeneration that can occur in 60% of all fibroids cases. The
most common change is replacement of the fibrous and
muscle fibroid tissues with the hyaline tissue (type of
connective tissue). Gradual decrease** in the blood supply to
the fibroids can cause this type of fibroids degeneration.
Although, hyaline fibroids degeneration is without symptoms,
it can cause central necrosis** (death of the cells and tissues)
and leave cystic spaces at the center. In this way, cystic
fibroids degeneration starts.
2. Cystic Fibroids Degeneration
Cystic degeneration is not so common type of fibroids
degeneration; it affects only 4% of all fibroids and usually
occurs after menopause**. As already mentioned, hyaline
degeneration often precedes cystic degeneration.
Decreased blood supply to the fibroids may influence
liquefaction of hyalinized areas that are seen as cystic changes
on the ultrasound. They resemble honeycomb pattern, and
sometimes can be misleading, especially with the submucous
type of fibroids. They are often misdiagnosed as other
gestational abnormalities, such as missed abortion, blighted
ovum, and hydatidiform mole.

3. Myxoid Fibroids Degeneration


This type of fibroids degeneration is not as common, though
some doctors think that myxoid degeneration is present in, as
much as, 50% of all fibroids.
4. Red (carneous) Fibroids Degeneration
This type of fibroids degeneration is common during
pregnancy or after pregnancy***. It is a well-known
complication especially during pregnancy. Red degeneration
occurs in 8% of fibroids complicating pregnancy, although the
prevalence is about 3% of all uterine leiomyoma.
Red fibroids degeneration is the hemorrhagic infraction of
uterine fibroids.** The exact mechanism of red degeneration
is not completely understood, but scientists believe that it
begins with the venous obstruction at the periphery of the
lesion, which leads to hemorrhagic infarction and extensive
necrosis that involves the entire lesion.
Red fibroids degeneration affects half of the fibroids during
pregnancy; its quite common. Fibroids during pregnancy
have the tendency to rapidly grow, because of the higher
levels of estrogen. They soon overgrow their blood supply and
start to decay.
The symptoms characteristic for the red fibroids degeneration
may include abdominal pain, tenderness localized to the
uterus associated with mild pyrexia and increased white blood
cells count (leukocytosis).

Q)

Q) CMV is most commonly transmitted by which


route?Breast feeding
Q)

Ans D
Q) A couple come to see you in clinic. The male partner
reveals he has cystic fibrosis and wants to know what his
likelihood of being infertile is. What is the male infertility rate
in CF patients? 98%..........ans

90%
75%
40%
18%
Q) A 26 year old patient who is currently 24 weeks pregnant
presents with vaginal discharge. Swabs show Chlamydia
Trachomatis detected. Which of the following is the most
appropriate treatment regime? Doxycycline 100mg bd for 7
days
Erythromycin 500 mg twice a day for 14 days
Azithromycin 1gm orally in a single dose
Ofloxacin 200mg bd for 7 days
Ofloxacin 400mg once a day for 7 days
the following treatment regimes are recommended for the
treatment of Chlamydia in PREGNANCY:
Erythromycin 500mg four times a day for 7 days or
Erythromycin 500 mg twice a day for 14 days or
Amoxicillin 500 mg three times a day for 7 days or
Azithromycin 1 gm stat (only if no alternative, safety in
pregnancy not fully assessed)
The following treatment regimes are recommended for the
treatment of Chlamydia in NON-PREGNANT patients:
Doxycycline 100mg bd for 7 days OR
Azithromycin 1gm orally in a single dose

Q) Patent ductus arteriosus


In preterm infants closure can be achieved by NSAIDs
typically indomethacin but ibuprofen could be used.
In term infants surgical closure is indicated. Prostaglandin
infusion may be used in this situation but this is to keep the
DA patent until surgery.

Q) X-ray: which is incorrect


A, are a form of electromagnetic radiation
B, are longer than gamma rays
C, are measured in candelaans
D, are very energy inefficient to produce
E,are formed by acceleration of electrons

C: Measured by millirad
One session = 1000 millirad
CT scan from 40-400 millirad
MRI : no radiation
Q)

Ans A

Q) Regarding MRI which statement is incorrect


Provides high contrast between different soft tissues
Produces the same amount of ionizing radiation as CT
scan..ans
Affects all protons in the body
Works by emitting a radio frequency pulse as protons move
between magnetic fields
Is used to characterize pathological tissue
Q)Foll substances can be reabsorbed from renal
tubule ?
A insulin
B sucrose
C creatine
D mannitol
E urea..ans
Q) True regarding Obturator Nerve?
Emarge from lateral border of psoas major
Has branch separated by Adductor
Brevis..ans
Is formed by post devision of Lumber nerves
Lies medial to the ureter
Supplies lateral knee joint
Q) A 30 year old women who is 24 weeks pregnant attends
EPU due to suprapubic pain. Ultrasound shows a viable foetus
and also a fibroid with a cystic fluid filled centre. What is the
likely diagnosis? Adenomyosis
Leiomyosarcoma
Cystic degeneration of fibroid

Red degeneration of fibroidans


Appendicitis
Red degeneration of fibroids is one of 5 methods
of fibroid degeneration.
Although uncommon outside pregnancy it is
thought to be the most common form of fibroid
degeneration during pregnancy and typically
occurs in the 2nd trimester.
It is thought to arise from the fibroid outgrowing
its blood supply and haemorrhagic infarction
occurs. Ultrasound will typically show a localised
fluid collection (blood) within the fibroid.
Q) Following is an action of cortisol ? A-analgesia
B dec glycogenesis
C Sec. Catabolism of proteins
D dec gastric acid production
E increase gluconeogenesis.ans
Q) What type of acidbase disturbance result in a
case of Conn's syndrome?
A. Metabolic acidosis
B. Metabolic alkalosis ..ans
C. No effect
D. Respiratory acidosis
E. Respiratory alkalosis

Q) Which HLA type is expressed only in extra


villous trophoblast?A)HLA A
b)HLA B
C)HLA C
D)HLA F
E)HLA G.ans
Q) Complete heart block in newborns is
associated with maternal?
A. Cyanotic heart disease.
B. Endemic goitre.
C. SLE.ans
D. Myasthenia gravis.
E. Diabetes mellitus.
SLE causes neonatal cutaneous lupus in 5% of
cases and complete heart block in 1-2% of cases
in pts with anti Ro la antibodies
Q) 69 year old lady has a stroke on the ward 4 days after
hysterectomy. What type of necrosis typically occurs in
cerebral infarction? Fat necrosis
Coagulative necrosis
Caseous necrosis
Granulomatous
Liquefactive necrosisans
Q) Closure of a patent ductus arteriosus in a term infant
should be via?
Indomethacin administration
Ibuprofen administration
Prostaglandin infusion

Oxytocin infusion
Surgical closure..ans
Q) Regarding gestational diabetes which of the following is
NOT a recognised risk factor
Smoking
Obesity
Hispanic/Latino ethnic origin
Previous unexplained stillbirth
High polyunsaturated fat intake..ans
Q) A 55 year old presents to clinic due to vulval itch and
discolouration. examination reveals pale white discoloured
areas to the vulva. A biopsy shows epidermal atrophy with
sub-epidermal hyalinization and deeper inflammatory
infiltrate. What is this characteristic of?
Lichen Simplex Chronicus
Vulval intraepithelial neoplasia
Vitiligo
Extramammary Pagets
Lichen Sclerosus..ans
Q) A patient comes to see you as she is considering pregnancy
and wants advice regarding VTE prophylaxis. She has
undergone private thrombophilia screening as her father was
found to have the factor V Leiden. Her PCR has shown she is
homozygous for the factor V Leiden mutation. What is her
relative risk of thrombosis compared to the general
population? 1.5
2
4

8
80.ans
Q) A couple present to the fertility clinic after failing to
conceive despite trying for 2.5 years. The semen analysis
shows azoospermia.You perform a full examination of the
male partner which reveals Height 190cm, BMI 20.0, small
testes and scant pubic hair. What is the likely diagnosis?
Cystic Fibrosis
Autoimmune orchitis
Klinefelters Syndrome.ans
Marfans
Diabetic induced hypogonadsm
Q) Regarding prostaglandins : 1)Number of double bonds is
indicated by subscript numerical after the letter pg
2)thromboxanes are produced from placenta, membranes and
decidua
3) platelate cox is 20 times sensative to antiprostaglandin such
as aspirin as vessel wall enzyme
4)labetalol enhances the prostacycline to thromboxane ratio
5)Pg can maintain fetal pattern of circulation
All true
Q) Regarding Venous Thomboembolism (VTE) in pregnancy
which of the following statements are TRUE? A. Obesity
increases DVT risk by 2 times
B. Obesity increases DVT risk by 4 to 5 times.ans

C. Inherited thrombophilia is present is present in 3-5% of


cases of pregnancy associated VTE
D. Inherited thrombophilia is present is present in 10-15% of
cases of
pregnancy associated VTE
E. 30-40% of VTEs in pregnancy are Pulmonary Emboli
Q) A couple have been referred to the infertility clinic. The
male partner Is 25 and has CF, his semen analysis shows
azoospermia. What is the likely underlying cause?
Epididymal obstruction by thickened secretions due to
chloride channel dysfunction
Oligospermia
Congenital absence of vas deferens..ans
Obstruction of vas deferens by thick secretions due to chloride
channel dysfunction
Congenital absence of epididymal tail
In CF the thickened intraluminal secretions lead
to progressive obstruction and destruction of the
vas deferens in utero leading to congenital
absence of the vas deferens in most cases.
In treating such patients for infertility sperm can
be retrieved from the epididymis.
Q) To which group of lymph node does lymph
from rectum first pass
a deep inguinal
b inferior mesentric
c internal iliac

d para aortic
e superfecisl inguinal

Q)

Q)

Ans D

Q) which on of the following is true regarding


Pudendal nerve?
Arises from S3,4,5
Enter perinium through obturator canal
leaves the pelvis through greater sciatic foramen
lies medial to ischial spine
lies on sacrospinous ligaments.ans

Q) Septum secundum develops from : A)atrial


wall foldingans
B)from septum primum
C)ventricular wall infolding
D)fossa ovalis
E)none

Q) Regarding prostaglandins :which is true ..?


1)Number of double bonds is indicated by subscript numerical
after the letter pg
2)thromboxanes are produced from placenta, membranes and
decidua
3) platelate cox is 20 times sensative to antiprostaglandin such
as aspirin as vessel wall enzyme
4)labetalol enhances the prostacycline to thromboxane ratio
5)Pg can maintain fetal pattern of circulation
All true

Q)

Ans B
Q) 36yrs old lady PG at 14 weeks of gestation
,following serum screening ,pregnancy is found to
have an increased risk of trisomy 21,she wishes
to confirm if fetus is affected In view of her
gestation what's most appropriate diagnostic test
? A-amniocentesis
B -cell free fetal DNA sampling
C- CVS ..ans
D-cordocentesis
E NT imaging
> 15 w amnio
Q) Which of the following statements regarding
phenylketonuria is true?
Type 1
follows a mitochondrial inheritance pattern
Phenylalanine blood assay must be performed >48 hrs after
birth
Treatment requires dietary supplemention with phenylalanine
Sufferers have low plasma phenylpyruvic acid and
phenylethylamine levels
UK Incidence is 1 in 14,000..ans
Q) A 35 y has a pelvic ultrasound scan showing multiple
fibroids. What is the mc form of fibroid degeneration?
Red degeneration
Hyaline degeneration.ans
Carneous degeneration

Myxoid degeneration
Cystic degeneration
Q) A 42 year old smoker attends clinic due to vulval soreness
and shows you a number of vulval lumps. Biopsy is taken and
reported as showing epithelial nuclear atypia, loss of surface
differentiation and increased mitosis. What is the diagnosis?
Lichen Sclerosus
Chronic atrophic vulvitis
Vulval intraepithelial neoplasia (VIN)ans
Extramammary Paget's disease
Lichen Simplex
Q) A 35 year old women attends clinic following
laparotomy and unilateral oophorectomy. The
histology shows Psammoma bodies. What type of
tumour would this be consistent with?
Serousans
Mucinous
Endometrial
Transitional
Clear cell
Q) Which of the following is non essential AA? Aarginine
B-leucine
C-methionine
D tryptophan
E tyrosine.ans

Q) Which type of ovarian cancer is responsible for the


majority of cases of ovarian cancer in the UK and also the
most cancer deaths? Epithelial ovarian cancersans
Germ cell tumours
Borderline ovarian tumours
Teratoma
Dysgerminoma
Malignant primary ovarian tumours can be broadly classified
into 3 types: 1. Epithelial
2. Germ Cell
3. Sex Cord and Stromal
Epithelial Ovarian Cancers (EOCs) are the most common type
with high grade serous ovarian carcinomas the most common

subtype. EOCs comprise:


Serous (68%)
Clear cell (13%)
Endometrioid (9%)
Mucinous (3%)
5 year survival is 43%
Lifetime risk is 1 in 70
Germ cell tumours account for 1-2% of ovarian cancers
Sex Cord and stromal cancers are rare.

Q) CRL at which fetal heart sound can be seen using


ultrasound machine
A)>2mmans
B)>3mm
C)>4mm
D)>6mm
E)none above
Q) 1. Fetal Circulation : Hypogastric arteries
Fetal blood is returned to the umbilical arteries and the
placenta through the
a) Hypogastric arteries
b) Ductus venosus
c) Portal vein
d) Inferior vena cava
e) Foramen ovale

Correct Answer: a) Hypogastric arteries. Oxygenated


Blood from the placenta is carried to the fetus by the
umbilical vein. Deoxygenated Blood moves from the aorta
through the internal iliac arteries to the umbilical arteries, and
re-enters the placenta. In the fetus, the internal iliac artery is
twice as large as the external iliac, and is the direct
continuation of the common iliac.
It ascends along the side of the bladder, and runs upward on
the back of the anterior wall of the abdomen to the umbilicus,
converging toward its fellow of the opposite side.
Having passed through the umbilical opening, the two
arteries, now termed umbilical, enter the umbilical cord,
where they are coiled around the umbilical vein, and
ultimately ramify in the placenta.
At birth, when the placental circulation ceases, the pelvic
portion only of the umbilical artery remains patent gives rise
to the superior vesical artery (or arteries) of the adult; the
remainder of the vessel is converted into a solid fibrous cord,
the medial umbilical ligament (otherwise known as the
obliterated hypogastric artery) which extends from the pelvis
to the umbilicus.
2. Which of the following statements is true of the fetal
circulation?
a) 100% of the cardiac output goes to the lungs via the
pulmonary artery.
b) The arterial duct (ductus arteriosus) helps send oxygenated
blood to the brain.
c) Blood shunts across the duct left to right (from the aorta
to the pulmonary artery).
d) The foramen ovale typically closes by 36 weeks gestation.
e) The umbilical vein carries well-oxygenated blood.

Correct Answer: e) The umbilical vein carries oxygenated


blood from the placenta to the IVC via the ductus venosus.
Less than 10% of the cardiac output goes to the fetal lung.
Relatively deoxygenated blood is pumped from the right
ventricle, up the pulmonary artery and across the duct where it
joins the aorta after the carotid vessels, diverting this blood
away from the brain. In the fetus the pulmonary vascular
resistance is high and the systemic vascular resistance is low,
so blood shunts right to left. The foramen ovale sends welloxygenated blood across to the left atrium and only closes
after birth.
3. After birth, all of the following fetal vessels constrict
EXCEPT:
a) Ductus arteriosus.
b) Umbilical arteries.
c) Ductus venosus.
d) Hepatic portal vein.
e) Umbilical vein.
Correct Answer: d) Hepatic portal vein. At birth, placental
blood flow ceases and lung respiration begins. The sudden
drop in right atrial pressure pushes the septum primum against
the septum secundum, closing the foramen ovale. The ductus
arteriosus begins to close almost immediately, and may be
kept open by the administration of prostaglandins. Other
embryonic circulatory vessels are slowly obliterated and
remain in the adult only as fibrous remnants.
Fetal Structure >>> Adult Remnant
1) Foramen ovale >>> Fossa ovalis of the heart
2) Ductus arteriosus >>> Ligamentum arteriosum
3) Left umbilical vein
a. Extra-hepatic >>> Ligamentum teres

b. Intra-hepatic (ductus venosus) >>> Ligamentum venosum


4) Left and right umbilical arteries
a. Proximal portions >>> Umbilical br.of int iliac art
b. Distal portions >>> Medial umbilical ligaments
4. Fetal Circulation : Oxygenated Blood
In the fetus, the most well oxygenated blood is allowed into
the systemic circulation by the:
a) Ductus arteriosus.
b) Foramen ovale.
c) Rt. Ventricle.
d) Ligamentum teres.
e) Ligamentum venosum
Correct Answer: Left Ventricle via > Eustachian valve > b)
Foramen ovale > left atrium > aorta. Fetal cardiovascular
system is designed in such a way that the most highly
oxygenated blood is delivered to the myocardium and brain.
In the fetus, deoxygenated blood arrives at the placenta via the
umbilical arteries and is returned to the fetus in the umbilical
vein. The partial pressure of oxygen in the umbilical vein is
around 4.7 kPa and fetal blood is 8090% saturated. Between
5060% of this placental venous flow bypasses the hepatic
circulation via the ductus venosus (DV) to enter the inferior
vena cava (IVC). In the IVC, the better oxygenated blood
flow from the DV tends to stream separately from the
extremely desaturated systemic venous blood, which is
returning from the lower portions of the body with an Formula
of around 2540%. At the junction of the IVC and the right
atrium (RA) is a tissue flap known as the Eustachian valve.
This flap tends to direct the more highly oxygenated blood,
streaming along the dorsal aspect of the IVC, across the
foramen ovale (FO) and into the left atrium (LA). In the LA,

the oxygen saturation of fetal blood is 65%.1 This better


oxygenated blood enters the left ventricle (LV) and is ejected
into the ascending aorta. The majority of the LV blood is
delivered to the brain and coronary circulation thus ensuring
that blood with the highest possible oxygen concentration is
delivered to these vital structures.
Q) 1. The main blood supply of the vulva is: Gynecology
MCQ
a) Inferior hemorrhoidal artery.
b) Pudendal artery.
c) Ilioinguinal artery.
d) Femoral artery.
e) InferiorHypogastricartery.
Correct Answer: b) Pudendal artery. Blood supply to the
external genitalia is mainly from the pudendal artery. The
lateral aspects of the external genitalia receive their blood
supply from the external pudendal artery, a branch of the
femoral artery. The mons pubis is supplied by the inferior
epigastric artery, a branch of the external iliac artery.
2. The principle supports of the uterus are : Gynecology
MCQ
a) Iliosacral ligaments
b) Pyriformis muscle
c) Transverse cervical ligaments
d) Infundibular ligaments
e) Uterosacral ligaments
Correct Answer: c and e. The principle supports of the
uterus are the transverse cervical ligaments (cardinal
ligaments) and uterosacral ligaments. The infundibulopelvic

ligament is a fold of peritoneum that extends out from the


ovary to the wall of the pelvis.
3. The external iliac artery is crossed by: (Select FALSE)
a) The corresponding vein
b) The ovarian vessels
c) The genital branch of the genital femoral nerve
d) The round ligament
e) The ureter
Correct Answer: a) The corresponding vein is FALSE.
Both external iliac veins are accompanied along their course
by external iliac arteries. Posteriorly, the artery comes in
relation with its vein in the upper portion of its course. In the
lower part of its course the vein lies medial to it.
4. The uterine artery: (Select FALSE) : Gynecology MCQ
a) Is a branch of the anterior division of the internal iliac
artery
b) Runs in front of the ureter
c) Gives a branch to the vagina
d) May anastomose with the obturator artery
e) Divides into arcuate arteries
Correct Answer: d) May anastomose with the obturator
artery is FALSE. it anastomoses with the tubal branches of
ovarian artery.
5. The ovarian arteries : Gynecology MCQ
a) Arise just above the renal artery
b) Are crossed by the ureters
c) On the right cross the inferior vena cava
d) On the left cross the left colic artery

e) Reach the ovary through the ovarian ligament


Correct Answer: c) On the right cross the inferior vena
cava. Corrected FALSE statements: a) Arise just below the
renal artery
b) Ureters crosses ovarian vessels
d) On the left does not cross the left colic artery
e) After emerging from the aorta, the artery travels down the
infundibulopelvic ligament (suspensory ligament of the
ovary), enters the mesovarium, and may anastamose with the
uterine artery in the broad ligament.
6. The most common benign neoplasm of the cervix &
endocervix a) Polyp.
b) Hematoma.
c) Nabothain cyst.
d) Cervical hood.
e) Gartners duct cyst.
Correct Answer: a) Polyp. These are the most common
benign neoplasms of the cervix (found in 4% of the
gynaecological population). These may be endocervical or
cervical.
7. Which grp of nerves all constitute branches of pudendal
nerve?a) Internal pudendal nv, perineal nv, dorsal clitoral nv
b) Mid rectal nerve, dostal clitoral nerve, perineal nerve
c) Perineal nerve and posterior clitoral nerve
d) Perineal nerve, dorsal clitoral nerve, inferior rectal nerve
e) Perineal nerve, inferior clitoral nerve, mid rectal nerve

Correct Answer: d) Perineal nerve, dorsal clitoral nerve


and inferior rectal nerve. These are the three important
branches of the pudendal nerve.
8. Pelvic Diaphragm : Gynecology MCQ
The pelvic diaphragm is composed of all of the following
muscles EXCEPT -a) Iliococcygeus
b) Puborectalis
c) Transversus perinei
d) Pubococcygeus
Correct Answer: c) Transversus perinei.
The muscles of the pelic diaphragm primarily provide pelvic
support. These muscles form a basin or covering of the pelvic
outlet and are often grouped together as the levator ani. The
most medial portion of the pelvic diaphragm is formed by the
puborectalis. Forming the bulk of the pelvic diaphragm, the
Pubococcygeus and Iliococcygeus muscles cover the posterior
and lateral portion of the pelvic outlet. Transversus perinea
muscle is the part of urogenital diaphragm.
9. In a sagittal cross section of the pelvis
a) Urethra lies anterior to the upper third of the vagina
b) Urethra lies anterior to the lower third of the vagina
c) Bladder when empty lies below and anterior to the uterine
body
d) Bladder when empty lies parallel and anterior to uterine
body
e) Rectum lies posterior to the body of the uterus
Correct Answer: b,c,e. The urethra is only 3.5 cm long and is
anterior to the lower third of the vagina. The bladder when

empty lies below the uterovesical fold which arises from the
junction between the uterine body and the cervix.
10. Bartholins abscess : Gynecology MCQ
20-y woman presents with Bartholins abscess. Which one of
foll glands is involved?a) Bulbourethral glands (Cowpers)
b) Glands of skini
c) Greater vestibular gland
d) Lesser vestibular gland
e) Seminal vescicle
Correct Answer: c) Greater vestibular (Bartholins) gland.
These are small paired glands located in the superficial
perineal pouch. They are partially covered by posterior
portions of the bulb of the vestibule and the bulbospongiousus
muscles. The duct opens into the vestibule between the hymen
and the labium minus. These glands provide lubrication at the
introitus. Bartholins abscess may occur due to infection and
blockage of these glands.
11. Which artery is a direct branch of the aorta?
a) Inferior vesical
b) Internal iliac
c) Ovarian
d) Uterian
e) Vaginal
Correct Answer: c) Ovarian. The ovarian artery is a branch
of the aorta. It arises anterolaterally just below the renal
artery, running retroperitoneally to leave the abdomen by
crossing the common or external iliac artery in the
infundibulopelvic fold. It crosses corresponding ureters and
supplies twigs to it but does not supply to abdominal organs.

The internal iliac artery arises from the common iliac and its
inferior branch further supplies to the pelvis.
12. What is the lymphatic drainage of the ovaries?
a) Common iliac nodes
b) External iliac and superficial iliac nodes via the round
ligament
c) External iliac nodes
d) Internal iliac nodes
e) Lateral aortic and preaortic nodes
Correct Answer: e) Lateral aortic and pre-aortic nodes. It
is useful to remember the following: the bladder drains to the
external iliac nodes; the urethra drains to the internal iliac
nodes; the fallopian tubes and fundus uteri drain to the
external iliac and superficial iliac nodes via the round
ligament; and the cervix drains to the external and internal
iliac, rectal and sacral nodes and occasionally obturator nodes.
13. Which of the following structures lie within the broad
ligament?a) The fallopian tube
b) The ureter
c) The uterine artery
d) The ovarian artery
e) The superior vesical artery
Correct Answer:a and c.The broad ligament is made of two
layers of peritoneum that covers the fallopian tube, round
ligament, and down the sides of the uterus to the cervix.
The ureter, superior vesical artery and the ovarian artery are
all retroperitoneal.
The uterine artery runs between the leaves of the broad
ligament along the lateral wall of the uterus. It also contains
ovarian ligament and round ligament.

14. Pudendal nerve does not supply : Gynecology MCQ


a) Labia Minora
b) Ischiocavernosus
c) Posterior Fornix of Vagina
d) Urethral Sphincter
Correct Answer: c) Posterior Fornix of Vagina. The
pudendal nerve is a mixed motor and sensory nerve, and does
not carry parasympathetic fibres. The nerve has 3 branches:
the clitoral, perineal, and inferior hemorrhoidal. It supplies:
1) Clitoris.
2) Bulbospongiosus, Ischiocavernosus muscles and
Transeversus perinei muscles. Also to the skin of the inner
portions of labia majora, labia minora and vestibule. The
external urethral sphincter is controlled by the deep perineal
branch of the pudendal nerve.
3) External anal sphincter and perineal skin.
upper vagina parasympathetic fibres from pelvic splanchnic
nerves (S2-S4)
lower 2-3cm vagina pudendal nerve
15. Transverse vaginal sulcus corresponds to
a) Junction of urethra and bladder
b) Junction of cervix and vagina
c) Lower limit of rectum
d) Upper limit of bladder
Correct Answer: a) Junction of urethra and bladder. On
inspection of vagina, 3 grooves can be distinguished in the
anterior vaginal wall : 1) Submeatal sulcus, 2) Transverse
vaginal sulcus corresponds to the upper border of posturethral
ligament and 3) Bladder sulcus. Bladder sulcus indicates the

upper limit of the relation of the bladder to the anterior


vaginal wall.
16. Which of the following structures lies posterior to the
ischial spine? a) Pudendal nerve/ vessels
b) Sciatic nerve
c) Hypogastric venous plexus
d) Inferior gluteal vessels
Correct Answer: a) Pudendal nerve/ vessels. The sciatic
nerve lies superior and lateral to the sacrospinous ligament.
Superior to the ligament lies the inferior gluteal vessels and
the hypogastric venous plexus.The pudendal nerve passes
between the piriformis muscle and ischiococcygeus muscles
and leaves the pelvis through the lower part of the greater
sciatic foramen. It then enters the gluteal region, crossing over
the sacrospinous ligament near to where it attaches to the
ischial spine. Re-entering the pelvis through the lesser sciatic
foramen, it accompanies the internal pudendal vessels
upwards and forwards along the lateral wall of the ischiorectal
fossa, being contained in a sheath of the obturator fascia
termed the pudendal canal (Alcocks canal), along with the
internal pudendal blood vessels.
17. Location of Gartners cyst : Gynecology MCQ
a) Hymen
b) Vagina
c) Cervix
d) Labia majora
e) Broad ligament
Correct Answer: b) Vagina

A Gartners duct cyst is a benign vaginal cystic lesion that


arises from the vestigial remnant of a mesonephric duct or
Gartners duct. They are typically small asymptomatic cysts
that occur along the lateral walls of the vagina, following the
course of the duct. They can, however, enlarge to substantial
proportions and be mistaken for urethral diverticulum or other
structures.
18. McCalls Culdoplasty : Gynecology MCQ
Which of the following structures is at risk for injury during a
McCalls culdoplasty?a) Sciatic nerve
b) Rectum
c) Internal iliac vein
d) Pudendal nerve
e) Urinary bladder
Correct Answer: b) Rectum. Most common complication of
McCalls culdoplasty is laceration of bowel or rectum as
perirectal fascia is incorporated into the sutures.
19. The lower third of the vagina has lymphatic drainage
to which nodes? a) Common iliac nodes
b) External iliac nodes
c) Internal iliac nodes
d) Superficial inguinal nodes
e) Para-aortic nodes
Correct Answer: d) superficial inguinal nodes. Remember
the embryological difference between the lower one-third of
the vagina and the upper two-thirds. The upper part is derived
from the fused mesonephric ducts, and the lower one-third
from canalisation of the ectodermal thickening. The upper

two-thirds of the vagina drain to the internal and external


nodes.
20. Motor function of the uterus occurs via sympathetic
and parasympathetic nerves.Foll sentences: T/F.
1) Intact nerve supply is essential to initiate normal labour.
2) Normal labor occurs in patients with a transected spinal
cord.
Correct Answer: 1) False, 2) True. Uterine contractions are
involuntary and for the most part, indepependent of
extrauterine control. Myometrial contractions in paraplegic
women are normal, though painless, as in women with
bilateral lumbar sympathectomy.
21. The canal of Nuck is associated with what?
a) Broad ligament
b) Inguinal ligament
c) Ischiorectal fossa
d) Round ligament
e) Urogenital diaphragm
Correct Answer: d) Round ligament. The round ligament
arises from the body of the uterus antero inferior to the
cornue. It passes through the layers of broad ligaments across
the psoas and external iliac vessels. It then passes through the
deep inguinal ring and the inguinal canal to the labium majus.
In the fetus, the round ligament is surrounded by a
peritoneum, processus vaginalis, which is obliterated at birth
but may remain patent as the canal of Nuck.
22. Urinary Catheter : Gynecology MCQ

A urinary catheter is vigorously inflated in the urethra at the


time of cesarean section. Urine leaks from the rupture into
which anatomical space?
a) Into the anterior abdominal wall and mons pubis
b) Into the lesser pelvis
c) Into the peritoneal cavity
d) Into the vagina
e) Laterally to the femoral triangles
Correct Answer: a) Into the anterior abdominal wall and
mons pubis. This is the only route possible due to the
anatomy of the urethra.

Q) An obstetrician performs a mediolateral


episiotomy to expand the birth canal during a
child birth. Which of the following muscles is
typically incised during this
procedure?
A. Bulbospongiosus and superficial transverse
perineal muscles.ans
B. Bulbospongiosus and deep transverse perineal
muscles
C. Bulbospongiosus and ischiocavernosu muscles
D. Ischiocavernosus and levator ani muscles
E. Bulbospongiosus and levator ani muscles
Q) Regarding urinary tract infections, which
micro-organism is most likely to be associated

with bladder catheterisation? A. Bacteroides


B. Escherichia coli
C. Proteus mirabilis
D. Pseudomonas aeruginosa .ans
E. Staphylococcus saprophyticus
Q) Acute retention of urine in women may be due
to all of the following except - Preoperative
anxiety
Vulva herps
Multiple sclerosis .ans
Utrine prolapse
Retroverted uterus in pregnancy
Q) Long term consequences very radiotherapy
include all except - A)VAGINAL
NECROSIS..ans
B)VVF
C)VAGINAL STENOSIS
D)RVF
E)RECTAL STRICTURE

Q) how to calculate cardiac output of fetal heart :


a. foramen ovale + left ventricleans
b. Lt ventricle alone
c. Ductus arteriosus + aorta

Q) Antiphospholipid syndrome associated with EXCEPT :


a. IUGR.
b. Preterm delivery.
c. Miscarriage

d. Diabetesans
Q) Which of following produce severe hyperkaleamia in
combination - Aspirin n allopurinol
Furosemide n amiloride
Lisinopril n furosemide
Losartan n amilorideans
Propranol nverapamil
Q)which one of foll is the termination of round
ligament? Deep Inguinal ring
Labia majora..ans
Labia minora
Superficial Inguinal ring
Lateral vaginal wall
Q) Which of the following would you expect to see reduce in
pregnancy? Fibrinogen
ESR
Neutrophil count
Factor VII
Platelet count..ans
Q) Adenocanthoma of endometrium :
a. adenocarcinoma + benign squamus.ans
b. Benign + malignant squamus.
c. adenocarcinoma + papillary formation.
d. Glandular + squamus

Q)

Ans B
Q)

Ans B

Q) Foll substances can be reabsorbed from renal


tubule ?
A insulin
B sucrose
C creatine
D mannitol
E urea..ans
Q) Which ascending artery can be damaged
during open appendicectomy?-Iliolumbar artery
-Superficial circumflex artery
-Deep.circumflex artery..ans
-Sup gluteal artery

Q) X-ray: which is incorrect


A, are a form of electromagnetic radiation
B, are longer than gamma rays
C, are measured in candela..ans
D, are very energy inefficient to produce
E,are formed by acceleration of electrons
Measured by millirad
One session = 1000 millirad
CT scan from 40-400 millirad
MRI : no radiation
Q)

Q) Diffusion of gases thro placental membrane


which one is true ? A-mean PO2 in mother is
approx 30mmgh
B-mean PO2 in fetus is 50mmgh
C-only way fetus can excrete CO2 thro placenta
ans
D- co2 crosses placenta Coz of higher conc
E- co2 diffuses through placental membrane 5
times quicker than O2
Q)

Q)

Q) You are reviewing a patient who is complaining of pain


and numbness to the proximal medial thigh following
abdominal hysterectomy. You suspect genitofemoral nerve
injury. What spinal segment(s) is the genitofemoral nerve
derived from?T12,L1
T12
L1,L2ans
L2-L4
T12-L4
Q) After disinfection which organism survive?
Fungi
Enteribacter
Actinomytes
Serratia
Psudomonas And Spores
Q) What is the average oblique diameter of the pelvic inlet
according to the RCOG? 7.5 cm
9 cm
10.5 cm
12 cmans
13.5 cm
Q) At what age does physiological gut herniation occur ?
1 week
3 weeks
5 weeks
8 weeks..ans
12 weeks

Q) Ootidogenesis refers to which process during Oogenesis?


1st Meiotic Division
2nd Meiotic Division
1st and 2nd Meiotic Divisions.ans
Differentiation
Growth and maturation

Q) Mc organism of chorioamniotis in premature


labour is? GBS

Q)

Ans B

Q) Regarding MRI which statement is incorrect


Provides high contrast between different soft
tissues
Produces the same amount of ionizing radiation
as CT scan..ans
Affects all protons in the body
Works by emitting a radio frequency pulse as
protons move between magnetic fields
Is used to characterize pathological tissue
Q) Regarding MCMA twins which one is false A)account for 1% Of twins
B)occur 9 to12 days after splitting
C)Carry increased risk of cord entagelments
D)steroid cover should always be given before
ceasrean
E)should always be delivered by El c/s at 38
wans
Q)

Ans C

Q) If the omphalomesenteric duct (YS stalk) fails to close


properly a Meckel's diverticulum may form in the small
intestine. What percentage of the population are effected?
0.1%
0.5%
2%..................ans
5%
25%
Meckels follows the rule of 2s
2% of the population
2 inches long
2 feet from the ileocecal valve
2/3rds have ectopic mucosa
2 types of ectopic tissue ( gastric and pancreatic)
and 2% become symptomatic
Q) what percentage of female with malignancy
have hypercalcemia?1%
2%
5%
10%
20%................ans

Q)

Ans E
Q) The mc anomalies with IDDM is cardiac
anomalies (8.3 for 100 live birth) specially
transposition of great arteries and SVD and
neurological anomalies is 2nd common anomalies
(5.2 for 100 live birth )and the caudal regression
is the most specific anomalies with IDDM (200
times more frequent than other healthy infants)
Q) - 60 y F, smoker patient in ward for preop prep
for TAH to endometrial CA. Which investi of foll
you dont require?
-Chest X ray
-Complete blood film

-Coagulation profileans
- Blood for cross match and saving
-ECG
Q) DUB can be due to all except A)Inc PGE2 /PGF2@ ratio
B)Dec PGI2/TXA2 ratioans, inc PGI2/TXA2
ratio
C)Inc fibrinolysis
D)Dec endothelin
E)platelet deactivation
Q)

Ans A
Q)

Q)

Q) During oogenesis which cell type has the greatest number


of chromatids?Oogonium
Primary Oocyteans
Secondary Oocyte
Ootid
Ovum
Q) Regarding electrosurgery, what is the right
answer:
1)the electric current used in monopolar
diathermy has a lower frequency than the current
of the main supply
2)low frequency currents have no effect on cells
3)high frequency currents directly produce heat
4) bipolar diathermy does not allow
cuttingans
Q) Best site for giving Anti-D ? Deltoid
Q) What mechanism is required for myometrial
contraction?
-Electron transport and n ATP generation
-Oxidation,
Citric acid cycle
,Elecrtron transport chain..ans
Q) Prolonged and excessive bleeding at irregular
intervals
A)metrohagia
B)Meno metrorhagia..ans
C)menorrhagia

D)poly menorrhoea
E)oligo menorrhoea
Q) 27yrs old PG has failure to progress in first
stage of labour and is started on synto infusion
Midwife calls reg Coz of suspicious CTG the
woman is 6cm dilated and reg performs blood
sampling ,PH came as 7.15 ,what should be
course of action?
A C-section ..ans
B instrumental delivery
C reassure that all is well and cont with labour
D repeat FBS in one hour
E repeat FBS in two hours
7.25_ 7.4 normal
7.20_ 7.24 suspecious
Below 7.20 mean acidodis & need intervention
Q) Ultrasound can not be used for the following:
A, fetal therapy.ans
B, to determine the nature of a tissue
C, to assess the movement of tissues
D, to measure blood flow
E, to measure structure

Q)

Answer D. Rest depends on GLUT1


Q) At what stage in the cell cycle is mitosis
arrested if there is a chromosomal abnormality?G
2 check point
Q) Woman attends USS in early pregnancy, fetal
poles seen but heartbeat not visible. What is the
likely GA? 5 w
Q) The following may be used safely to accelerate labor,
except:Intravenous oxytocin
Ergometrine..ans
Prostaglandin
Rupture of membranes
Stimulation of the nipples
Q) Sertoli cells contain receptors to which hormone? Inhibin
Oestradiol
Testosterone

LH
FSH..ans
Summary points of the two key testicular cell types:
Sertoli Cells = Secrete Inhibin. Forms blood-testis barrier.
Have FSH receptors
Leydig Cells = Secrete testosterone. Have LH receptors
Q) In surgical infection, which of foll is true?
A ABics are only advised once a septic focus has been drained
B Primary refers to a planned surgical trauma
C There is rarely a focus
D Tissue necrosis is rarely an association
E Tissue necrosis results in inflammation.ans
Q) An anxious 38y pregnant undergoes a
combined test for Down syndrome ,risk comes as
1/1000.whats appropriate course of action?
A-advise that diagnostic test are not indicated
.ans ,low risk
B-amniocentesis
C-CVS
D-inform that baby doesn't have Down syndrome
E termination of pregnancy
Q) True statements about PIH include which of foll?
a. The incidence varies little around the world
b. Women who have had hypertension of pregnancy once have
a 10% chance of developing it in a later pregnancy

c. Elevations in systolic or diastolic blood pressures do not


become diagnostically significant until blood pressure values
reach 140/90 mmHg
d. Young primiparous women have the lowest incidence
e. Having a baby by a different father increases the risk of
preeclampsia in a multigravid woman.ans

Q) What is key histologic diagnostic feature of


chorioCA ?
a- An increase in
Cytotrophoplast
b- Decrease in syncytial trophoplast
c- Absence of cellular anaplasia
d- Absence of villous pattern..ans
Q) Whats in body secretions e.g cervical
secretions attack the bacterial lipoprotien?
-Immunoglobulin
-Lysozyme.ans
Q)3rd pharyngeal arch gives rise to which of foll structures?
Thrigeminal Nerve
Glossopharyngeal nerveans
Facial nerve
Vagus nerve
Muscles of mastication
1st Arch = Trigeminal V2 & V3 (CN V)
2nd Arch = Facial (CN VII)

3rd Arch = Glossopharyngeal (CN IX)


4th and 6th Arches = Vagus (CN X)
Q) HPV -A)consists of 5 group
B) low risk include subtype 6 and 11
C)high risk include subtype 16 and 18
D)only infect epithelial cells
E) RNA virus.ans
Q) Approximately how many oocytes are present in the
ovaries at birth?1,000
10,000
100,000
1 million..ans
10 million

Q)Rx of choice in a patient with a cystocele during preg is:


Anterior repair operation in the first trimester
Repair 2 weeks after delivery
Anterior repair operation in the second trimester
Immediate repair after delivery of the placenta
Re-evaluation 6 w after delivery.ans

Q)

Ans D&E
Q) The half life of the drug will be increased by:
A-increased volume of distribution.ans
B-increased rate of clearance.
C-increased age,for drugs eliminated mainly by
the kidney.
D-pregnancy for drugs metabolized in the liver.
Q) When a women comes for labor
a. Oxytocin drip is given at 1st stage labor.
b. Oxytocin drip is given at 2nd stage labor.
c. Oxytocin drip is given at 3rd stage labor.
d. Drip without oxytocin given at 1st stage.
e. None of the aboveans
Q) All of foll are possible indications for classical c/s , except:
Carcinoma of the cervix

Impacted shoulder presentationans


Severe adhesions in LUS
Large cervical fibroid
Posterior placenta previa grade II
Q) Which pharyngeal arch is closest to head of embryo 1st ..ans
2nd
3rd
4th
6th
The arches are numbered according to their
proximity to the head i.e the 1st is the closest to
the head end of the embryo and the 6th closest
to the tail end ..

Q)

Q)

Ans A
Q) ADH increases permeability of which part of the nephron?
Proximal convoluted tubule
Distal convoluted tubule
Collecting duct.ans
Descending loop of Henle
Ascending loop of Henle

Q) Gonorrhea infect mucus membrane all except


A)endocervix
B)rectum
C)ectocervix .ans
D)pharynx
E)conjunctiva
Q) Concerning hep E infection?
A-can be transmitted with hepatitis B
B-a recognized cause of chronic liver disease
C-CT scan of liver with contrast shows diagnostic
appearances
D- incidence of chronic liver disease is reduced
with administration of alpha interferons
E-it does not result in carrier stateans
Q) The embryonic life starts at:
A. The 3rd week after ovulationans
B. The 5th week after ovulation
C. The 7th week after ovulation
D. The 9th week after ovulation
E. The 11th week after ovulation

Das könnte Ihnen auch gefallen